cardiac

Réussis tes devoirs et examens dès maintenant avec Quizwiz!

The elderly client has coronary artery disease. Which question should the nurse ask the client during the client teaching?

"Are you sexually active?" - sexual activity is a risk factor for angina from coronary disease BUT client's being elderly should not affect teaching about sexual activity

A nurse and a nursing student are caring for a client with pericarditis and perform the physical assessment together. The client has a pericardial friction rub audible on auscultation. When the nurse and student leave the room, the student asks how to distinguish a pericardial from a pleural friction rub. The nurse's best response is which of the following?

"Ask the client to hold the breath while you auscultate; the pericardial friction rub will continue, while the pleural friction rub will stop."

The client with coronary artery disease asks the nurse, "Why do I get chest pain?" Which statement would be the most appropriate response by the nurse?

"Chest pain is caused by decreased oxygen to the heart muscle." - this stated in layman's terms - do not use medical terms - explain the correct procedure!

During a teaching session, a client who is having a valuloplasty tomorrow asks the nurse about the difference between a mechanical valve replacement and a tissue valve. Which of the following answers from the nurse is correct information?

"A mechanical valve is thought to be more durable and so requires replacement less often."

A nurse is teaching a client who is having a valuloplasty tomorrow. The client asks what the advantage is for having a tissue valve replacement instead of a mechanical valve. The correct answer by the nurse is which of the following?

"A tissue valve is less likely to generate blood clots, and so long-term anticoagulation therapy is not required."

The nurse is gathering data from a client recently admitted to the hospital. The nurse asks the client about experiencing orthopnea. What question would the nurse ask to obtain this information?

"Are you only able to breathe when you are sitting upright?"

The nurse determines that a 49-year-old client recently diagnosed with subacute bacterial endocarditis understands discharge teaching upon which client statement?

"I have to call my doctor so I can get antibiotics before seeing the dentist."

Which statement by the client diagnosed with coronary artery disease indicates that the client understands the discharge teaching concerning diet?

"I should bake or grill any meats I eat." - grill or bake do not fry - no more than 3 eggs a week, especially yolk - low fat milk, not whole milk - avoid pork products

A community health nurse teaches a group of seniors about modifiable risk factors that contribute to the development of peripheral arterial disease (PAD). The nurse knows that the teaching was effective based on which of the following statements?

"I will need to stop smoking because the nicotine causes less blood to flow to my hands and feet."

The male client is diagnosed with coronary artery disease (CAD) and is prescribed sublingual nitroglycerin. Which statement indicates the client needs more teaching?

"If my chest pain is not gone with one tablet, I will go to the ER." - keep in the dark - the tabs should burn on sting when put under the tongue - client should take 1 pill every 5 minutes and after the third pill still chest pain then call 911

A nurse working in the medical intensive care unit has a patient admitted with mitral stenosis. The nurse is precepting a new graduate and explains the pathophysiology of the condition. Which of the following statements made by the new graduate would reflect an appropriate understanding of the disease process? Choose all that apply.

"Increased blood flow in the left atrium causes left atrial hypertrophy." "There is a narrowing between the left atrium and left ventricle."

A patient with a recent myocardial infarction was admitted to the hospital with a new diagnosis of mitral valve regurgitation. Which of the following assessment data obtained by the nurse should be immediately communicated to the health care provider?

"The pericardial surfaces lose their lubricating fluid because of inflammation and rub against each other."

The nurse is assisting a patient with peripheral arterial disease to ambulate in the hallway. What should the nurse include in the education of the patient during ambulation?

"Walk to the point of pain, rest until the pain subsides, then resume ambulation."

A nursing student is assigned to a patient with a mechanical valve replacement. The patient asks the student, "Why do I have to take antibiotics before getting my teeth cleaned?" Which response by the nursing student is most appropriate?

"You are at risk of developing an infection in your heart."

The client diagnosed with a myocardial infarction asks the nurse, "Why do I have to rest and take it easy? My chest doesn't hurt anymore." Which statement would be the nurse's best response?

"Your heart is damaged and needs about four (4) to six (6) weeks to heal."

The nurse is caring for a client diagnosed with a myocardial infarction who is experiencing chest pain. Which interventions should the nurse implement? Select all that apply.

- Administer an aspirin orally. - Apply oxygen via a nasal cannula. PAY ATTENTION THE ROUTES! - MORPHINE ONLY IV - NITROGLYCERIN ONLY SUBLINGUAL IN THIS SITUATION

Which client teaching should the nurse implement for the client diagnosed with coronary artery disease? Select all that apply.

1. Encourage a low-fat, low-cholesterol diet. 2. Instruct client to walk 30 minutes a day. 3. Decrease the salt intake to two (2) g a day. WRONG! restricted in hypertension 4. Refer to counselor for stress reduction techniques. 5. Teach the client to increase fiber in the diet.

The client has chronic atrial fibrillation. Which discharge teaching should the nurse discuss with the client?

1. Instruct the client to use a soft-bristle toothbrush. - they will be on anticoagulants, risk for bleeding - scheduled INR test for therapeutic levels of warfarin, PTT for heparin - synchronized cardioversion for new-onset atrial fib not chronic

The nurse is completing a neurovascular assessment on the client with chronic venous insufficiency. What should be included in this assessment? Select all that apply. 1. Assess for paresthesia. 2. Assess for pedal pulses. 3. Assess for paralysis. 4. Assess for pallor. 5. Assess for paresthesia.

1. The nurse should determine if the client has any numbness or tingling. 2. The nurse should determine if the client has pulses, the presence of which indicates there is no circulatory compromise. 3. The nurse should determine if the client can move the feet and legs. 4. The nurse should determine if the client's feet are pink or pale. 5. The nurse should assess the feet to determine if they are cold or warm.

When administering heparin anticoagulant therapy, the nurse needs to make certain that the activated partial thromboplastin time (aPTT) is within the therapeutic range of:

1.5 to 2.5 times the baseline control

The intensive care department nurse is calculating the total intake for a client diagnosed with hypertensive crisis. The client has received 880 mL of D5W, IVPB of 100 mL of 0.9% NS, 8 ounces of water, 4 ounces of milk, and 6 ounces of chicken broth. The client has had a urinary output of 1480 mL. What is the total intake for this client?_______

1520 mL

The client diagnosed with a DVT is placed on a medical unit. Which nursing interventions should be implemented? Select all that apply. 2. Instruct the client to stay in bed and not ambulate. 3. Encourage fluids and a diet high in roughage. 4. Monitor IV site every shift and PRN.

2. Clients should be on bed rest for five (5) to seven (7) days after diagnosis to allow time for the clot to adhere to the vein wall, thereby preventing embolization. 3. Bed rest and limited activity predispose the client to constipation. Fluids and diets high in fiber will help prevent constipation. Fluids will also help provide adequate fluid volume in the vasculature. 4. The client will be administered a heparin IV drip, which should be monitored.

The nurse is developing a discharge-teaching plan for the client diagnosed with congestive heart failure. Which interventions should be included in the plan? Select all that apply.

2. Teach client how to count the radial pulse when taking digoxin, a cardiac glycoside. 3. Instruct client to remove the saltshaker from the dinner table.

The nurse is caring for clients on a telemetry floor. Which nursing task would be most appropriate to delegate to an unlicensed nursing assistant? 2. Assist feeding the client diagnosed with congestive heart failure.

2. The nursing assistant can feed a client.

The nurse is caring for a client receiving heparin sodium via constant infusion. The heparin protocol reads to increase the IV rate by 100 units/hour if the PTT is less than 50 seconds. The current PTT level is 46 seconds. The heparin comes in 500 mL of D5W with 25,000 units of heparin added. The current rate on the IV pump is 18 mL per hour. At what rate should the nurse set the pump?

20 mL per hour

The telemetry nurse is unable to read the telemetry monitor at the nurse's station. Which intervention should the telemetry nurse implement first?

3. Contact the client on the client call system.

The client diagnosed with a myocardial infarction is six (6) hours post-right femoral percutaneous transluminal coronary angioplasty (PTCA), also known as balloon surgery. Which assessment data would require immediate intervention by the nurse?

3. The client is complaining of numbness in the right foot. - indicate decreased blood supply to the right foot - do neurovascular assessment after PTCA - keep legs straight for 6-8 hours

A patient who had a prosthetic valve replacement was taking Coumadin to reduce the risk of postoperative thrombosis. He visited the nurse practitioner at the Coumadin clinic once a week. Select the INR level that would alert the nurse to notify the health care provider.

3.8

The client has just had a pericardiocentesis. Which interventions should the nurse implement? Select all that apply. 1. Monitor vital signs every 15 minutes for the first hour. 2. Assess the client's heart and lung sounds. 3. Record the amount of fluid removed as output. 4. Evaluate the client's cardiac rhythm.

71. $$1. The nurse should monitor the vital signs for any client who has just undergone surgery. $$2. A pericardiocentesis involves entering the pericardial sac. Assessing heart and lung sounds involves entering the pericardial sac and allows assessment for cardiac failure. $$3. The pericardial fluid is documented as output. $$4. Evaluating the client's cardiac rhythm allows the nurse to assess for cardiac failure, which is a complication of pericardial centesis.

6. A patient in the intensive care unit with ADHF complains of severe dyspnea and is anxious, tachypneic, and tachycardic. All these medications have been ordered for the patient. The first action by the nurse will be to a. administer IV morphine sulfate 2 mg. b. give IV diazepam (Valium) 2.5 mg. c. increase dopamine (Intropin) infusion by 2 mcg/kg/min. d. increase nitroglycerin (Tridil) infusion by 5 mcg/min.

A Rationale: Morphine improves alveolar gas exchange, improves cardiac output by reducing ventricular preload and afterload, decreases anxiety, and assists in reducing the subjective feeling of dyspnea. Diazepam may decrease patient anxiety, but it will not improve the cardiac output or gas exchange. Increasing the dopamine may improve cardiac output but will also increase the heart rate and myocardial oxygen consumption. Nitroglycerin will improve cardiac output and may be appropriate for this patient, but it will not directly reduce anxiety and will not act as quickly as morphine to decrease dyspnea. Cognitive Level: Analysis Text Reference: pp. 828-829 Nursing Process: Implementation NCLEX: Physiological Integrity

21. A patient with ADHF who is receiving nesiritide (Natrecor) asks the nurse how the medication will work to help improve the symptoms of dyspnea and orthopnea. The nurse's reply will be based on the information that nesiritide will a. dilate arterial and venous blood vessels, decreasing ventricular preload and afterload. b. improve the ability of the ventricular myocardium to contract, strengthening contractility. c. enhance the speed of impulse conduction through the heart, increasing the heart rate. d. increase calcium sensitivity in vascular smooth muscle, boosting systemic vascular resistance.

A Rationale: Nesiritide, a recombinant form of BNP, causes both arterial and venous vasodilation, leading to reductions in preload and afterload. Inotropic medications, such as dopamine and dobutamine, may be used in ADHF to improve ventricular contractility. Nesiritide does not increase impulse conduction or calcium sensitivity in the heart. Cognitive Level: Application Text Reference: p. 829 Nursing Process: Implementation NCLEX: Physiological Integrity

19. An outpatient who has developed heart failure after having an acute myocardial infarction has a new prescription for carvedilol (Coreg). After 2 weeks, the patient returns to the clinic. The assessment finding that will be of most concern to the nurse is that the patient a. has BP of 88/42. b. has an apical pulse rate of 56. c. complains of feeling tired. d. has 2+ pedal edema.

A Rationale: The patient's BP indicates that the dose of carvedilol may need to be decreased because the mean arterial pressure is only 57. Bradycardia is a frequent adverse effect of -Adrenergic blockade, but the rate of 56 is not as great a concern as the hypotension. -adrenergic blockade will initially worsen symptoms of heart failure in many patients, and patients should be taught that some increase in symptoms, such as fatigue and edema, is expected during the initiation of therapy with this class of drugs. Cognitive Level: Application Text Reference: p. 832 Nursing Process: Assessment NCLEX: Analysis

1. A patient with a history of chronic heart failure is admitted to the emergency department with severe dyspnea and a dry, hacking cough. The patient has pitting edema in both ankles, blood pressure (BP) of 170/100, an apical pulse rate of 92, and respirations 28. The most important assessment for the nurse to accomplish next is to a. auscultate the lung sounds. b. assess the orientation. c. check the capillary refill. d. palpate the abdomen.

A Rationale: When caring for a patient with severe dyspnea, the nurse should use the ABCs to guide initial care. This patient's severe dyspnea and cough indicate that acute decompensated heart failure (ADHF) is occurring. ADHF usually manifests as pulmonary edema, which should be detected and treated immediately to prevent ongoing hypoxemia and cardiac/respiratory arrest. The other assessments will provide useful data about the patient's volume status and should also be accomplished rapidly, but detection (and treatment) of fluid-filled alveoli is the priority. Cognitive Level: Application Text Reference: pp. 824-825 Nursing Process: Assessment NCLEX: Physiological Integrity

The nurse is preparing to administer a beta blocker to the client diagnosed with coronary artery disease. Which assessment data would cause the nurse to question administering the medication? 2. The client has an apical pulse of 56.

A beta blocker decreases sympathetic stimulation to the heart, thereby decreasing the heart rate. An apical rate less than 60 indicates lower-than-normal heart rate and should make the nurse question administering this medication because it will further decrease the heart rate.

Following an acute myocardial infarction (AMI), a patient ambulates in the hospital hallway. When the nurse is evaluating the patient's response to the activity, which assessment data would indicate that the exercise level should be decreased? c. Heart rate increases from 66 to 92 beats/minute.

A change in heart rate of more than 20 beats over the resting heart rate indicates that the patient should stop and rest.

The nurse is assigned the following client assignment on the clinical unit. For which client does the nurse anticipate cardioversion as a possible medical treatment? a) A client with poor kidney perfusion b) A new myocardial infarction client c) A client with atrial dysrhythmias d) A client with third-degree heart block

A client with atrial dysrhythmias Explanation: The nurse is correct to identify a client with atrial dysrhythmias as a candidate for cardioversion. The goal of cardioversion is to restore the normal pacemaker of the heart, as well as, normal conduction. A client with a myocardial infarction has tissue damage. The client with poor perfusion has circulation problems. The client with heart block has an impairment in the conduction system and may require a pacemaker. pg.716

The client has chronic atrial fibrillation. Which discharge teaching should the nurse discuss with the client? 1. Instruct the client to use a soft-bristle toothbrush.

A client with chronic atrial fibrillation will be taking an anticoagulant to help prevent clot formation. Therefore, the client is at risk for bleeding and should be instructed to use a soft-bristle toothbrush.

The nurse is caring for clients on a surgical floor. Which client should be assessed first? 1. The client who is four (4) day post-operative abdominal surgery and is complaining of left calf pain when ambulating.

A complication of immobility after surgery is developing a DVT. This client with left calf pain should be assessed for a DVT.

C. Antistreptolysin-O titer (ASO) titer The most reliable and best standardized lab for antistreptococcal antibodies is an Antistreptolysin-O (ASO) titer. A throat culture indicates a current streptococcal infection. C-reactive protein (CRP) laboratory test indicates inflammation. An elevated white blood cell (WBC) may indicate a possible infection but does not indicate a causative agent. An erythrocyte sedimentation rate (ESR) indicates inflammation.

A diagnosis of rheumatic fever is being ruled out for a child. Which lab test(s) is/are the most reliable? (Select all that apply.) A. Throat culture B. C-reactive protein (CRP) C. Antistreptolysin-O titer (ASO) titer D. Elevated white blood cell count (WBC) E. Erythrocyte sedimentation rate (ESR)

Which client teaching should the nurse implement for the client diagnosed with coronary artery disease? Select all that apply. 1. Encourage a low-fat, low-cholesterol diet. 2. Instruct client to walk 30 minutes a day. 4. Refer to counselor for stress reduction techniques. 5. Increase fiber in the diet.

A low-fat, low-cholesterol diet will help decrease the buildup of atherosclerosis in the arteries. Walking will help increase collateral circulation. Stress reduction is encouraged for clients with CAD because this helps prevent excess stress on the heart muscle. Increasing fiber in the diet will help remove cholesterol via the gastrointestinal system.

The nurse is teaching a class on coronary artery disease. Which modifiable risk factors should the nurse discuss when teaching about atherosclerosis? 1. Stress.

A modifiable risk factor is a risk factor that can possibly be altered by modifying or changing behavior, such as developing new ways to deal with stress.

The charge nurse is making assignments for clients on a cardiac unit. Which client should the charge nurse assign to a new graduate nurse? 3. The 75-year-old client scheduled for a cardiac catheterization.

A new graduate should be able to complete A pre-procedure checklist and get this client to the catheterization lab.

The nurse is discussing the pathophysiology of atherosclerosis with a client who has a normal high-density lipoprotein (HDL) level. Which information should the nurse discuss with the client concerning HDL? A normal HDL is good because it has a protective action in the body.

A normal HDL level is good because HDL transports cholesterol away from the tissues and cells of the arterial wall to the liver for excretion. This helps decrease the development of atherosclerosis.

The client diagnosed with pericarditis is experiencing cardiac tamponade. Which collaborative intervention should the nurse anticipate for this client? 1. Prepare for a pericardiocentesis.

A pericardiocentesis removes fluid from the pericardial sac and is the emergency treatment for cardiac tamponade.

CK-MB indicate MI not heart failure

A positive D-dimer indicate pulmonary embolism

Which assessment data would warrant immediate intervention by the nurse? 1. The client diagnosed with DVT who complains of pain on inspiration.

A potentially life-threatening complication of DVT is a pulmonary embolus, which causes chest pain. The nurse should determine if the client has "thrown" a pulmonary embolus.

The nurse conducts a complete physical assessment on a patient admitted with infective endocarditis. Which finding is significant? B. Regurgitant murmur at the mitral valve area

A regurgitant murmur of the aortic or mitral valves would indicate valvular disease, which is a complication of endocarditis.

Which assessment data would support the diagnosis of abdominal aortic aneurysm (AAA)? 2. Abdominal bruit.

A systolic bruit over the abdomen is a diagnostic indication of an AAA.

D. Squatting. Low Fowler's would assist with respiratory issues but would not assist with the need for cardiac compensation. Prone does not offer any advantage to the child. Supine does not offer any advantage to the child. The squatting or knee-chest position decreases the amount of blood returning to the heart and allows the child time to compensate.

A young child with tetralogy of Fallot may assume a posturing position as a compensatory mechanism. The position automatically assumed by the child is: A. Low Fowler's. B. Prone. C. Supine. D. Squatting.

The nurse should explain to the parents that their child is receiving Lasix for severe congestive heart failure because it is a/an: A. Diuretic. B. â-blocker. C. Form of digitalis. D. ACE inhibitor.

A. Diuretic. Furosemide (Lasix) is a diuretic used to eliminate excess water and salt to prevent reaccumulation of the fluid. Lasix is a diuretic. Lasix is a diuretic. Lasix is a diuretic.

The nurse should instruct a child to remain completely still during which procedure in which high frequency sound waves are translated into images by a transducer? A. Echocardiography B. Electrocardiography C. Cardiac catheterization D. Electrophysiology

A. Echocardiography Echocardiography uses high-frequency sound waves. The child must lie completely still. With the improvements in technology, diagnosis can sometimes be made without cardiac catheterization. Electrocardiography is a tracing of the electrical path of the depolarization action of myocardial cells. Cardiac catheterization is an invasive procedure in which a catheter is threaded into the heart. Electrophysiology is an invasive procedure in which catheters with electrodes are used to record the impulses of the heart directly from the conduction system.

The nurse is preparing to give digoxin to a 9-month-old infant. He or she checks the dose and draws up 4 ml of the drug. The MOST appropriate nursing action is to: A. Not give the dose; suspect dosage error. B. Mix the dose with juice to disguise its taste. C. Check heart rate; administer the dose by placing it to the back and side of the mouth. D. Check heart rate; administer the dose by letting the infant suck it through a nipple.

A. Not give the dose; suspect dosage error. Digoxin is often prescribed in micrograms. Rarely is more than 1 ml administered to an infant. Because it is a potentially dangerous drug, administration guidelines are very precise. Some institutions require that digoxin dosages be checked with another professional before administration. The nurse has drawn up too much medication. These are correct procedures, but too much medication has been prepared. These are correct procedures, but too much medication has been prepared.

An early sign of congestive heart failure that the nurse should recognize is: A. Tachypnea. B. Bradycardia. C. Inability to sweat. D. Increased urine output.

A. Tachypnea. Tachypnea is one of the early signs that should be identified. Tachycardia at rest, dyspnea, retractions, and activity intolerance are other physical signs and symptoms. Tachycardia, not bradycardia, is one of the symptoms suggestive of congestive heart failure. The child may be diaphoretic. Urine output usually will be decreased due to decreased kidney perfusion.

Which is the acceptable mg/dl level, or below this level, low-density lipoprotein (LDL) cholesterol level for a child from a family with heart disease? _____ Record your answer as a whole number.

ANS: 110 The low-density lipoproteins (LDLs) contain low concentrations of triglycerides, high levels of cholesterol, and moderate levels of protein. LDL is the major carrier of cholesterol to the cells. Cells use cholesterol for synthesis of membranes and steroid production. Elevated circulating LDL is a strong risk factor in cardiovascular disease. For children from families with a history of heart disease, the LDL should be <110.

To auscultate for S3 or S4 gallops in the mitral area, the nurse listens with the a. bell of the stethoscope with the patient in the left lateral position. b. diaphragm of the stethoscope with the patient in a supine position. c. bell of the stethoscope with the patient sitting and leaning forward. d. diaphragm of the stethoscope with the patient lying flat on the left side

ANS: A Gallop rhythms generate low-pitched sounds and are most easily heard with the bell of the stethoscope. Sounds associated with the mitral valve are accentuated by turning the patient to the left side, which brings the heart closer to the chest wall. The diaphragm of the stethoscope is best to use for the higher-pitched sounds such as S1 and S2.

The nurse hears a murmur between the S1 and S2 heart sounds at the patient's left fifth intercostal space and midclavicular line. How will the nurse record this information? a. Systolic murmur heard at mitral area b. Systolic murmur heard at Erb's point c. Diastolic murmur heard at aortic area d. Diastolic murmur heard at the point of maximal impulse

ANS: A The S1 signifies the onset of ventricular systole. S2 signifies the onset of diastole. A murmur occurring between these two sounds is a systolic murmur. The mitral area is the intersection of the left fifth intercostal space and the midclavicular line. The other responses describe murmurs heard at different landmarks on the chest and/or during the diastolic phase of the cardiac cycle.

The standard policy on the cardiac unit states, "Notify the health care provider for mean arterial pressure (MAP) less than 70 mm Hg." The nurse will need to call the health care provider about the a. postoperative patient with a BP of 116/42. b. newly admitted patient with a BP of 150/87. c. patient with left ventricular failure who has a BP of 110/70. d. patient with a myocardial infarction who has a BP of 140/86.

ANS: A The mean arterial pressure (MAP) is calculated using the formula MAP = (systolic BP + 2 diastolic BP)/3. The MAP for the postoperative patient in answer 3 is 67. The MAP in the other three patients is higher than 70 mm Hg.

What is the nurse's first action when planning to teach the parents of an infant with a congenital heart defect (CHD)? a. Assess the parents' anxiety level and readiness to learn. b. Gather literature for the parents. c. Secure a quiet place for teaching. d. Discuss the plan with the nursing team.

ANS: A Any effort to organize the right environment, plan, or literature is of no use if the parents are not ready to learn or have high anxiety. Decreasing their level of anxiety is often needed before new information can be processed. A baseline assessment of prior knowledge should be taken into consideration before developing any teaching plan. Locating a quiet place for meeting with parents is appropriate; however, an assessment should be done before any teaching is done. Discussing a teaching plan with the nursing team is appropriate after an assessment of the parents' knowledge and readiness.

40. What is the nurse's first action when planning to teach the parents of an infant with a congenital heart defect (CHD)? a. Assess the parents' anxiety level and readiness to learn. b. Gather literature for the parents. c. Secure a quiet place for teaching. d. Discuss the plan with the nursing team.

ANS: A Any effort to organize the right environment, plan, or literature is of no use if the parents are not ready to learn or have high anxiety. Decreasing their level of anxiety is often needed before new information can be processed. A baseline assessment of prior knowledge should be taken into consideration before developing any teaching plan. Locating a quiet place for meeting with parents is appropriate; however, an assessment should be done before any teaching is done. Discussing a teaching plan with the nursing team is appropriate after an assessment of the parents' knowledge and readiness. PTS: 1 DIF: Cognitive Level: Application REF: 1339 OBJ: Nursing Process: Planning MSC: Client Needs: Health Promotion and Maintenance

The nurse is assessing a child post-cardiac catheterization. Which complication might the nurse anticipate? a. Cardiac arrhythmia c. Congestive heart failure b. Hypostatic pneumonia d. Rapidly increasing blood pressure

ANS: A Because a catheter is introduced into the heart, a risk exists of catheter-induced arrhythmias occurring during the procedure. These are usually transient. Hypostatic pneumonia, congestive heart failure, and rapidly increasing blood pressure are not risks usually associated with cardiac catheterization.

1. The nurse is assessing a child post-cardiac catheterization. Which complication might the nurse anticipate? a. Cardiac arrhythmia c. Congestive heart failure b. Hypostatic pneumonia d. Rapidly increasing blood pressure

ANS: A Because a catheter is introduced into the heart, a risk exists of catheter-induced arrhythmias occurring during the procedure. These are usually transient. Hypostatic pneumonia, congestive heart failure, and rapidly increasing blood pressure are not risks usually associated with cardiac catheterization. PTS: 1 DIF: Cognitive Level: Application REF: 1320 OBJ: Nursing Process: Assessment MSC: Client Needs: Physiologic Integrity

Which drug is an angiotensin-converting enzyme (ACE) inhibitor? a. Captopril (Capoten) c. Spironolactone (Aldactone) b. Furosemide (Lasix) d. Chlorothiazide (Diuril)

ANS: A Capoten is an ACE inhibitor. Lasix is a loop diuretic. Aldactone blocks the action of aldosterone. Diuril works on the distal tubules.

9. Which drug is an angiotensin-converting enzyme (ACE) inhibitor? a. Captopril (Capoten) c. Spironolactone (Aldactone) b. Furosemide (Lasix) d. Chlorothiazide (Diuril)

ANS: A Capoten is an ACE inhibitor. Lasix is a loop diuretic. Aldactone blocks the action of aldosterone. Diuril works on the distal tubules. PTS: 1 DIF: Cognitive Level: Comprehension REF: 1332 OBJ: Nursing Process: Implementation MSC: Client Needs: Physiologic Integrity

A beneficial effect of administering digoxin (Lanoxin) is that it: a. Decreases edema. c. Increases heart size. b. Decreases cardiac output. d. Increases venous pressure.

ANS: A Digoxin has a rapid onset and is useful in increasing cardiac output, decreasing venous pressure, and as a result decreasing edema. Heart size is decreased by digoxin.

8. A beneficial effect of administering digoxin (Lanoxin) is that it: a. Decreases edema. c. Increases heart size. b. Decreases cardiac output. d. Increases venous pressure.

ANS: A Digoxin has a rapid onset and is useful in increasing cardiac output, decreasing venous pressure, and as a result decreasing edema. Heart size is decreased by digoxin. PTS: 1 DIF: Cognitive Level: Comprehension REF: 1332 OBJ: Nursing Process: Implementation MSC: Client Needs: Physiologic Integrity

Which intervention should be included in the plan of care for an infant with the nursing diagnosis of Excess Fluid Volume related to congestive heart failure? a. Weigh the infant every day on the same scale at the same time. b. Notify the physician when weight gain exceeds more than 20 g/day. c. Put the infant in a car seat to minimize movement. d. Administer digoxin (Lanoxin) as ordered by the physician.

ANS: A Excess fluid volume may not be overtly visible. Weight changes may indicate fluid retention. Weighing the infant on the same scale at the same time each day ensures consistency. An excessive weight gain for an infant is an increase of more than 50 g/day. With fluid volume excess, skin will be edematous. The infant's position should be changed frequently to prevent undesirable pooling of fluid in certain areas. Lanoxin is used in the treatment of congestive heart failure to improve cardiac function. Diuretics will help the body get rid of excess fluid.

35. Which intervention should be included in the plan of care for an infant with the nursing diagnosis of Excess Fluid Volume related to congestive heart failure? a. Weigh the infant every day on the same scale at the same time. b. Notify the physician when weight gain exceeds more than 20 g/day. c. Put the infant in a car seat to minimize movement. d. Administer digoxin (Lanoxin) as ordered by the physician.

ANS: A Excess fluid volume may not be overtly visible. Weight changes may indicate fluid retention. Weighing the infant on the same scale at the same time each day ensures consistency. An excessive weight gain for an infant is an increase of more than 50 g/day. With fluid volume excess, skin will be edematous. The infant's position should be changed frequently to prevent undesirable pooling of fluid in certain areas. Lanoxin is used in the treatment of congestive heart failure to improve cardiac function. Diuretics will help the body get rid of excess fluid. PTS: 1 DIF: Cognitive Level: Application REF: 1334 OBJ: Nursing Process: Implementation MSC: Client Needs: Physiologic Integrity

Which painful, tender, pea-sized nodules may appear on the pads of the fingers or toes in bacterial endocarditis? a. Osler's nodes c. Subcutaneous nodules b. Janeway lesions d. Aschoff's nodules

ANS: A Osler's nodes are red, painful, intradermal nodes found on pads of the phalanges in bacterial endocarditis. Janeway lesions are painless hemorrhagic areas on palms and soles in bacterial endocarditis. Subcutaneous nodules are nontender swellings located over bony prominences, commonly found in rheumatic fever. Aschoff's nodules are small nodules composed of cells and leukocytes found in the interstitial tissues of the heart in rheumatic myocarditis.

22. Which painful, tender, pea-sized nodules may appear on the pads of the fingers or toes in bacterial endocarditis? a. Osler's nodes c. Subcutaneous nodules b. Janeway lesions d. Aschoff's nodules

ANS: A Osler's nodes are red, painful, intradermal nodes found on pads of the phalanges in bacterial endocarditis. Janeway lesions are painless hemorrhagic areas on palms and soles in bacterial endocarditis. Subcutaneous nodules are nontender swellings located over bony prominences, commonly found in rheumatic fever. Aschoff's nodules are small nodules composed of cells and leukocytes found in the interstitial tissues of the heart in rheumatic myocarditis. PTS: 1 DIF: Cognitive Level: Comprehension REF: 1344 OBJ: Nursing Process: Assessment MSC: Client Needs: Physiologic Integrity

A major clinical manifestation of rheumatic fever is: a. Polyarthritis. b. Osler's nodes. c. Janeway spots. d. Splinter hemorrhages of distal third of nails.

ANS: A Polyarthritis is swollen, hot, red, and painful joints. The affected joints will change every 1 to 2 days. Primarily the large joints are affected. Osler's nodes, Janeway spots, and splinter hemorrhages are characteristic of infective endocarditis.

25. A major clinical manifestation of rheumatic fever is: a. Polyarthritis. b. Osler's nodes. c. Janeway spots. d. Splinter hemorrhages of distal third of nails.

ANS: A Polyarthritis is swollen, hot, red, and painful joints. The affected joints will change every 1 to 2 days. Primarily the large joints are affected. Osler's nodes, Janeway spots, and splinter hemorrhages are characteristic of infective endocarditis. PTS: 1 DIF: Cognitive Level: Analysis REF: 1345 OBJ: Nursing Process: Assessment MSC: Client Needs: Physiologic Integrity

A child with pulmonary atresia exhibits cyanosis with feeding. On reviewing this child's laboratory values, the nurse is not surprised to notice which abnormality? a. Polycythemia c. Dehydration b. Infection d. Anemia

ANS: A Polycythemia is a compensatory response to chronic hypoxia. The body attempts to improve tissue oxygenation by producing additional red blood cells and thereby increases the oxygen-carrying capacity of the blood. Infection is not a clinical consequence of cyanosis. Although dehydration can occur in cyanotic heart disease, it is not a compensatory mechanism for chronic hypoxia. It is not a clinical consequence of cyanosis. Anemia may develop as a result of increased blood viscosity. Anemia is not a clinical consequence of cyanosis.

38. A child with pulmonary atresia exhibits cyanosis with feeding. On reviewing this child's laboratory values, the nurse is not surprised to notice which abnormality? a. Polycythemia c. Dehydration b. Infection d. Anemia

ANS: A Polycythemia is a compensatory response to chronic hypoxia. The body attempts to improve tissue oxygenation by producing additional red blood cells and thereby increases the oxygen-carrying capacity of the blood. Infection is not a clinical consequence of cyanosis. Although dehydration can occur in cyanotic heart disease, it is not a compensatory mechanism for chronic hypoxia. It is not a clinical consequence of cyanosis. Anemia may develop as a result of increased blood viscosity. Anemia is not a clinical consequence of cyanosis. PTS: 1 DIF: Cognitive Level: Analysis REF: 1337 OBJ: Nursing Process: Assessment MSC: Client Needs: Physiologic Integrity

Which structural defects constitute tetralogy of Fallot? a. Pulmonic stenosis, ventricular septal defect, overriding aorta, right ventricular hypertrophy b. Aortic stenosis, ventricular septal defect, overriding aorta, right ventricular hypertrophy c. Aortic stenosis, atrial septal defect, overriding aorta, left ventricular hypertrophy d. Pulmonic stenosis, ventricular septal defect, aortic hypertrophy, left ventricular hypertrophy

ANS: A Tetralogy of Fallot has these four characteristics: pulmonary stenosis, ventricular septal defect, overriding aorta, and right ventricular hypertrophy. There is pulmonic stenosis but not aortic stenosis in tetralogy of Fallot. Right ventricular hypertrophy, not left ventricular hypertrophy, is present in tetralogy of Fallot. There is a ventricular septal defect, not an atrial septal defect, and overriding aorta, not aortic hypertrophy, is present.

5. Which structural defects constitute tetralogy of Fallot? a. Pulmonic stenosis, ventricular septal defect, overriding aorta, right ventricular hypertrophy b. Aortic stenosis, ventricular septal defect, overriding aorta, right ventricular hypertrophy c. Aortic stenosis, atrial septal defect, overriding aorta, left ventricular hypertrophy d. Pulmonic stenosis, ventricular septal defect, aortic hypertrophy, left ventricular hypertrophy

ANS: A Tetralogy of Fallot has these four characteristics: pulmonary stenosis, ventricular septal defect, overriding aorta, and right ventricular hypertrophy. There is pulmonic stenosis but not aortic stenosis in tetralogy of Fallot. Right ventricular hypertrophy, not left ventricular hypertrophy, is present in tetralogy of Fallot. There is a ventricular septal defect, not an atrial septal defect, and overriding aorta, not aortic hypertrophy, is present. PTS: 1 DIF: Cognitive Level: Comprehension REF: 1327 OBJ: Nursing Process: Assessment MSC: Client Needs: Physiologic Integrity

Parents of a 3-year-old child with congenital heart disease are afraid to let their child play with other children because of possible overexertion. The nurse's reply should be based on knowing that: a. The child needs opportunities to play with peers. b. The child needs to understand that peers' activities are too strenuous. c. Parents can meet all the child's needs. d. Constant parental supervision is needed to avoid overexertion.

ANS: A The child needs opportunities for social development. Children usually limit their activities if allowed to set their own pace and regulate their activities. The child will limit activities as necessary. Parents must be encouraged to seek appropriate social activities for the child, especially before kindergarten. The child needs to have activities that foster independence.

15. Parents of a 3-year-old child with congenital heart disease are afraid to let their child play with other children because of possible overexertion. The nurse's reply should be based on knowing that: a. The child needs opportunities to play with peers. b. The child needs to understand that peers' activities are too strenuous. c. Parents can meet all the child's needs. d. Constant parental supervision is needed to avoid overexertion.

ANS: A The child needs opportunities for social development. Children usually limit their activities if allowed to set their own pace and regulate their activities. The child will limit activities as necessary. Parents must be encouraged to seek appropriate social activities for the child, especially before kindergarten. The child needs to have activities that foster independence. PTS: 1 DIF: Cognitive Level: Analysis REF: 1339 OBJ: Nursing Process: Implementation MSC: Client Needs: Psychosocial Integrity

The nurse is admitting a child with rheumatic fever. Which therapeutic management should the nurse expect to implement? a. Administering penicillin b. Avoiding salicylates (aspirin) c. Imposing strict bed rest for 4 to 6 weeks d. Administering corticosteroids if chorea develops

ANS: A The goal of medical management is the eradication of the hemolytic streptococci. Penicillin is the drug of choice. Salicylates can be used to control the inflammatory process, especially in the joints, and reduce the fever and discomfort. Bed rest is recommended for the acute febrile stage, but it does not need to be strict. The chorea is transient and will resolve without treatment.

49. The nurse is admitting a child with rheumatic fever. Which therapeutic management should the nurse expect to implement? a. Administering penicillin b. Avoiding salicylates (aspirin) c. Imposing strict bed rest for 4 to 6 weeks d. Administering corticosteroids if chorea develops

ANS: A The goal of medical management is the eradication of the hemolytic streptococci. Penicillin is the drug of choice. Salicylates can be used to control the inflammatory process, especially in the joints, and reduce the fever and discomfort. Bed rest is recommended for the acute febrile stage, but it does not need to be strict. The chorea is transient and will resolve without treatment. PTS: 1 DIF: Cognitive Level: Application REF: 1345 OBJ: Nursing Process: Implementation MSC: Client Needs: Physiologic Integrity

In which situation is there the greatest risk that a newborn infant will have a congenital heart defect (CHD)? a. Trisomy 21 detected on amniocentesis b. Family history of myocardial infarction c. Father has type 1 diabetes mellitus d. Older sibling born with Turner's syndrome

ANS: A The incidence of congenital heart disease is approximately 50% in children with trisomy 21 (Down syndrome). A family history of congenital heart disease, not acquired heart disease, increases the risk of giving birth to a child with CHD. Infants born to mothers who are insulin dependent have an increased risk of CHD. Infants identified as having certain genetic defects, such as Turner's syndrome, have a higher incidence of CHD.

34. In which situation is there the greatest risk that a newborn infant will have a congenital heart defect (CHD)? a. Trisomy 21 detected on amniocentesis b. Family history of myocardial infarction c. Father has type 1 diabetes mellitus d. Older sibling born with Turner's syndrome

ANS: A The incidence of congenital heart disease is approximately 50% in children with trisomy 21 (Down syndrome). A family history of congenital heart disease, not acquired heart disease, increases the risk of giving birth to a child with CHD. Infants born to mothers who are insulin dependent have an increased risk of CHD. Infants identified as having certain genetic defects, such as Turner's syndrome, have a higher incidence of CHD. PTS: 1 DIF: Cognitive Level: Application REF: 1321 OBJ: Nursing Process: Assessment MSC: Client Needs: Health Promotion and Maintenance

The nurse is caring for an infant with congestive heart disease (CHD). The nurse should plan which intervention to decrease cardiac demands? a. Organize nursing activities to allow for uninterrupted sleep. b. Allow the infant to sleep through feedings during the night. c. Wait for the infant to cry to show definite signs of hunger. d. Discourage parents from rocking the infant

ANS: A The infant requires rest and conservation of energy for feeding. Every effort is made to organize nursing activities to allow for uninterrupted periods of sleep. Whenever possible, parents are encouraged to stay with their infant to provide the holding, rocking, and cuddling that help children sleep more soundly. To minimize disturbing the infant, changing bed linens and complete bathing are done only when necessary. Feeding is planned to accommodate the infant's sleep and wake patterns. The child is fed at the first sign of hunger, such as when sucking on fists, rather than waiting until he or she cries for a bottle because the stress of crying exhausts the limited energy supply. Because infants with CHD tire easily and may sleep through feedings, smaller feedings every 3 hours may be helpful.

52. The nurse is caring for an infant with congestive heart disease (CHD). The nurse should plan which intervention to decrease cardiac demands? a. Organize nursing activities to allow for uninterrupted sleep. b. Allow the infant to sleep through feedings during the night. c. Wait for the infant to cry to show definite signs of hunger. d. Discourage parents from rocking the infant

ANS: A The infant requires rest and conservation of energy for feeding. Every effort is made to organize nursing activities to allow for uninterrupted periods of sleep. Whenever possible, parents are encouraged to stay with their infant to provide the holding, rocking, and cuddling that help children sleep more soundly. To minimize disturbing the infant, changing bed linens and complete bathing are done only when necessary. Feeding is planned to accommodate the infant's sleep and wake patterns. The child is fed at the first sign of hunger, such as when sucking on fists, rather than waiting until he or she cries for a bottle because the stress of crying exhausts the limited energy supply. Because infants with CHD tire easily and may sleep through feedings, smaller feedings every 3 hours may be helpful. PTS: 1 DIF: Cognitive Level: Application REF: 1335 OBJ: Nursing Process: Implementation MSC: Client Needs: Physiologic Integrity

The nurse is talking to a parent of an infant with heart failure about feeding the infant. Which statement about feeding the child is correct? a. "You may need to increase the caloric density of your infant's formula." b. "You should feed your baby every 2 hours." c. "You may need to increase the amount of formula your infant eats with each feeding." d. "You should place a nasal oxygen cannula on your infant during and after each feeding."

ANS: A The metabolic rate of infants with heart failure is greater because of poor cardiac function and increased heart and respiratory rates. Their caloric needs are greater than those of the average infants, yet their ability to take in the calories is diminished by their fatigue. Infants with heart failure should be fed every 3 hours; a 2-hour schedule does not allow for enough rest, and a 4-hour schedule is too long. Fluids must be carefully monitored because of the heart failure. Infants do not require supplemental oxygen with feedings.

48. The nurse is talking to a parent of an infant with heart failure about feeding the infant. Which statement about feeding the child is correct? a. "You may need to increase the caloric density of your infant's formula." b. "You should feed your baby every 2 hours." c. "You may need to increase the amount of formula your infant eats with each feeding." d. "You should place a nasal oxygen cannula on your infant during and after each feeding."

ANS: A The metabolic rate of infants with heart failure is greater because of poor cardiac function and increased heart and respiratory rates. Their caloric needs are greater than those of the average infants, yet their ability to take in the calories is diminished by their fatigue. Infants with heart failure should be fed every 3 hours; a 2-hour schedule does not allow for enough rest, and a 4-hour schedule is too long. Fluids must be carefully monitored because of the heart failure. Infants do not require supplemental oxygen with feedings. PTS: 1 DIF: Cognitive Level: Application REF: 1334 OBJ: Nursing Process: Planning MSC: Client Needs: Physiologic Integrity: Physiologic Adaptation

A child has a total cholesterol level of 180 mg/dL. What dietary recommendations should the nurse make to the child and the child's parents (Select all that apply)? a. Replace whole milk with 2% or 1% milk b. Increase servings of red meat c. Increase servings of fish d. Avoid excessive intake of fruit juices e. Limit servings of whole grain

ANS: A, C, D A low-fat diet includes using nonfat or low-fat dairy products, limiting red meat intake, and increasing intake of fish, vegetables, whole grains, and legumes. Children should avoid excessive intake of fruit juices and other sweetened drinks, sugars, and saturated fats.

55. A child has a total cholesterol level of 180 mg/dL. What dietary recommendations should the nurse make to the child and the child's parents (Select all that apply)? a. Replace whole milk with 2% or 1% milk b. Increase servings of red meat c. Increase servings of fish d. Avoid excessive intake of fruit juices e. Limit servings of whole grain

ANS: A, C, D A low-fat diet includes using nonfat or low-fat dairy products, limiting red meat intake, and increasing intake of fish, vegetables, whole grains, and legumes. Children should avoid excessive intake of fruit juices and other sweetened drinks, sugars, and saturated fats. PTS: 1 DIF: Cognitive Level: Application REF: 1346 OBJ: Nursing Process: Implementation MSC: Client Needs: Health Promotion and Maintenance

When auscultating over the patient's abdominal aorta, the nurse hears a humming sound. The nurse documents this finding as a a. thrill. b. bruit. c. murmur. d. normal finding.

ANS: B A bruit is the sound created by turbulent blood flow in an artery. Thrills are palpable vibrations felt when there is turbulent blood flow through the heart or in a blood vessel. A murmur is the sound caused by turbulent blood flow through the heart. Auscultating a bruit in an artery is not normal and indicates pathology.

. A patient is scheduled for a cardiac catheterization with coronary angiography. Before the test, the nurse informs the patient that a. it will be important to lie completely still during the procedure. b. a flushed feeling may be noted when the contrast dye is injected. c. monitored anesthesia care will be provided during the procedure. d. arterial pressure monitoring will be required for 24 hours after the test.

ANS: B A sensation of warmth or flushing is common when the contrast material is injected, which can be anxiety-producing unless it has been discussed with the patient. The patient may receive a sedative drug before the procedure, but monitored anesthesia care is not used. Arterial pressure monitoring is not routinely used after the procedure to monitor blood pressure. The patient is not immobile during cardiac catheterization and may be asked to cough or take deep breaths.

When assessing a newly admitted patient, the nurse notes a murmur along the left sternal border. To document more information about the murmur, which action will the nurse take next? a. Find the point of maximal impulse. b. Determine the timing of the murmur. c. Compare the apical and radial pulse rates. d. Palpate the quality of the peripheral pulses.

ANS: B Murmurs are caused by turbulent blood flow, such as occurs when blood flows through a damaged valve. Relevant information includes the position in which the murmur is heard best (e.g., sitting and leaning forward), the timing of the murmur in relation to the cardiac cycle (e.g., systole, diastole), and where on the thorax the murmur is heard best. The other information is also important in the cardiac assessment but will not provide information that is relevant to the murmur.

A registered nurse (RN) is observing a student nurse who is doing a physical assessment on a patient. The RN will need to intervene immediately if the student nurse a. presses on the skin over the tibia for 10 seconds to check for edema. b. palpates both carotid arteries simultaneously to compare pulse quality. c. documents a murmur heard along the right sternal border as a pulmonic murmur. d. places the patient in the left lateral position to check for the point of maximal impulse.

ANS: B The carotid pulses should never be palpated at the same time to avoid vagal stimulation, dysrhythmias, and decreased cerebral blood flow. The other assessment techniques also need to be corrected. However, they are not dangerous to the patient.

When admitting a patient for a cardiac catheterization and coronary angiogram, which information about the patient is most important for the nurse to communicate to the health care provider? a. The patient's pedal pulses are +1. b. The patient is allergic to shellfish. c. The patient had a heart attack a year ago. d. The patient has not eaten anything today.

ANS: B The contrast dye used for the procedure is iodine based, so patients who have shellfish allergies will require treatment with medications such as corticosteroids and antihistamines before the angiogram. The other information is also communicated to the health care provider but will not require a change in the usual precardiac catheterization orders or medications.

A transesophageal echocardiogram (TEE) is ordered for a patient with possible endocarditis. Which action included in the standard TEE orders will the nurse need to accomplish first? a. Start an IV line. b. Place the patient on NPO status. c. Administer O2 per nasal cannula. d. Give lorazepam (Ativan) 1 mg IV.

ANS: B The patient will need to be NPO for 6 hours preceding the TEE, so the nurse should place the patient on NPO status as soon as the order is received. The other actions also will need to be accomplished but not until just before or during the procedure.

The nurse and unlicensed assistive personnel (UAP) on the telemetry unit are caring for four patients. Which nursing action can be delegated to the UAP? a. Teaching a patient scheduled for exercise electrocardiography about the procedure b. Placing electrodes in the correct position for a patient who is to receive ECG monitoring c. Checking the catheter insertion site for a patient who is recovering from a coronary angiogram d. Monitoring a patient who has just returned to the unit after a transesophageal echocardiogram

ANS: B UAP can be educated in standardized lead placement for ECG monitoring. Assessment of patients who have had procedures where airway maintenance (transesophageal echocardiography) or bleeding (coronary angiogram) is a concern must be done by the registered nurse (RN). Patient teaching requires RN level education and scope of practice.

One of the most frequent causes of hypovolemic shock in children is: a. Myocardial infarction. c. Anaphylaxis. b. Blood loss. d. Congenital heart disease.

ANS: B Blood loss and extracellular fluid loss are two of the most frequent causes of hypovolemic shock in children. Myocardial infarction is rare in a child; if it occurred, the resulting shock would be cardiogenic, not hypovolemic. Anaphylaxis results in distributive shock from extreme allergy or hypersensitivity to a foreign substance. Congenital heart disease tends to contribute to hypervolemia, not hypovolemia.

29. One of the most frequent causes of hypovolemic shock in children is: a. Myocardial infarction. c. Anaphylaxis. b. Blood loss. d. Congenital heart disease.

ANS: B Blood loss and extracellular fluid loss are two of the most frequent causes of hypovolemic shock in children. Myocardial infarction is rare in a child; if it occurred, the resulting shock would be cardiogenic, not hypovolemic. Anaphylaxis results in distributive shock from extreme allergy or hypersensitivity to a foreign substance. Congenital heart disease tends to contribute to hypervolemia, not hypovolemia. PTS: 1 DIF: Cognitive Level: Comprehension REF: 1355 OBJ: Nursing Process: Assessment MSC: Client Needs: Physiologic Integrity

As part of the treatment for congestive heart failure, the child takes the diuretic furosemide. As part of teaching home care, the nurse encourages the family to give the child foods such as bananas, oranges, and leafy vegetables. These foods are recommended because they are high in: a. Chlorides. c. Sodium. b. Potassium. d. Vitamins.

ANS: B Diuretics that work on the proximal and distal renal tubules contribute to increased losses of potassium. The child's diet should be supplemented with potassium.

12. As part of the treatment for congestive heart failure, the child takes the diuretic furosemide. As part of teaching home care, the nurse encourages the family to give the child foods such as bananas, oranges, and leafy vegetables. These foods are recommended because they are high in: a. Chlorides. c. Sodium. b. Potassium. d. Vitamins.

ANS: B Diuretics that work on the proximal and distal renal tubules contribute to increased losses of potassium. The child's diet should be supplemented with potassium. PTS: 1 DIF: Cognitive Level: Comprehension REF: 1333 OBJ: Nursing Process: Planning MSC: Client Needs: Physiologic Integrity

44. An adolescent being seen by the nurse practitioner for a sports physical is identified as having hypertension. On further testing, it is discovered the child has a cardiac abnormality. The initial treatment of secondary hypertension initially involves: a. Weight control and diet. b. Treating the underlying disease. c. Administration of digoxin. d. Administration of -adrenergic receptor blockers.

ANS: B Identification of the underlying disease should be the first step in treating secondary hypertension. Weight control and diet are nonpharmacologic treatments for primary hypertension. Digoxin is indicated in the treatment of congestive heart failure. -Adrenergic receptor blockers are indicated in the treatment of primary hypertension. PTS: 1 DIF: Cognitive Level: Application REF: 1350 OBJ: Nursing Process: Implementation MSC: Client Needs: Physiologic Integrity

An adolescent being seen by the nurse practitioner for a sports physical is identified as having hypertension. On further testing, it is discovered the child has a cardiac abnormality. The initial treatment of secondary hypertension initially involves: a. Weight control and diet. b. Treating the underlying disease. c. Administration of digoxin. d. Administration of b-adrenergic receptor blockers.

ANS: B Identification of the underlying disease should be the first step in treating secondary hypertension. Weight control and diet are nonpharmacologic treatments for primary hypertension. Digoxin is indicated in the treatment of congestive heart failure. b-Adrenergic receptor blockers are indicated in the treatment of primary hypertension.

The nurse is caring for a child with persistent hypoxia secondary to a cardiac defect. The nurse recognizes that a risk of cerebrovascular accidents (strokes) exists. An important objective to decrease this risk is to: a. Minimize seizures. c. Promote cardiac output. b. Prevent dehydration. d. Reduce energy expenditure.

ANS: B In children with persistent hypoxia, polycythemia develops. Dehydration must be prevented in hypoxemic children because it potentiates the risk of strokes. Minimizing seizures, promoting cardiac output, and reducing energy expenditure will not reduce the risk of cerebrovascular accidents.

14. The nurse is caring for a child with persistent hypoxia secondary to a cardiac defect. The nurse recognizes that a risk of cerebrovascular accidents (strokes) exists. An important objective to decrease this risk is to: a. Minimize seizures. c. Promote cardiac output. b. Prevent dehydration. d. Reduce energy expenditure.

ANS: B In children with persistent hypoxia, polycythemia develops. Dehydration must be prevented in hypoxemic children because it potentiates the risk of strokes. Minimizing seizures, promoting cardiac output, and reducing energy expenditure will not reduce the risk of cerebrovascular accidents. PTS: 1 DIF: Cognitive Level: Analysis REF: 1337 OBJ: Nursing Process: Implementation MSC: Client Needs: Physiologic Integrity

Which clinical changes occur as a result of septic shock? a. Hypothermia c. Vasoconstriction b. Increased cardiac output d. Angioneurotic edema

ANS: B Increased cardiac output, which results in warm, flushed skin, is one of the manifestations of septic shock. Fever and chills are characteristic of septic shock. Vasodilation is more common in septic shock. Angioneurotic edema occurs as a manifestation in anaphylactic shock.

31. Which clinical changes occur as a result of septic shock? a. Hypothermia c. Vasoconstriction b. Increased cardiac output d. Angioneurotic edema

ANS: B Increased cardiac output, which results in warm, flushed skin, is one of the manifestations of septic shock. Fever and chills are characteristic of septic shock. Vasodilation is more common in septic shock. Angioneurotic edema occurs as a manifestation in anaphylactic shock. PTS: 1 DIF: Cognitive Level: Analysis REF: 1356 OBJ: Nursing Process: Assessment MSC: Client Needs: Physiologic Integrity

A nurse is teaching nursing students the physiology of congenital heart defects. Which defect results in decreased pulmonary blood flow? a. Atrial septal defect c. Ventricular septal defect b. Tetralogy of Fallot d. Patent ductus arteriosus

ANS: B Tetralogy of Fallot results in decreased blood flow to the lungs. The pulmonic stenosis increases the pressure in the right ventricle, causing the blood to go from right to left across the interventricular septal defect. Atrial and ventricular septal defects and patent ductus arteriosus result in increased pulmonary blood flow.

47. A nurse is teaching nursing students the physiology of congenital heart defects. Which defect results in decreased pulmonary blood flow? a. Atrial septal defect c. Ventricular septal defect b. Tetralogy of Fallot d. Patent ductus arteriosus

ANS: B Tetralogy of Fallot results in decreased blood flow to the lungs. The pulmonic stenosis increases the pressure in the right ventricle, causing the blood to go from right to left across the interventricular septal defect. Atrial and ventricular septal defects and patent ductus arteriosus result in increased pulmonary blood flow. PTS: 1 DIF: Cognitive Level: Comprehension REF: 1327 OBJ: Nursing Process: Assessment MSC: Client Needs: Physiologic Integrity: Physiologic Adaptation

When preparing a school-age child and the family for heart surgery, the nurse should consider: a. Not showing unfamiliar equipment. b. Letting child hear the sounds of an electrocardiograph monitor. c. Avoiding mentioning postoperative discomfort and interventions. d. Explaining that an endotracheal tube will not be needed if the surgery goes well.

ANS: B The child and family should be exposed to the sights and sounds of the intensive care unit. All positive, nonfrightening aspects of the environment are emphasized. The child should be shown unfamiliar equipment, and its use should be demonstrated on a doll. Carefully prepare the child for the postoperative experience, including intravenous lines, incision, and endotracheal tube.

16. When preparing a school-age child and the family for heart surgery, the nurse should consider: a. Not showing unfamiliar equipment. b. Letting child hear the sounds of an electrocardiograph monitor. c. Avoiding mentioning postoperative discomfort and interventions. d. Explaining that an endotracheal tube will not be needed if the surgery goes well.

ANS: B The child and family should be exposed to the sights and sounds of the intensive care unit. All positive, nonfrightening aspects of the environment are emphasized. The child should be shown unfamiliar equipment, and its use should be demonstrated on a doll. Carefully prepare the child for the postoperative experience, including intravenous lines, incision, and endotracheal tube. PTS: 1 DIF: Cognitive Level: Analysis REF: 1341 OBJ: Nursing Process: Implementation MSC: Client Needs: Health Promotion and Maintenance

The nurse is preparing an adolescent for discharge after a cardiac catheterization. Which statement by the adolescent would indicate a need for further teaching? a. "I should avoid tub baths but may shower." b. "I have to stay on strict bed rest for 3 days." c. "I should remove the pressure dressing the day after the procedure." d. "I may attend school but should avoid exercise for several days."

ANS: B The child does not need to be on strict bed rest for 3 days. Showers are recommended; children should avoid a tub bath. The pressure dressing is removed the day after the catheterization and replaced by an adhesive bandage to keep the area clean. Strenuous activity must be avoided for several days, but the child can return to school.

45. The nurse is preparing an adolescent for discharge after a cardiac catheterization. Which statement by the adolescent would indicate a need for further teaching? a. "I should avoid tub baths but may shower." b. "I have to stay on strict bed rest for 3 days." c. "I should remove the pressure dressing the day after the procedure." d. "I may attend school but should avoid exercise for several days."

ANS: B The child does not need to be on strict bed rest for 3 days. Showers are recommended; children should avoid a tub bath. The pressure dressing is removed the day after the catheterization and replaced by an adhesive bandage to keep the area clean. Strenuous activity must be avoided for several days, but the child can return to school. PTS: 1 DIF: Cognitive Level: Analysis REF: 1320 OBJ: Nursing Process: Implementation MSC: Client Needs: Physiologic Integrity

The nurse assessing a premature newborn infant auscultates a continuous machinery-like murmur. This finding is associated with which congenital heart defect? a. Pulmonary stenosis c. Ventricular septal defect b. Patent ductus arteriosus d. Coarctation of the aorta

ANS: B The classic murmur associated with patent ductus arteriosus is a machinery-like one that can be heard throughout both systole and diastole. A systolic ejection murmur that may be accompanied by a palpable thrill is a manifestation of pulmonary stenosis. The characteristic murmur associated with ventricular septal defect is a loud, harsh, holosystolic murmur. A systolic murmur that is accompanied by an ejection click may be heard on auscultation when coarctation of the aorta is present.

36. The nurse assessing a premature newborn infant auscultates a continuous machinery-like murmur. This finding is associated with which congenital heart defect? a. Pulmonary stenosis c. Ventricular septal defect b. Patent ductus arteriosus d. Coarctation of the aorta

ANS: B The classic murmur associated with patent ductus arteriosus is a machinery-like one that can be heard throughout both systole and diastole. A systolic ejection murmur that may be accompanied by a palpable thrill is a manifestation of pulmonary stenosis. The characteristic murmur associated with ventricular septal defect is a loud, harsh, holosystolic murmur. A systolic murmur that is accompanied by an ejection click may be heard on auscultation when coarctation of the aorta is present. PTS: 1 DIF: Cognitive Level: Application REF: 1323 OBJ: Nursing Process: Assessment MSC: Client Needs: Physiologic Integrity

An 8-month-old infant has a hypercyanotic spell while blood is being drawn. The nurse's first action should be to: a. Assess for neurologic defects. b. Place the child in the knee-chest position. c. Begin cardiopulmonary resuscitation. d. Prepare the family for imminent death.

ANS: B The first action is to place the infant in the knee-chest position. Blow-by oxygen may be indicated. Neurologic defects are unlikely. The child should be assessed for airway, breathing, and circulation. Often calming the child and administering oxygen and morphine can alleviate the hypercyanotic spell; cardiopulmonary resuscitation is not necessary, and death is unlikely.

13. An 8-month-old infant has a hypercyanotic spell while blood is being drawn. The nurse's first action should be to: a. Assess for neurologic defects. b. Place the child in the knee-chest position. c. Begin cardiopulmonary resuscitation. d. Prepare the family for imminent death.

ANS: B The first action is to place the infant in the knee-chest position. Blow-by oxygen may be indicated. Neurologic defects are unlikely. The child should be assessed for airway, breathing, and circulation. Often calming the child and administering oxygen and morphine can alleviate the hypercyanotic spell; cardiopulmonary resuscitation is not necessary, and death is unlikely. PTS: 1 DIF: Cognitive Level: Application REF: 1337 OBJ: Nursing Process: Implementation MSC: Client Needs: Physiologic Integrity

23. The primary nursing intervention necessary to prevent bacterial endocarditis is to: a. Institute measures to prevent dental procedures. b. Counsel parents of high risk children about prophylactic antibiotics. c. Observe children for complications such as embolism and heart failure. d. Encourage restricted mobility in susceptible children.

ANS: B The objective of nursing care is to counsel the parents of high risk children about both the need for prophylactic antibiotics for dental procedures and the necessity of maintaining excellent oral health. The child's dentist should be aware of the child's cardiac condition. Dental procedures should be done to maintain a high level of oral health. Prophylactic antibiotics are necessary. Observing for complications and encouraging restricted mobility in susceptible children should be done, but maintaining good oral health and using prophylactic antibiotics are most important.

The primary nursing intervention necessary to prevent bacterial endocarditis is to: a. Institute measures to prevent dental procedures. b. Counsel parents of high risk children about prophylactic antibiotics. c. Observe children for complications such as embolism and heart failure. d. Encourage restricted mobility in susceptible children.

ANS: B The objective of nursing care is to counsel the parents of high risk children about both the need for prophylactic antibiotics for dental procedures and the necessity of maintaining excellent oral health. The child's dentist should be aware of the child's cardiac condition. Dental procedures should be done to maintain a high level of oral health. Prophylactic antibiotics are necessary. Observing for complications and encouraging restricted mobility in susceptible children should be done, but maintaining good oral health and using prophylactic antibiotics are most important.

The leading cause of death after heart transplantation is: a. Infection. c. Cardiomyopathy. b. Rejection. d. Congestive heart failure.

ANS: B The posttransplantation course is complex. The leading cause of death after cardiac transplantation is rejection. Infection is a continued risk secondary to the immunosuppression necessary to prevent rejection. Cardiomyopathy is one of the indications for cardiac transplant. Congestive heart failure is not a leading cause of death.

27. The leading cause of death after heart transplantation is: a. Infection. c. Cardiomyopathy. b. Rejection. d. Congestive heart failure.

ANS: B The posttransplantation course is complex. The leading cause of death after cardiac transplantation is rejection. Infection is a continued risk secondary to the immunosuppression necessary to prevent rejection. Cardiomyopathy is one of the indications for cardiac transplant. Congestive heart failure is not a leading cause of death. PTS: 1 DIF: Cognitive Level: Comprehension REF: 1351 OBJ: Nursing Process: Assessment MSC: Client Needs: Physiologic Integrity

What is the appropriate priority nursing action for the infant with a CHD who has an increased respiratory rate, is sweating, and is not feeding well? a. Recheck the infant's blood pressure. c. Withhold oral feeding. b. Alert the physician. d. Increase the oxygen rate.

ANS: B These are signs of early congestive heart failure, and the physician should be notified. Although rechecking blood pressure may be indicated, it is not the priority action. Withholding the infant's feeding is an incomplete response to the problem. Increasing oxygen may alleviate symptoms; however, medications such as digoxin and furosemide are necessary to improve heart function and fluid retention. Notifying the physician is the priority nursing action.

42. What is the appropriate priority nursing action for the infant with a CHD who has an increased respiratory rate, is sweating, and is not feeding well? a. Recheck the infant's blood pressure. c. Withhold oral feeding. b. Alert the physician. d. Increase the oxygen rate.

ANS: B These are signs of early congestive heart failure, and the physician should be notified. Although rechecking blood pressure may be indicated, it is not the priority action. Withholding the infant's feeding is an incomplete response to the problem. Increasing oxygen may alleviate symptoms; however, medications such as digoxin and furosemide are necessary to improve heart function and fluid retention. Notifying the physician is the priority nursing action. PTS: 1 DIF: Cognitive Level: Analysis REF: 1331 OBJ: Nursing Process: Implementation MSC: Client Needs: Physiologic Integrity

The nurse is evaluating a child who is taking digoxin for her cardiac condition. The nurse is cognizant that a common sign of digoxin toxicity is: a. Seizures. c. Bradypnea. b. Vomiting. d. Tachycardia.

ANS: B Vomiting is a common sign of digoxin toxicity. Seizures are not associated with digoxin toxicity. The child will have a slower heart rate, not respiratory rate.

10. The nurse is evaluating a child who is taking digoxin for her cardiac condition. The nurse is cognizant that a common sign of digoxin toxicity is: a. Seizures. c. Bradypnea. b. Vomiting. d. Tachycardia.

ANS: B Vomiting is a common sign of digoxin toxicity. Seizures are not associated with digoxin toxicity. The child will have a slower heart rate, not respiratory rate. PTS: 1 DIF: Cognitive Level: Comprehension REF: 1335 OBJ: Nursing Process: Implementation MSC: Client Needs: Physiologic Integrity

The nurse has received the laboratory results for a patient who developed chest pain 4 hours ago and may be having a myocardial infarction. The most important laboratory result to review will be a. myoglobin. b. low-density lipoprotein (LDL) cholesterol. c. troponins T and I. d. creatine kinase-MB (CK-MB).

ANS: C Cardiac troponins start to elevate 4 to 6 hours after myocardial injury and are highly specific to myocardium. They are the preferred diagnostic marker for myocardial infarction. Myoglobin rises in response to myocardial injury within 30 to 60 minutes. It is rapidly cleared from the body, thus limiting its use in the diagnosis of myocardial infarction. LDL cholesterol is useful in assessing cardiovascular risk but is not helpful in determining whether a patient is having an acute myocardial infarction. Creatine kinase (CK-MB) is specific to myocardial injury and infarction and increases 4 to 6 hours after the infarction occurs. It is often trended with troponin levels.

Which information obtained by the nurse who is admitting the patient for magnetic resonance imaging (MRI) will be most important to report to the health care provider before the MRI? a. The patient has an allergy to shellfish. b. The patient has a history of atherosclerosis. c. The patient has a permanent ventricular pacemaker. d. The patient took all the prescribed cardiac medications today.

ANS: C MRI is contraindicated for patients with implanted metallic devices such as pacemakers. The other information also will be reported to the health care provider but does not impact on whether or not the patient can have an MRI.

The nurse is reviewing the laboratory results for newly admitted patients on the cardiovascular unit. Which patient laboratory result is most important to communicate as soon as possible to the health care provider? a. Patient whose triglyceride level is high b. Patient who has very low homocysteine level c. Patient with increase in troponin T and troponin I level d. Patient with elevated high-sensitivity C-reactive protein level

ANS: C The elevation in troponin T and I indicates that the patient has had an acute myocardial infarction. Further assessment and interventions are indicated. The other laboratory results are indicative of increased risk for coronary artery disease but are not associated with acute cardiac problems that need immediate intervention.

. Which action will the nurse implement for a patient who arrives for a calcium-scoring CT scan? a. Insert an IV catheter. b. Administer oral sedative medications. c. Teach the patient about the procedure. d. Confirm that the patient has been fasting.

ANS: C The nurse will need to teach the patient that the procedure is rapid and involves little risk. None of the other actions are necessary.

When the nurse is screening patients for possible peripheral arterial disease, indicate where the posterior tibial artery will be palpated. a. 1 b. 2 c. 3 d. 4

ANS: C The posterior tibial site is located behind the medial malleolus of the tibia.

A child is brought to the emergency department experiencing an anaphylactic reaction to a bee sting. While an airway is being established, what medication should the nurse prepare for immediate administration? a. Diphenhydramine (Benadryl) c. Epinephrine b. Dopamine d. Calcium chloride

ANS: C After the first priority of establishing an airway, epinephrine is the drug of choice. Benadryl is not a strong enough antihistamine for this severe a reaction. Dopamine and calcium chloride are not appropriate drugs for this type of reaction.

32. A child is brought to the emergency department experiencing an anaphylactic reaction to a bee sting. While an airway is being established, what medication should the nurse prepare for immediate administration? a. Diphenhydramine (Benadryl) c. Epinephrine b. Dopamine d. Calcium chloride

ANS: C After the first priority of establishing an airway, epinephrine is the drug of choice. Benadryl is not a strong enough antihistamine for this severe a reaction. Dopamine and calcium chloride are not appropriate drugs for this type of reaction. PTS: 1 DIF: Cognitive Level: Comprehension REF: 1358 OBJ: Nursing Process: Assessment MSC: Client Needs: Physiologic Integrity

A preschool child is scheduled for an echocardiogram. Parents ask the nurse whether they can hold the child during the procedure. The nurse should answer with which response? a. "You will be able to hold your child during the procedure." b. "Your child can be active during the procedure, but can't sit in your lap." c. "Your child must lie quietly; sometimes a mild sedative is administered before the procedure." d. "The procedure is invasive so your child will be restrained during the echocardiogram."

ANS: C Although an echocardiogram is noninvasive, painless, and associated with no known side effects, it can be stressful for children. The child must lie quietly in the standard echocardiographic positions; crying, nursing, being held, or sitting up often leads to diagnostic errors or omissions. Therefore, infants and young children may need a mild sedative; older children benefit from psychologic preparation for the test. The distraction of a video or movie is often helpful.

51. A preschool child is scheduled for an echocardiogram. Parents ask the nurse whether they can hold the child during the procedure. The nurse should answer with which response? a. "You will be able to hold your child during the procedure." b. "Your child can be active during the procedure, but can't sit in your lap." c. "Your child must lie quietly; sometimes a mild sedative is administered before the procedure." d. "The procedure is invasive so your child will be restrained during the echocardiogram."

ANS: C Although an echocardiogram is noninvasive, painless, and associated with no known side effects, it can be stressful for children. The child must lie quietly in the standard echocardiographic positions; crying, nursing, being held, or sitting up often leads to diagnostic errors or omissions. Therefore, infants and young children may need a mild sedative; older children benefit from psychologic preparation for the test. The distraction of a video or movie is often helpful. PTS: 1 DIF: Cognitive Level: Application REF: 1352 OBJ: Nursing Process: Implementation MSC: Client Needs: Physiologic Integrity

Which defect results in increased pulmonary blood flow? a. Pulmonic stenosis c. Atrial septal defect b. Tricuspid atresia d. Transposition of the great arteries

ANS: C Atrial septal defect results in increased pulmonary blood flow. Blood flows from the left atrium (higher pressure) into the right atrium (lower pressure) and then to the lungs via the pulmonary artery. Pulmonic stenosis is an obstruction to blood flowing from the ventricles. Tricuspid atresia results in decreased pulmonary blood flow. Transposition of the great arteries results in mixed blood flow.

4. Which defect results in increased pulmonary blood flow? a. Pulmonic stenosis c. Atrial septal defect b. Tricuspid atresia d. Transposition of the great arteries

ANS: C Atrial septal defect results in increased pulmonary blood flow. Blood flows from the left atrium (higher pressure) into the right atrium (lower pressure) and then to the lungs via the pulmonary artery. Pulmonic stenosis is an obstruction to blood flowing from the ventricles. Tricuspid atresia results in decreased pulmonary blood flow. Transposition of the great arteries results in mixed blood flow. PTS: 1 DIF: Cognitive Level: Comprehension REF: 1322 OBJ: Nursing Process: Assessment MSC: Client Needs: Physiologic Integrity

The nurse is caring for a child after heart surgery. What should she or he do if evidence is found of cardiac tamponade? a. Increase analgesia. b. Apply warming blankets. c. Immediately report this to the physician. d. Encourage the child to cough, turn, and breathe deeply.

ANS: C If evidence is noted of cardiac tamponade (blood or fluid in the pericardial space constricting the heart), the physician is notified immediately of this life-threatening complication. Increasing analgesia may be done before the physician drains the fluid, but the physician must be notified. Warming blankets are not indicated at this time. Encouraging the child to cough, turn, and breathe deeply should be deferred until after the evaluation by the physician.

19. The nurse is caring for a child after heart surgery. What should she or he do if evidence is found of cardiac tamponade? a. Increase analgesia. b. Apply warming blankets. c. Immediately report this to the physician. d. Encourage the child to cough, turn, and breathe deeply.

ANS: C If evidence is noted of cardiac tamponade (blood or fluid in the pericardial space constricting the heart), the physician is notified immediately of this life-threatening complication. Increasing analgesia may be done before the physician drains the fluid, but the physician must be notified. Warming blankets are not indicated at this time. Encouraging the child to cough, turn, and breathe deeply should be deferred until after the evaluation by the physician. PTS: 1 DIF: Cognitive Level: Analysis REF: 1342 OBJ: Nursing Process: Implementation MSC: Client Needs: Physiologic Integrity

An important nursing consideration when suctioning a young child who has had heart surgery is to: a. Perform suctioning at least every hour. b. Suction for no longer than 30 seconds at a time. c. Administer supplemental oxygen before and after suctioning. d. Expect symptoms of respiratory distress when suctioning.

ANS: C If suctioning is indicated, supplemental oxygen is administered with a manual resuscitation bag before and after the procedure to prevent hypoxia. Suctioning should be done only as indicated, not on a routine basis. The child should be suctioned for no more than 5 seconds at one time. Symptoms of respiratory distress are to be avoided by using the appropriate technique.

18. An important nursing consideration when suctioning a young child who has had heart surgery is to: a. Perform suctioning at least every hour. b. Suction for no longer than 30 seconds at a time. c. Administer supplemental oxygen before and after suctioning. d. Expect symptoms of respiratory distress when suctioning.

ANS: C If suctioning is indicated, supplemental oxygen is administered with a manual resuscitation bag before and after the procedure to prevent hypoxia. Suctioning should be done only as indicated, not on a routine basis. The child should be suctioned for no more than 5 seconds at one time. Symptoms of respiratory distress are to be avoided by using the appropriate technique. PTS: 1 DIF: Cognitive Level: Application REF: 1342 OBJ: Nursing Process: Implementation MSC: Client Needs: Physiologic Integrity

An important nursing consideration when chest tubes will be removed from a child is to: a. Explain that it is not painful. b. Explain that only a Band-Aid will be needed. c. Administer analgesics before the procedure. d. Expect bright red drainage for several hours after removal.

ANS: C It is appropriate to prepare the child for the removal of chest tubes with analgesics. Short-acting medications can be used that are administered through an existing intravenous line. It is not a pain-free procedure. A sharp, momentary pain is felt, and this should not be misrepresented to the child. A petroleum gauze/airtight dressing is needed. Little or no drainage should be found on removal.

20. An important nursing consideration when chest tubes will be removed from a child is to: a. Explain that it is not painful. b. Explain that only a Band-Aid will be needed. c. Administer analgesics before the procedure. d. Expect bright red drainage for several hours after removal.

ANS: C It is appropriate to prepare the child for the removal of chest tubes with analgesics. Short-acting medications can be used that are administered through an existing intravenous line. It is not a pain-free procedure. A sharp, momentary pain is felt, and this should not be misrepresented to the child. A petroleum gauze/airtight dressing is needed. Little or no drainage should be found on removal. PTS: 1 DIF: Cognitive Level: Analysis REF: 1342 OBJ: Nursing Process: Planning MSC: Client Needs: Physiologic Integrity

50. Which action by the school nurse is important in the prevention of rheumatic fever? a. Encourage routine cholesterol screenings. b. Conduct routine blood pressure screenings. c. Refer children with sore throats for throat cultures. d. Recommend salicylates instead of acetaminophen for minor discomforts.

ANS: C Nurses have a role in prevention—primarily in screening school-age children for sore throats caused by group A -hemolytic streptococci. They can achieve this by actively participating in throat culture screening or by referring children with possible streptococcal sore throats for testing. Cholesterol and blood pressure screenings do not facilitate the recognition and treatment of group A -hemolytic streptococci. Salicylates should be avoided routinely because of the risk of Reye's syndrome after viral illnesses. PTS: 1 DIF: Cognitive Level: Application REF: 1346 OBJ: Nursing Process: Planning MSC: Client Needs: Physiologic Integrity: Reduction of Risk Potential

Which action by the school nurse is important in the prevention of rheumatic fever? a. Encourage routine cholesterol screenings. b. Conduct routine blood pressure screenings. c. Refer children with sore throats for throat cultures. d. Recommend salicylates instead of acetaminophen for minor discomforts.

ANS: C Nurses have a role in prevention—primarily in screening school-age children for sore throats caused by group A b-hemolytic streptococci. They can achieve this by actively participating in throat culture screening or by referring children with possible streptococcal sore throats for testing. Cholesterol and blood pressure screenings do not facilitate the recognition and treatment of group A b-hemolytic streptococci. Salicylates should be avoided routinely because of the risk of Reye's syndrome after viral illnesses.

A clinical manifestation of the systemic venous congestion that can occur with congestive heart failure is: a. Tachypnea. c. Peripheral edema. b. Tachycardia. d. Pale, cool extremities.

ANS: C Peripheral edema, especially periorbital edema, is a clinical manifestation of systemic venous congestion. Tachypnea is a manifestation of pulmonary congestion. Tachycardia and pale, cool extremities are clinical manifestations of impaired myocardial function.

7. A clinical manifestation of the systemic venous congestion that can occur with congestive heart failure is: a. Tachypnea. c. Peripheral edema. b. Tachycardia. d. Pale, cool extremities.

ANS: C Peripheral edema, especially periorbital edema, is a clinical manifestation of systemic venous congestion. Tachypnea is a manifestation of pulmonary congestion. Tachycardia and pale, cool extremities are clinical manifestations of impaired myocardial function. PTS: 1 DIF: Cognitive Level: Analysis REF: 1332 OBJ: Nursing Process: Assessment MSC: Client Needs: Physiologic Integrity

A nurse is teaching an adolescent about primary hypertension. The nurse knows that which of the following is correct? a. Primary hypertension should be treated with diuretics as soon as it is detected. b. Congenital heart defects are the most common cause of primary hypertension. c. Primary hypertension may be treated with weight reduction. d. Primary hypertension is not affected by exercise.

ANS: C Primary hypertension in children may be treated with weight reduction and exercise programs. If ineffective, pharmacologic intervention may be needed. Primary hypertension is considered an inherited disorder.

43. A nurse is teaching an adolescent about primary hypertension. The nurse knows that which of the following is correct? a. Primary hypertension should be treated with diuretics as soon as it is detected. b. Congenital heart defects are the most common cause of primary hypertension. c. Primary hypertension may be treated with weight reduction. d. Primary hypertension is not affected by exercise.

ANS: C Primary hypertension in children may be treated with weight reduction and exercise programs. If ineffective, pharmacologic intervention may be needed. Primary hypertension is considered an inherited disorder. PTS: 1 DIF: Cognitive Level: Comprehension REF: 1350 OBJ: Nursing Process: Evaluation MSC: Client Needs: Health Promotion and Maintenance

What is best described as the inability of the heart to pump an adequate amount of blood to the systemic circulation at normal filling pressures? a. Pulmonary congestion c. Congestive heart failure b. Congenital heart defect d. Systemic venous congestion

ANS: C The definition of congestive heart failure is the inability of the heart to pump an adequate amount of blood to the systemic circulation at normal filling pressures to meet the metabolic demands of the body. Pulmonary congestion is an excessive accumulation of fluid in the lungs. Congenital heart defect is a malformation of the heart present at birth. Systemic venous congestion is an excessive accumulation of fluid in the systemic vasculature.

6. What is best described as the inability of the heart to pump an adequate amount of blood to the systemic circulation at normal filling pressures? a. Pulmonary congestion c. Congestive heart failure b. Congenital heart defect d. Systemic venous congestion

ANS: C The definition of congestive heart failure is the inability of the heart to pump an adequate amount of blood to the systemic circulation at normal filling pressures to meet the metabolic demands of the body. Pulmonary congestion is an excessive accumulation of fluid in the lungs. Congenital heart defect is a malformation of the heart present at birth. Systemic venous congestion is an excessive accumulation of fluid in the systemic vasculature. PTS: 1 DIF: Cognitive Level: Comprehension REF: 1331 OBJ: Nursing Process: Assessment MSC: Client Needs: Physiologic Integrity

When assessing a child for possible congenital heart defects (CHDs), where should the nurse measure blood pressure? a. The right arm c. All four extremities b. The left arm d. Both arms while the child is crying

ANS: C When a CHD is suspected, the blood pressure should be measured in all four extremities while the child is quiet. Discrepancies between upper and lower extremities may indicate cardiac disease. Blood pressure measurements for upper and lower extremities are compared during an assessment for CHDs. Blood pressure measurements when the child is crying are likely to be elevated; thus the readings will be inaccurate.

39. When assessing a child for possible congenital heart defects (CHDs), where should the nurse measure blood pressure? a. The right arm c. All four extremities b. The left arm d. Both arms while the child is crying

ANS: C When a CHD is suspected, the blood pressure should be measured in all four extremities while the child is quiet. Discrepancies between upper and lower extremities may indicate cardiac disease. Blood pressure measurements for upper and lower extremities are compared during an assessment for CHDs. Blood pressure measurements when the child is crying are likely to be elevated; thus the readings will be inaccurate. PTS: 1 DIF: Cognitive Level: Application REF: 1334 OBJ: Nursing Process: Assessment MSC: Client Needs: Health Promotion and Maintenance

A nurse is conducting discharge teaching to parents about the care of their infant after cardiac surgery. The nurse instructs the parents to notify the physician if what conditions occur (Select all that apply)? a. Respiratory rate of 36 at rest b. Appetite slowly increasing c. Temperature above 37.7° C (100° F) d. New, frequent coughing e. Turning blue or bluer than normal

ANS: C, D, E The parents should be instructed to notify the physician after their infant's cardiac surgery for a temperature above 37.7° C (100° F); new, frequent coughing; and any episodes of the infant turning blue or bluer than normal. A respiratory rate of 36 at rest for an infant is within normal expectations, and it is expected that the appetite will increase slowly.

56. A nurse is conducting discharge teaching to parents about the care of their infant after cardiac surgery. The nurse instructs the parents to notify the physician if what conditions occur (Select all that apply)? a. Respiratory rate of 36 at rest b. Appetite slowly increasing c. Temperature above 37.7° C (100° F) d. New, frequent coughing e. Turning blue or bluer than normal

ANS: C, D, E The parents should be instructed to notify the physician after their infant's cardiac surgery for a temperature above 37.7° C (100° F); new, frequent coughing; and any episodes of the infant turning blue or bluer than normal. A respiratory rate of 36 at rest for an infant is within normal expectations, and it is expected that the appetite will increase slowly. PTS: 1 DIF: Cognitive Level: Application REF: 1342 OBJ: Nursing Process: Implementation MSC: Client Needs: Health Promotion and Maintenance

57. The nurse is conducting discharge teaching about signs and symptoms of heart failure to parents of an infant with a repaired tetralogy of Fallot. Which signs and symptoms should the nurse include (Select all that apply)? a. Warm flushed extremities b. Weight loss c. Decreased urinary output d. Sweating (inappropriate) e. Fatigue

ANS: C, D, E The signs and symptoms of heart failure include decreased urinary output, sweating, and fatigue. Other signs include pale, cool extremities, not warm and flushed, and weight gain, not weight loss.

The nurse is conducting discharge teaching about signs and symptoms of heart failure to parents of an infant with a repaired tetralogy of Fallot. Which signs and symptoms should the nurse include (Select all that apply)? a. Warm flushed extremities b. Weight loss c. Decreased urinary output d. Sweating (inappropriate) e. Fatigue

ANS: C, D, E The signs and symptoms of heart failure include decreased urinary output, sweating, and fatigue. Other signs include pale, cool extremities, not warm and flushed, and weight gain, not weight loss.

Which clinical manifestations would the nurse expect to see as shock progresses in a child and becomes decompensated shock (Select all that apply)? a. Thirst and diminished urinary output b. Irritability and apprehension c. Cool extremities and decreased skin turgor d. Confusion and somnolence e. Normal blood pressure and narrowing pulse pressure f. Tachypnea and poor capillary refill time

ANS: C, D, F Cool extremities, decreased skin turgor, confusion, somnolence, tachypnea, and poor capillary refill time are beginning signs of decompensated shock.

54. Which clinical manifestations would the nurse expect to see as shock progresses in a child and becomes decompensated shock (Select all that apply)? a. Thirst and diminished urinary output b. Irritability and apprehension c. Cool extremities and decreased skin turgor d. Confusion and somnolence e. Normal blood pressure and narrowing pulse pressure f. Tachypnea and poor capillary refill time

ANS: C, D, F Cool extremities, decreased skin turgor, confusion, somnolence, tachypnea, and poor capillary refill time are beginning signs of decompensated shock. PTS: 1 DIF: Cognitive Level: Analysis REF: 1356 OBJ: Nursing Process: Assessment MSC: Client Needs: Physiologic Integrity

Nursing interventions for the child after a cardiac catheterization include which of the following (Select all that apply)? a. Allow ambulation as tolerated. b. Monitor vital signs every 2 hours. c. Assess the affected extremity for temperature and color. d. Check pulses above the catheterization site for equality and symmetry. e. Remove pressure dressing after 4 hours. f. Maintain a patent peripheral intravenous catheter until discharge.

ANS: C, F The extremity that was used for access for the cardiac catheterization must be checked for temperature and color. Coolness and blanching may indicate arterial occlusion. The child should have a patent peripheral intravenous line to ensure adequate hydration. Allowing ambulation, monitoring vital signs every 2 hours, checking pulses, and removing the pressure dressing after 4 hours are interventions that do not apply to a child after a cardiac catheterization

53. Nursing interventions for the child after a cardiac catheterization include which of the following (Select all that apply)? a. Allow ambulation as tolerated. b. Monitor vital signs every 2 hours. c. Assess the affected extremity for temperature and color. d. Check pulses above the catheterization site for equality and symmetry. e. Remove pressure dressing after 4 hours. f. Maintain a patent peripheral intravenous catheter until discharge.

ANS: C, F The extremity that was used for access for the cardiac catheterization must be checked for temperature and color. Coolness and blanching may indicate arterial occlusion. The child should have a patent peripheral intravenous line to ensure adequate hydration. Allowing ambulation, monitoring vital signs every 2 hours, checking pulses, and removing the pressure dressing after 4 hours are interventions that do not apply to a child after a cardiac catheterization. PTS: 1 DIF: Cognitive Level: Application REF: 1320 OBJ: Nursing Process: Implementation MSC: Client Needs: Physiologic Integrity

When the nurse is monitoring a patient who is undergoing exercise (stress) testing on a treadmill, which assessment finding requires the most rapid action by the nurse? a. Patient complaint of feeling tired b. Pulse change from 87 to 101 beats/minute c. Blood pressure (BP) increase from 134/68 to 150/80 mm Hg d. Newly inverted T waves on the electrocardiogram

ANS: D ECG changes associated with coronary ischemia (such as T-wave inversions and ST segment depression) indicate that the myocardium is not getting adequate oxygen delivery and that the exercise test should be terminated immediately. Increases in BP and heart rate (HR) are normal responses to aerobic exercise. Feeling tired is also normal as the intensity of exercise increases during the stress testing.

To determine the effects of therapy for a patient who is being treated for heart failure, which laboratory result will the nurse plan to review? a. Troponin b. Homocysteine (Hcy) c. Low-density lipoprotein (LDL) d. B-type natriuretic peptide (BNP)

ANS: D Increased levels of BNP are a marker for heart failure. The other laboratory results would be used to assess for myocardial infarction (troponin) or risk for coronary artery disease (Hcy and LDL).

After noting a pulse deficit when assessing a 74-year-old patient who has just arrived in the emergency department, the nurse will anticipate that the patient may require a. emergent cardioversion. b. a cardiac catheterization. c. hourly blood pressure (BP) checks. d. electrocardiographic (ECG) monitoring.

ANS: D Pulse deficit is a difference between simultaneously obtained apical and radial pulses. It indicates that there may be a cardiac dysrhythmia that would best be detected with ECG monitoring. Frequent BP monitoring, cardiac catheterization, and emergent cardioversion are used for diagnosis and/or treatment of cardiovascular disorders but would not be as helpful in determining the immediate reason for the pulse deficit

During a physical examination of a 74-year-old patient, the nurse palpates the point of maximal impulse (PMI) in the sixth intercostal space lateral to the left midclavicular line. The most appropriate action for the nurse to take next will be to a. ask the patient about risk factors for atherosclerosis. b. document that the PMI is in the normal anatomic location. c. auscultate both the carotid arteries for the presence of a bruit. d. assess the patient for symptoms of left ventricular hypertrophy.

ANS: D The PMI should be felt at the intersection of the fifth intercostal space and the left midclavicular line. A PMI located outside these landmarks indicates possible cardiac enlargement, such as with left ventricular hypertrophy. Cardiac enlargement is not necessarily associated with atherosclerosis or carotid artery disease.

The nurse teaches the patient being evaluated for rhythm disturbances with a Holter monitor to a. connect the recorder to a computer once daily. b. exercise more than usual while the monitor is in place. c. remove the electrodes when taking a shower or tub bath. d. keep a diary of daily activities while the monitor is worn.

ANS: D The patient is instructed to keep a diary describing daily activities while Holter monitoring is being accomplished to help correlate any rhythm disturbances with patient activities. Patients are taught that they should not take a shower or bath during Holter monitoring and that they should continue with their usual daily activities. The recorder stores the information about the patient's rhythm until the end of the testing, when it is removed and the data are analyzed.

When reviewing the 12-lead electrocardiograph (ECG) for a healthy 79-year-old patient who is having an annual physical examination, what will be of most concern to the nurse? a. The PR interval is 0.21 seconds. b. The QRS duration is 0.13 seconds. c. There is a right bundle-branch block. d. The heart rate (HR) is 42 beats/minute.

ANS: D The resting HR does not change with aging, so the decrease in HR requires further investigation. Bundle-branch block and slight increases in PR interval or QRS duration are common in older individuals because of increases in conduction time through the AV node, bundle of His, and bundle branches

While doing the admission assessment for a thin 76-year-old patient, the nurse observes pulsation of the abdominal aorta in the epigastric area. Which action should the nurse take? a. Teach the patient about aneurysms. b. Notify the hospital rapid response team. c. Instruct the patient to remain on bed rest. d. Document the finding in the patient chart.

ANS: D Visible pulsation of the abdominal aorta is commonly observed in the epigastric area for thin individuals. The nurse should simply document the finding in the admission assessment. Unless there are other abnormal findings (such as a bruit, pain, or hyper/hypotension) associated with the pulsation, the other actions are not necessary.

While assessing a patient who was admitted with heart failure, the nurse notes that the patient has jugular venous distention (JVD) when lying flat in bed. Which action should the nurse take next? a. Document this finding in the patient's record. b. Obtain vital signs, including oxygen saturation. c. Have the patient perform the Valsalva maneuver. d. Observe for JVD with the patient upright at 45 degrees.

ANS: D When the patient is lying flat, the jugular veins are at the level of the right atrium, so JVD is a common (but not a clinically significant) finding. Obtaining vital signs and oxygen saturation is not warranted at this point. JVD is an expected finding when a patient performs the Valsalva maneuver because right atrial pressure increases. JVD that persists when the patient is sitting at a 30- to 45-degree angle or greater is significant. The nurse will document the JVD in the medical record if it persists when the head is elevated.

What type of shock is characterized by a hypersensitivity reaction causing massive vasodilation and capillary leaks, which may occur with drug or latex allergy? a. Neurogenic shock c. Hypovolemic shock b. Cardiogenic shock d. Anaphylactic shock

ANS: D Anaphylactic shock results from extreme allergy or hypersensitivity to a foreign substance. Neurogenic shock results from loss of neuronal control, such as the interruption of neuronal transmission that occurs from a spinal cord injury. Cardiogenic shock is decreased cardiac output. Hypovolemic shock is a reduction in the size of the vascular compartment, decreasing blood pressure, and low central venous pressure.

30. What type of shock is characterized by a hypersensitivity reaction causing massive vasodilation and capillary leaks, which may occur with drug or latex allergy? a. Neurogenic shock c. Hypovolemic shock b. Cardiogenic shock d. Anaphylactic shock

ANS: D Anaphylactic shock results from extreme allergy or hypersensitivity to a foreign substance. Neurogenic shock results from loss of neuronal control, such as the interruption of neuronal transmission that occurs from a spinal cord injury. Cardiogenic shock is decreased cardiac output. Hypovolemic shock is a reduction in the size of the vascular compartment, decreasing blood pressure, and low central venous pressure. PTS: 1 DIF: Cognitive Level: Comprehension REF: 1356 OBJ: Nursing Process: Assessment MSC: Client Needs: Physiologic Integrity

A common, serious complication of rheumatic fever is: a. Seizures. c. Pulmonary hypertension. b. Cardiac arrhythmias. d. Cardiac valve damage.

ANS: D Cardiac valve damage is the most significant complication of rheumatic fever. Seizures, cardiac arrhythmias, and pulmonary hypertension are not common complications of rheumatic fever.

24. A common, serious complication of rheumatic fever is: a. Seizures. c. Pulmonary hypertension. b. Cardiac arrhythmias. d. Cardiac valve damage.

ANS: D Cardiac valve damage is the most significant complication of rheumatic fever. Seizures, cardiac arrhythmias, and pulmonary hypertension are not common complications of rheumatic fever. PTS: 1 DIF: Cognitive Level: Comprehension REF: 1345 OBJ: Nursing Process: Assessment MSC: Client Needs: Physiologic Integrity

The parents of a young child with congestive heart failure tell the nurse that they are "nervous" about giving digoxin. The nurse's response should be based on knowing that: a. It is a safe, frequently used drug. b. It is difficult to either overmedicate or undermedicate with digoxin. c. Parents lack the expertise necessary to administer digoxin. d. Parents must learn specific, important guidelines for administration of digoxin.

ANS: D Digoxin has a narrow therapeutic range. The margin of safety between therapeutic, toxic, and lethal doses is very small. Specific guidelines are available for parents to learn how to administer the drug safely and monitor for side effects. Digoxin is a frequently used drug, but it has a narrow therapeutic range. Very small amounts of the liquid are given to infants, which makes it easy to overmedicate or undermedicate. Parents may lack the necessary expertise to administer the drug at first, but with discharge preparation they should be prepared to administer the drug safely.

11. The parents of a young child with congestive heart failure tell the nurse that they are "nervous" about giving digoxin. The nurse's response should be based on knowing that: a. It is a safe, frequently used drug. b. It is difficult to either overmedicate or undermedicate with digoxin. c. Parents lack the expertise necessary to administer digoxin. d. Parents must learn specific, important guidelines for administration of digoxin.

ANS: D Digoxin has a narrow therapeutic range. The margin of safety between therapeutic, toxic, and lethal doses is very small. Specific guidelines are available for parents to learn how to administer the drug safely and monitor for side effects. Digoxin is a frequently used drug, but it has a narrow therapeutic range. Very small amounts of the liquid are given to infants, which makes it easy to overmedicate or undermedicate. Parents may lack the necessary expertise to administer the drug at first, but with discharge preparation they should be prepared to administer the drug safely. PTS: 1 DIF: Cognitive Level: Analysis REF: 1351 OBJ: Nursing Process: Implementation MSC: Client Needs: Physiologic Integrity

When discussing hyperlipidemia with a group of adolescents, the nurse should explain that high levels of what substance are thought to protect against cardiovascular disease? a. Cholesterol c. Low-density lipoproteins (LDLs) b. Triglycerides d. High-density lipoproteins (HDLs).

ANS: D HDLs contain very low concentrations of triglycerides, relatively little cholesterol, and high levels of proteins. It is thought that HDLs protect against cardiovascular disease. Cholesterol, triglycerides, and LDLs do not protect against cardiovascular disease.

26. When discussing hyperlipidemia with a group of adolescents, the nurse should explain that high levels of what substance are thought to protect against cardiovascular disease? a. Cholesterol c. Low-density lipoproteins (LDLs) b. Triglycerides d. High-density lipoproteins (HDLs).

ANS: D HDLs contain very low concentrations of triglycerides, relatively little cholesterol, and high levels of proteins. It is thought that HDLs protect against cardiovascular disease. Cholesterol, triglycerides, and LDLs do not protect against cardiovascular disease. PTS: 1 DIF: Cognitive Level: Application REF: 1346 OBJ: Nursing Process: Implementation MSC: Client Needs: Physiologic Integrity

When caring for the child with Kawasaki disease, the nurse should understand that: a. The child's fever is usually responsive to antibiotics within 48 hours. b. The principal area of involvement is the joints. c. Aspirin is contraindicated. d. Therapeutic management includes administration of gamma globulin and aspirin.

ANS: D High-dose intravenous gamma globulin and aspirin therapy are indicated to reduce the incidence of coronary artery abnormalities when given within the first 10 days of the illness. The fever of Kawasaki disease is unresponsive to antibiotics and antipyretics. Involvement of mucous membranes and conjunctiva, changes in the extremities, and cardiac involvement are seen.

28. When caring for the child with Kawasaki disease, the nurse should understand that: a. The child's fever is usually responsive to antibiotics within 48 hours. b. The principal area of involvement is the joints. c. Aspirin is contraindicated. d. Therapeutic management includes administration of gamma globulin and aspirin.

ANS: D High-dose intravenous gamma globulin and aspirin therapy are indicated to reduce the incidence of coronary artery abnormalities when given within the first 10 days of the illness. The fever of Kawasaki disease is unresponsive to antibiotics and antipyretics. Involvement of mucous membranes and conjunctiva, changes in the extremities, and cardiac involvement are seen. PTS: 1 DIF: Cognitive Level: Comprehension REF: 1354 OBJ: Nursing Process: Implementation MSC: Client Needs: Physiologic Integrity

The nurse is caring for a school-age girl who has had a cardiac catheterization. The child tells the nurse that her bandage is "too wet." The nurse finds the bandage and bed soaked with blood. The most appropriate initial nursing action is to: a. Notify the physician. b. Apply a new bandage with more pressure. c. Place the child in the Trendelenburg position. d. Apply direct pressure above the catheterization site.

ANS: D If bleeding occurs, direct continuous pressure is applied 2.5 cm (1 inch) above the percutaneous skin site to localize pressure over the vessel puncture. Notifying the physician and applying a new bandage with more pressure can be done after pressure is applied. The nurse can have someone else notify the physician while the pressure is being maintained. The Trendelenburg position would not be helpful; it would increase the drainage from the lower extremities.

3. The nurse is caring for a school-age girl who has had a cardiac catheterization. The child tells the nurse that her bandage is "too wet." The nurse finds the bandage and bed soaked with blood. The most appropriate initial nursing action is to: a. Notify the physician. b. Apply a new bandage with more pressure. c. Place the child in the Trendelenburg position. d. Apply direct pressure above the catheterization site.

ANS: D If bleeding occurs, direct continuous pressure is applied 2.5 cm (1 inch) above the percutaneous skin site to localize pressure over the vessel puncture. Notifying the physician and applying a new bandage with more pressure can be done after pressure is applied. The nurse can have someone else notify the physician while the pressure is being maintained. The Trendelenburg position would not be helpful; it would increase the drainage from the lower extremities. PTS: 1 DIF: Cognitive Level: Analysis REF: 1320 OBJ: Nursing Process: Implementation MSC: Client Needs: Physiologic Integrity

17. Seventy-two hours after cardiac surgery, a young child has a temperature of 37.7 C (101 F). The nurse should: a. Keep the child warm with blankets. b. Apply a hypothermia blanket. c. Record the temperature on nurses' notes. d. Report findings to physician.

ANS: D In the first 24 to 48 hours after surgery, the body temperature may increase to 37.7 C (100 F) as part of the inflammatory response to tissue trauma. If the temperature is higher or an elevated temperature continues after this period, it is most likely a sign of an infection and immediate investigation is indicated. Blankets should be removed from the child to keep the temperature from increasing. A hypothermia blanket is not indicated for this level of temperature. The temperature should be recorded, but the physician must be notified for evaluation. PTS: 1 DIF: Cognitive Level: Analysis REF: 1341 OBJ: Nursing Process: Assessment MSC: Client Needs: Physiologic Integrity

Seventy-two hours after cardiac surgery, a young child has a temperature of 37.7° C (101° F). The nurse should: a. Keep the child warm with blankets. b. Apply a hypothermia blanket. c. Record the temperature on nurses' notes. d. Report findings to physician.

ANS: D In the first 24 to 48 hours after surgery, the body temperature may increase to 37.7° C (100° F) as part of the inflammatory response to tissue trauma. If the temperature is higher or an elevated temperature continues after this period, it is most likely a sign of an infection and immediate investigation is indicated. Blankets should be removed from the child to keep the temperature from increasing. A hypothermia blanket is not indicated for this level of temperature. The temperature should be recorded, but the physician must be notified for evaluation.

José is a 4-year-old child scheduled for a cardiac catheterization. Preoperative teaching should be: a. Directed at his parents because he is too young to understand. b. Detailed in regard to the actual procedures so he will know what to expect. c. Done several days before the procedure so that he will be prepared. d. Adapted to his level of development so that he can understand.

ANS: D Preoperative teaching should always be directed at the child's stage of development. The caregivers also benefit from the same explanations. The parents may ask additional questions, which should be answered, but the child needs to receive the information based on developmental level. This age group does not understand in-depth descriptions. Preschoolers should be prepared close to the time of the cardiac catheterization.

2. José is a 4-year-old child scheduled for a cardiac catheterization. Preoperative teaching should be: a. Directed at his parents because he is too young to understand. b. Detailed in regard to the actual procedures so he will know what to expect. c. Done several days before the procedure so that he will be prepared. d. Adapted to his level of development so that he can understand.

ANS: D Preoperative teaching should always be directed at the child's stage of development. The caregivers also benefit from the same explanations. The parents may ask additional questions, which should be answered, but the child needs to receive the information based on developmental level. This age group does not understand in-depth descriptions. Preschoolers should be prepared close to the time of the cardiac catheterization. PTS: 1 DIF: Cognitive Level: Application REF: 1320 OBJ: Nursing Process: Implementation MSC: Client Needs: Health Promotion and Maintenance

For what reason might a newborn infant with a cardiac defect, such as coarctation of the aorta, that results in a right-to-left shunt receive prostaglandin E1? a. To decrease inflammation c. To decrease respirations b. To control pain d. To improve oxygenation

ANS: D Prostaglandin E1 is given to infants with a right-to-left shunt to keep the ductus arteriosus patent. This will improve oxygenation and increase pulmonary blood flow.

41. For what reason might a newborn infant with a cardiac defect, such as coarctation of the aorta, that results in a right-to-left shunt receive prostaglandin E1? a. To decrease inflammation c. To decrease respirations b. To control pain d. To improve oxygenation

ANS: D Prostaglandin E1 is given to infants with a right-to-left shunt to keep the ductus arteriosus patent. This will improve oxygenation and increase pulmonary blood flow. PTS: 1 DIF: Cognitive Level: Analysis REF: 1324 OBJ: Nursing Process: Implementation MSC: Client Needs: Physiologic Integrity

21. The most common causative agent of bacterial endocarditis is: a. Staphylococcus albus. c. Staphylococcus albicans. b. Streptococcus hemolyticus. d. Streptococcus viridans.

ANS: D Staphylococcus viridans is the most common causative agent in bacterial (infective) endocarditis. Staphylococcus albus, Streptococcus hemolyticus, and Staphylococcus albicans are not common causative agents. PTS: 1 DIF: Cognitive Level: Comprehension REF: 1344 OBJ: Nursing Process: Assessment MSC: Client Needs: Physiologic Integrity

The most common causative agent of bacterial endocarditis is: a. Staphylococcus albus. c. Staphylococcus albicans. b. Streptococcus hemolyticus. d. Streptococcus viridans.

ANS: D Staphylococcus viridans is the most common causative agent in bacterial (infective) endocarditis.Staphylococcus albus, Streptococcus hemolyticus, and Staphylococcus albicans are not common causative agents.

Which postoperative intervention should be questioned for a child after a cardiac catheterization? a. Continue intravenous (IV) fluids until the infant is tolerating oral fluids. b. Check the dressing for bleeding. c. Assess peripheral circulation on the affected extremity. d. Keep the affected leg flexed and elevated.

ANS: D The child should be positioned with the affected leg straight for 4 to 6 hours after the procedure. IV fluid administration continues until the child is taking and retaining adequate amounts of oral fluids. The insertion site dressing should be observed frequently for bleeding. The nurse should also look under the child to check for pooled blood. Peripheral perfusion is monitored after catheterization. Distal pulses should be palpable, although they may be weaker than in the contralateral extremity.

33. Which postoperative intervention should be questioned for a child after a cardiac catheterization? a. Continue intravenous (IV) fluids until the infant is tolerating oral fluids. b. Check the dressing for bleeding. c. Assess peripheral circulation on the affected extremity. d. Keep the affected leg flexed and elevated.

ANS: D The child should be positioned with the affected leg straight for 4 to 6 hours after the procedure. IV fluid administration continues until the child is taking and retaining adequate amounts of oral fluids. The insertion site dressing should be observed frequently for bleeding. The nurse should also look under the child to check for pooled blood. Peripheral perfusion is monitored after catheterization. Distal pulses should be palpable, although they may be weaker than in the contralateral extremity. PTS: 1 DIF: Cognitive Level: Analysis REF: 1320 OBJ: Nursing Process: Implementation MSC: Client Needs: Physiologic Integrity

What is an expected assessment finding in a child with coarctation of the aorta? a. Orthostatic hypotension b. Systolic hypertension in the lower extremities c. Blood pressure higher on the left side of the body d. Disparity in blood pressure between the upper and lower extremities

ANS: D The classic finding in children with coarctation of the aorta is a disparity in pulses and blood pressures between the upper and lower extremities. Orthostatic hypotension is not present with coarctation of the aorta. Systolic hypertension may be detected in the upper extremities. The left arm may not accurately reflect systolic hypertension because the left subclavian artery can be involved in the coarctation.

37. What is an expected assessment finding in a child with coarctation of the aorta? a. Orthostatic hypotension b. Systolic hypertension in the lower extremities c. Blood pressure higher on the left side of the body d. Disparity in blood pressure between the upper and lower extremities

ANS: D The classic finding in children with coarctation of the aorta is a disparity in pulses and blood pressures between the upper and lower extremities. Orthostatic hypotension is not present with coarctation of the aorta. Systolic hypertension may be detected in the upper extremities. The left arm may not accurately reflect systolic hypertension because the left subclavian artery can be involved in the coarctation. PTS: 1 DIF: Cognitive Level: Application REF: 1324 OBJ: Nursing Process: Assessment MSC: Client Needs: Physiologic Integrity

Surgical closure of the ductus arteriosus would: a. Stop the loss of unoxygenated blood to the systemic circulation. b. Decrease the edema in legs and feet. c. Increase the oxygenation of blood. d. Prevent the return of oxygenated blood to the lungs.

ANS: D The ductus arteriosus allows blood to flow from the higher-pressure aorta to the lower-pressure pulmonary artery, causing a right-to-left shunt. If this is surgically closed, no additional oxygenated blood (from the aorta) will return to the lungs through the pulmonary artery. The aorta carries oxygenated blood to the systemic circulation. Because of the higher pressure in the aorta, blood is shunted into the pulmonary artery and the pulmonary circulation. Edema in the legs and feet is usually a sign of heart failure. This repair would not directly affect the edema. Increasing the oxygenation of blood would not interfere with the return of oxygenated blood to the lungs.

46. Surgical closure of the ductus arteriosus would: a. Stop the loss of unoxygenated blood to the systemic circulation. b. Decrease the edema in legs and feet. c. Increase the oxygenation of blood. d. Prevent the return of oxygenated blood to the lungs.

ANS: D The ductus arteriosus allows blood to flow from the higher-pressure aorta to the lower-pressure pulmonary artery, causing a right-to-left shunt. If this is surgically closed, no additional oxygenated blood (from the aorta) will return to the lungs through the pulmonary artery. The aorta carries oxygenated blood to the systemic circulation. Because of the higher pressure in the aorta, blood is shunted into the pulmonary artery and the pulmonary circulation. Edema in the legs and feet is usually a sign of heart failure. This repair would not directly affect the edema. Increasing the oxygenation of blood would not interfere with the return of oxygenated blood to the lungs. PTS: 1 DIF: Cognitive Level: Analysis REF: 1323 OBJ: Nursing Process: Planning MSC: Client Needs: Physiologic Integrity: Physiologic Adaptation

The nurse is obtaining a health history from a 24-year-old patient with hypertrophic cardiomyopathy (HC). Which information obtained by the nurse is most important? d. The patient's 29-year-old brother died from a sudden cardiac arrest.

About half of all cases of HC have a genetic basis, and it is the most common cause of sudden cardiac death in otherwise healthy young people.

Disease processes or abnormalities that occur after birth and can be seen in the normal heart or in the presence of congenital heart defects; resulting from factors such as infection, autoimmune responses, environmental factors, and familial tendencies

Acquired cardiac disorders

Which nursing diagnosis would be priority for the client diagnosed with myocarditis? Activity intolerance related to impaired cardiac muscle function.

Activity intolerance is priority for the client with myocarditis, an inflammation of the heart muscle. Nursing care is aimed at decreasing myocardial work and maintaining cardiac output.

Based on assessment data for a patient with pericarditis, select a primary nursing diagnosis

Acute pain related to inflammation of the pericardium

The client is diagnosed with acute pericarditis. Which sign/symptom warrants immediate attention by the nurse? 1. Muffled heart sounds.

Acute pericardial effusion interferes with normal cardiac filling and pumping, causing venous congestion and decreased cardiac output. Muffled heart sounds, indicative of acute pericarditis, must be reported to the health-care provider.

Which interventions should the nurse discuss with the client diagnosed with atherosclerosis?Select all that apply. 1. Include significant other in the discussion. 2. Stop smoking or using any type of tobacco products

Adherence to lifestyle modifications is enhanced when the client receives support from significant others. 2. Tobacco use is the most significant modifi- able risk factor that contributes to the development of atherosclerosis.

The nurse is preparing to administer warfarin (Coumadin), an oral anticoagulant, to a patient with a mechanical valve replacement. The patient's INR is 2.7. Which action should the nurse take?

Administer the medication as ordered. Patients with mechanical valve replacements who take warfarin (Coumadin) usually have individualized target international normalized ratios (INRs) between 2.0 and 3.5. The nurse would give the medication as ordered

When caring for a patient who has just arrived on the medical-surgical unit after having cardiac catheterization, which nursing intervention should the nurse delegate to a licensed practical/vocational nurse (LPN/LVN)? a. Give the scheduled aspirin and lipid-lowering medication..

Administration of oral medications is within the scope of practice for LPNs/LVNs.

B. Record the data on the nurse's notes. Elevation is not necessary; the extremity is kept straight. The pulse distal to the catheter insertion site may be weaker for the first few hours after catheterization. It should gradually increase in strength. Because a weaker pulse is an expected finding, the nurse should document this and continue to monitor. The insertion site is kept dry.

After a patient returns from cardiac catheterization, the nurse assesses that the pulse distal to the catheter insertion site is weaker. The nurse should: A. Elevate the affected extremity. B. Record the data on the nurse's notes. C. Notify the physician of the observation. D. Apply warm compresses to the insertion site.

. The nurse and an unlicensed nursing assistant are caring for a 64-year-old client who is four (4) hours post-operative bilateral femoral-popliteal bypass surgery. Which nursing task should be delegated to the unlicensed nursing assistant? Elevate the foot of the client's bed.

After the surgery, the client's legs will be elevated to help decrease edema. The surgery has corrected the decreased blood supply to the lower legs.

A patient admitted to the coronary care unit (CCU) with an ST-segment-elevation myocardial infarction (STEMI) is restless and anxious. The blood pressure is 86/40 and heart rate is 123. Based on this information, which nursing diagnosis is a priority for the patient? c. Stress overload related to acute change in health

All the nursing diagnoses may be appropriate for this patient, but the hypotension and tachycardia indicate decreased cardiac output and shock from the damaged myocardium

Which of the following medications is considered a thrombolytic?

Alteplase

A nurse plans to have an education session with a client with cardiomyopathy and the client's spouse about ways to increase activity tolerance. Which of the following instructions would provide that information?

Alternative active periods with rest periods

Which statement made by a patient with coronary artery disease after the nurse has completed teaching about therapeutic lifestyle changes (TLC) diet indicates that further teaching is needed? d. "I will miss being able to eat peanut butter sandwiches."

Although only 30% of the daily calories should come from fats, most of the fat in the TLC diet should come from monosaturated fats such as are found in nuts, olive oil, and canola oil.

The nurse is providing teaching to a patient recovering from an MI. How should resumption of sexual activity be discussed?r b) Discussed along with other physical activities

Although some nurses may not feel comfortable discussing sexual activity with patients, it is a necessary component of patient teaching.

A patient experienced sudden cardiac death (SCD) and survived. What should the nurse expect to be used as preventive treatment for the patient? d) Implantable cardioverter-defibrillator (ICD)

An ICD is the most common approach to preventing recurrence of SCD.

The nurse on the telemetry unit has just received the A.M. shift report. Which client should the nurse assess first? 1. The client diagnosed with myocardial infarction who has an audible S3 heart sound.

An S3 heart sound indicates left ventricular failure, and the nurse must assess this client first because it is an emergency situation.

The intensive care department nurse is assessing the client who is 12 hours postmyocardial infarction. The nurse assesses a S3 heart sound. Which intervention should the nurse implement? 1. Notify the health-care provider immediately.

An S3 indicates left ventricular failure and should be reported to the health-care provider. It is a potential life-threatening complication of a myocardial infarction.

The nurse is unable to assess a pedal pulse in the client diagnosed with arterial occlusive disease. Which intervention should the nurse implement first? 1. Complete a neurovascular assessment.

An absent pulse is not uncommon in a client diagnosed with arterial occlusive disease, but the nurse must ensure that the feet can be moved and are warm, which indicates adequate blood supply to the feet.

A. Tachypnea. Tachypnea is one of the early signs that should be identified. Tachycardia at rest, dyspnea, retractions, and activity intolerance are other physical signs and symptoms. Tachycardia, not bradycardia, is one of the symptoms suggestive of congestive heart failure. The child may be diaphoretic. Urine output usually will be decreased due to decreased kidney perfusion.

An early sign of congestive heart failure that the nurse should recognize is: A. Tachypnea. B. Bradycardia. C. Inability to sweat. D. Increased urine output.

The nurse is assessing the client diagnosed with congestive heart failure. Which signs/symptoms would indicate that the medical treatment has been effective?

An elevated B-type natriuretic peptide (BNP). specific test, higher the number worse the condition

The nurse is auscultating the heart of a patient diagnosed with mitral valve prolapse. Which of the following is often the first and only manifestation of mitral valve prolapse?

An extra heart sound

C. Count the apical rate for 1 full minute and compare it with the radial rate. This may be indicated after conferring with the practitioner. The radial pulse needs to be compared with the apical. This is the nurse's first action. If an arrhythmia is occurring, the radial pulse may be lower than the apical rate. It is the responsibility of the nurse to check both rates.

An important nursing responsibility when a dysrhythmia is suspected is to: A. Order an immediate electrocardiogram. B. Count the radial rate at 1-minute intervals 5 times in a row. C. Count the apical rate for 1 full minute and compare it with the radial rate. D. Have someone else take the radial rate while the nurse simultaneously checks the apical rate.

The health-care provider prescribes an ACE inhibitor for the client diagnosed with essential hypertension. Which statement is the most appropriate rationale for administering this medication? 3. ACE inhibitors prevent vasoconstriction and sodium and water retention.

Angiotensin-converting enzyme (ACE) inhibitors prevent the conversion of angiotensin I to angiotensin II, and this, in turn, prevents vasoconstriction and sodium and water retention.

The nurse is planning the care of a patient with HF. The nurse should identify what overall goals of this patient's care? Select all that applies A) Improve functional status B) Prevent endocarditis. C) Extend survival. D) Limit physical activity. E) Relieve patient symptoms.

Ans: A) Improve functional status, C) Extend survival, E) Relieve patient symptoms Feedback: The overall goals of management of HF are to relieve the patient's symptoms, to improve functional status and quality of life, and to extend survival. Activity limitations should be accommodated, but reducing activity is not a goal. Endocarditis is not a common complication of HF and preventing it is not a major goal of care.

The critical care nurse is caring for a patient who is in cardiogenic shock. What assessments must the nurse perform on this patient? Select all that apply. A) Platelet level B) Fluid status C) Cardiac rhythm D) Action of medications E) Sputum volume

Ans: B) Fluid status, C) Cardiac rhythm, D) Action of medications Feedback: The critical care nurse must carefully assess the patient in cardiogenic shock, observe the cardiac rhythm, monitor hemodynamic parameters, monitor fluid status, and adjust medications and therapies based on the assessment data. Platelet levels and sputum production are not major assessment parameters in a patient who is experiencing cardiogenic shock.

2. A patient with primary hypertension comes to the clinic complaining of a gradual onset of blurry vision and decreased visual acuity over the past several weeks. The nurse is aware that these symptoms could be indicative of what? A) Retinal blood vessel damage B) Glaucoma C) Cranial nerve damage D) Hypertensive emergency

Ans: A Feedback: Blurred vision, spots in front of the eyes, and diminished visual acuity can mean retinal blood vessel damage indicative of damage elsewhere in the vascular system as a result of hypertension. Glaucoma and cranial nerve damage do not normally cause these symptoms. A hypertensive emergency would have a more rapid onset.

20. The nurse is developing a nursing care plan for a patient who is being treated for hypertension. What is a measurable patient outcome that the nurse should include? A) Patient will reduce Na+ intake to no more than 2.4 g daily. B) Patient will have a stable BUN and serum creatinine levels. C) Patient will abstain from fat intake and reduce calorie intake. D) Patient will maintain a normal body weight.

Ans: A Feedback: Dietary sodium intake of no more than 2.4 g sodium is recommended as a dietary lifestyle modification to prevent and manage hypertension. Giving a specific amount of allowable sodium intake makes this a measurable goal. None of the other listed goals is quantifiable and measurable.

14. A nurse is teaching an adult female patient about the risk factors for hypertension. What should the nurse explain as risk factors for primary hypertension? A) Obesity and high intake of sodium and saturated fat B) Diabetes and use of oral contraceptives C) Metabolic syndrome and smoking D) Renal disease and coarctation of the aorta

Ans: A Feedback: Obesity, stress, high intake of sodium or saturated fat, and family history are all risk factors for primary hypertension. Diabetes and oral contraceptives are risk factors for secondary hypertension. Metabolic syndrome, renal disease, and coarctation of the aorta are causes of secondary hypertension.

10. A patient with primary hypertension complains of dizziness with ambulation. The patient is currently on an alpha-adrenergic blocker and the nurse assesses characteristic signs and symptoms of postural hypotension. When teaching this patient about risks associated with postural hypotension, what should the nurse emphasize? A) Rising slowly from a lying or sitting position B) Increasing fluids to maintain BP C) Stopping medication if dizziness persists D) Taking medication first thing in the morning

Ans: A Feedback: Patients who experience postural hypotension should be taught to rise slowly from a lying or sitting position and use a cane or walker if necessary for safety. It is not necessary to teach these patients about increasing fluids or taking medication in the morning (this would increase the effects of dizziness). Patient should not be taught to stop the medication if dizziness persists because this is unsafe and beyond the nurse's scope of practice.

18. A patient has come to the clinic for a follow-up assessment that will include a BP reading. To ensure an accurate reading, the nurse should confirm that the patient has done which of the following? A) Tried to rest quietly for 5 minutes before the reading is taken B) Refrained from smoking for at least 8 hours C) Drunk adequate fluids during the day prior D) Avoided drinking coffee for 12 hours before the visit

Ans: A Feedback: Prior to the nurse assessing the patient's BP, the patient should try to rest quietly for 5 minutes. The forearm should be positioned at heart level. Caffeine products and cigarette smoking should be avoided for at least 30 minutes prior to the visit. Recent fluid intake is not normally relevant.

16. A patient with secondary hypertension has come into the clinic for a routine check-up. The nurse is aware that the difference between primary hypertension and secondary hypertension is which of the following? A) Secondary hypertension has a specific cause. B) Secondary hypertension has a more gradual onset than primary hypertension. C) Secondary hypertension does not cause target organ damage. D) Secondary hypertension does not normally respond to antihypertensive drug therapy.

Ans: A Feedback: Secondary hypertension has a specific identified cause. A cause could include narrowing of the renal arteries, renal parenchymal disease, hyperaldosteronism, certain medications, pregnancy, and coarctation of the aorta. Secondary hypertension does respond to antihypertensive drug therapy and can cause target organ damage if left untreated.

7. A patient has been prescribed antihypertensives. After assessment and analysis, the nurse has identified a nursing diagnosis of risk for ineffective health maintenance related to nonadherence to therapeutic regimen. When planning this patient's care, what desired outcome should the nurse identify? A) Patient takes medication as prescribed and reports any adverse effects. B) Patient's BP remains consistently below 140/90 mm Hg. C) Patient denies signs and symptoms of hypertensive urgency. D) Patient is able to describe modifiable risk factors for hypertension.

Ans: A Feedback: The most appropriate expected outcome for a patient who is given the nursing diagnosis of risk for ineffective health maintenance is that he or she takes the medication as prescribed. The other listed goals are valid aspects of care, but none directly relates to the patient's role in his or her treatment regimen.

9. A patient newly diagnosed with hypertension asks the nurse what happens when uncontrolled hypertension is prolonged. The nurse explains that a patient with prolonged, uncontrolled hypertension is at risk for developing what health problem? A) Renal failure B) Right ventricular hypertrophy C) Glaucoma D) Anemia

Ans: A Feedback: When uncontrolled hypertension is prolonged, it can result in renal failure, myocardial infarction, stroke, impaired vision, left ventricular hypertrophy, and cardiac failure. Glaucoma and anemia are not directly associated with hypertension.

A nurse is caring for a patient who is exhibiting ventricular tachycardia (VT). Because the patient is pulseless, the nurse should prepare for what intervention? A) Defibrillation B) ECG monitoring C) Implantation of a cardioverter defibrillator D) Angioplasty

Ans: A Feedback: Any type of VT in a patient who is unconscious and without a pulse is treated in the same manner as ventricular fibrillation: Immediate defibrillation is the action of choice. ECG monitoring is appropriate, but this is an assessment, not an intervention, and will resolve the problem. An ICD and angioplasty do not address the dysrhythmia.

A patient is brought to the ED and determined to be experiencing symptomatic sinus bradycardia. The nurse caring for this patient is aware the medication of choice for treatment of this dysrhythmia is the administration of atropine. What guidelines will the nurse follow when administering atropine? A) Administer atropine 0.5 mg as an IV bolus every 3 to 5 minutes to a maximum of 3.0 mg. B) Administer atropine as a continuous infusion until symptoms resolve. C) Administer atropine as a continuous infusion to a maximum of 30 mg in 24 hours. D) Administer atropine 1.0 mg sublingually.

Ans: A Feedback: Atropine 0.5 mg given rapidly as an intravenous (IV) bolus every 3 to 5 minutes to a maximum total dose of 3.0 mg is the medication of choice in treating symptomatic sinus bradycardia. By this guideline, the other listed options are inappropriate.

The nurse is caring for a patient who has had a biventricular pacemaker implanted. When planning the patient's care, the nurse should recognize what goal of this intervention? A) Resynchronization B) Defibrillation C) Angioplasty D) Ablation

Ans: A Feedback: Biventricular (both ventricles) pacing, also called resynchronization therapy, may be used to treat advanced heart failure that does not respond to medication. This type of pacing therapy is not called defibrillation, angioplasty, or ablation therapy.

The nurse is caring for a patient who has had an ECG. The nurse notes that leads I, II, and III differ from one another on the cardiac rhythm strip. How should the nurse best respond? A) Recognize that the view of the electrical current changes in relation to the lead placement. B) Recognize that the electrophysiological conduction of the heart differs with lead placement. C) Inform the technician that the ECG equipment has malfunctioned. D) Inform the physician that the patient is experiencing a new onset of dysrhythmia.

Ans: A Feedback: Each lead offers a different reference point to view the electrical activity of the heart. The lead displays the configuration of electrical activity of the heart. Differences between leads are not necessarily attributable to equipment malfunction or dysrhythmias.

The nurse and the other members of the team are caring for a patient who converted to ventricular fibrillation (VF). The patient was defibrillated unsuccessfully and the patient remains in VF. According to national standards, the nurse should anticipate the administration of what medication? A) Epinephrine 1 mg IV push B) Lidocaine 100 mg IV push C) Amiodarone 300 mg IV push D) Sodium bicarbonate 1 amp IV push

Ans: A Feedback: Epinephrine should be administered as soon as possible after the first unsuccessful defibrillation and then every 3 to 5 minutes. Antiarrhythmic medications such as amiodarone and licocaine are given if ventricular dysrhythmia persists.

The nurse is caring for a patient who has just undergone catheter ablation therapy. The nurse in the step-down unit should prioritize what assessment? A) Cardiac monitoring B) Monitoring the implanted device signal C) Pain assessment D) Monitoring the patient's level of consciousness (LOC)

Ans: A Feedback: Following catheter ablation therapy, the patient is closely monitored to ensure the dysrhythmia does not reemerge. This is a priority over monitoring of LOC and pain, although these are valid and important assessments. Ablation does not involve the implantation of a device.

The nurse is caring for a patient who is in the recovery room following the implantation of an ICD. The patient has developed ventricular tachycardia (VT). What should the nurse assess and document? A) ECG to compare time of onset of VT and onset of device's shock B) ECG so physician can see what type of dysrhythmia the patient has C) Patient's level of consciousness (LOC) at the time of the dysrhythmia D) Patient's activity at time of dysrhythmia

Ans: A Feedback: If the patient has an ICD implanted and develops VT or ventricular fibrillation, the ECG should be recorded to note the time between the onset of the dysrhythmia and the onset of the device's shock or antitachycardia pacing. This is a priority over LOC or activity at the time of onset.

A patient is undergoing preoperative teaching before his cardiac surgery and the nurse is aware that a temporary pacemaker will be placed later that day. What is the nurse's responsibility in the care of the patient's pacemaker? A) Monitoring for pacemaker malfunction or battery failure B) Determining when it is appropriate to remove the pacemaker C) Making necessary changes to the pacemaker settings D) Selecting alternatives to future pacemaker use

Ans: A Feedback: Monitoring for pacemaker malfunctioning and battery failure is a nursing responsibility. The other listed actions are physician responsibilities.

A patient who is a candidate for an implantable cardioverter defibrillator (ICD) asks the nurse about the purpose of this device. What would be the nurse's best response? A) "To detect and treat dysrhythmias such as ventricular fibrillation and ventricular tachycardia" B) "To detect and treat bradycardia, which is an excessively slow heart rate" C) "To detect and treat atrial fibrillation, in which your heart beats too quickly and inefficiently" D) "To shock your heart if you have a heart attack at home"

Ans: A Feedback: The ICD is a device that detects and terminates life-threatening episodes of ventricular tachycardia and ventricular fibrillation. It does not treat atrial fibrillation, MI, or bradycardia.

A patient the nurse is caring for has a permanent pacemaker implanted with the identification code beginning with VVI. What does this indicate? A) Ventricular paced, ventricular sensed, inhibited B) Variable paced, ventricular sensed, inhibited C) Ventricular sensed, ventricular situated, implanted D) Variable sensed, variable paced, inhibited

Ans: A Feedback: The identification of VVI indicates ventricular paced, ventricular sensed, inhibited.

The nurse is caring for an adult patient who has gone into ventricular fibrillation. When assisting with defibrillating the patient, what must the nurse do? A) Maintain firm contact between paddles and patient skin. B) Apply a layer of water as a conducting agent. C) Call ìall clearî once before discharging the defibrillator. D) Ensure the defibrillator is in the sync mode.

Ans: A Feedback: When defibrillating an adult patient, the nurse should maintain good contact between the paddles and the patient's skin to prevent arcing, apply an appropriate conducting agent (not water) between the skin and the paddles, and ensure the defibrillator is in the nonsync mode. ìClearî should be called three times before discharging the paddles.

The nurse is reviewing the medication administration record of a patient diagnosed with systolic HF. What medication should the nurse anticipate administering to this patient? A) A beta-adrenergic blocker B) An antiplatelet aggregator C) A calcium channel blocker D) A nonsteroidal anti-inflammatory drug (NSAID)

Ans: A) A beta-adrenergic blocker Feedback: Several medications are routinely prescribed for systolic HF, including ACE inhibitors, beta-blockers, diuretics, and digitalis. Calcium channel blockers, antiplatelet aggregators, and NSAIDs are not commonly prescribed.

A cardiac patient's resistance to left ventricular filling has caused blood to back up into the patient's circulatory system. What health problem is likely to result? A) Acute pulmonary edema B) Right-sided HF C) Right ventricular hypertrophy D) Left-sided HF

Ans: A) Acute pulmonary edema Feedback: With increased resistance to left ventricular filling, blood backs up into the pulmonary circulation. The patient quickly develops pulmonary edema from the blood volume overload in the lungs. When the blood backs up into the pulmonary circulation, right-sided HF, left-sided HF, and right ventricular hypertrophy do not directly occur.

The nurse is performing a physical assessment on a patient suspected of having HF. The presence of what sound would signal the possibility of impending HF? A) An S3 heart sound B) Pleural friction rub C) Faint breath sounds D) A heart murmur

Ans: A) An S3 heart sound Feedback: The heart is auscultated for an S3 heart sound, a sign that the heart is beginning to fail and that increased blood volume fills the ventricle with each beat. HF does not normally cause a pleural friction rub or murmurs. Changes in breath sounds occur, such as the emergence of crackles or wheezes, but faint breath sounds are less characteristic of HF.

A patient who is at high risk for developing intracardiac thrombi has been placed on long-term anticoagulation. What aspect of the patient's health history creates a heightened risk of intracardiac thrombi? A) Atrial fibrillation B) Infective endocarditis C) Recurrent pneumonia D) Recent surgery

Ans: A) Atrial fibrillation Feedback: Intracardiac thrombi are especially common in patients with atrial fibrillation, because the atria do not contract forcefully and blood flows slowly and turbulently, increasing the likelihood of thrombus formation. Endocarditis, pneumonia, and recent surgery do not normally cause an increased risk for intracardiac thrombi formation.

A patient with HF has met with his primary care provider and begun treatment with an angiotensin-converting enzyme (ACE) inhibitor. When the patient begins treatment, the nurse should prioritize what assessment? A) Blood pressure B) Level of consciousness (LOC) C) Assessment for nausea D) Oxygen saturation

Ans: A) Blood pressure Feedback: Patients receiving ACE inhibitors are monitored for hypotension, hyperkalemia (increased potassium in the blood), and alterations in renal function. ACE inhibitors do not typically cause alterations in LOC. Oxygen saturation must be monitored in patients with HF, but this is not particular to ACE inhibitor therapy. ACE inhibitors do not normally cause nausea.

The nurse is caring for an adult patient with HF who is prescribed digoxin. When assessing the patient for adverse effects, the nurse should assess for which of the following signs and symptoms? A) Confusion and bradycardia B) Uncontrolled diuresis and tachycardia C) Numbness and tingling in the extremities D) Chest pain and shortness of breath

Ans: A) Confusion and bradycardia Feedback: A key concern associated with digitalis therapy is digitalis toxicity. Symptoms include anorexia, nausea, visual disturbances, confusion, and bradycardia. The other listed signs and symptoms are not characteristic of digitalis toxicity.

A patient with a diagnosis of HF is started on a beta-blocker. What is the nurse's priority role during gradual increases in the patient's dose? A) Educating the patient that symptom relief may not occur for several weeks B) Stressing that symptom relief may take up to 4 months to occur C) Making adjustments to each day's dose based on the blood pressure trends D) Educating the patient about the potential changes in LOC that may result from the drug

Ans: A) Educating the patient that symptom relief may not occur for several weeks Feedback: An important nursing role during titration is educating the patient about the potential worsening of symptoms during the early phase of treatment and stressing that improvement may take several weeks. Relief does not take 4 months, however. The nurse monitors blood pressure, but changes are not made based on short-term assessment results. Beta-blockers rarely affect LOC.

The nurse is creating a care plan for a patient diagnosed with HF. When addressing the problem of anxiety, what interventions should the nurse include in the care plan? Select all that apply. A) Facilitate the presence of friends and family whenever possible. B) Teach the patient about the harmful effects of anxiety on cardiac function. C) Provide supplemental oxygen, as needed. D) Provide validation of the patient's expressions of anxiety. E) Administer benzodiazepines two to three times daily.

Ans: A) Facilitate the presence of friends and family whenever possible C) Provide supplemental oxygen, as needed D) Provide validation of the patient's expressions of anxiety Feedback: The nurse should empathically validate the patient's sensations of anxiety. The presence of friends and family are frequently beneficial and oxygen supplementation promotes comfort. Antianxiety medications may be necessary for some patients, but alternative methods of relief should be prioritized. As well, medications are administered on a PRN basis. Teaching the patient about the potential harms of anxiety is likely to exacerbate, not relieve, the problem.

The nurse is performing an initial assessment of a client diagnosed with HF. The nurse also assesses the patient's sensorium and LOC. Why is the assessment of the patient's sensorium and LOC important in patients with HF? A) HF ultimately affects oxygen transportation to the brain. B) Patients with HF are susceptible to overstimulation of the sympathetic nervous system. C) Decreased LOC causes an exacerbation of the signs and symptoms of HF. D) The most significant adverse effect of medications used for HF treatment is altered LOC.

Ans: A) HF ultimately affects oxygen transportation to the brain. Feedback: As the volume of blood ejected by the heart decreases, so does the amount of oxygen transported to the brain. Sympathetic stimulation is not a primary concern in patients with HF, although it is a possibility. HF affects LOC but the reverse is not usually true. Medications used to treat HF carry many adverse effects, but the most common and significant effects are cardiovascular.

A patient admitted to the medical unit with HF is exhibiting signs and symptoms of pulmonary edema. The nurse is aware that positioning will promote circulation. How should the nurse best position the patient? A) In a high Fowler's position B) On the left side-lying position C) In a flat, supine position D) In the Trendelenburg position

Ans: A) In a high Fowler's position Feedback: Proper positioning can help reduce venous return to the heart. The patient is positioned upright. If the patient is unable to sit with the lower extremities dependent, the patient may be placed in an upright position in bed. The supine position and Trendelenburg positions will not reduce venous return, lower the output of the right ventricle, or decrease lung congestion. Similarly, side-lying does not promote circulation.

The nurse is caring for a patient with severe left ventricular dysfunction who has been identified as being at risk for sudden cardiac death. What medical intervention can be performed that may extend the survival of the patient? A) Insertion of an implantable cardioverter defibrillator B) Insertion of an implantable pacemaker C) Administration of a calcium channel blocker D) Administration of a beta-blocker

Ans: A) Insertion of an implantable cardioverter defibrillator Feedback: In patients with severe left ventricular dysfunction and the possibility of life-threatening dysrhythmias, placement of an implantable cardioverter defibrillator (ICD) can prevent sudden cardiac death and extend survival. A pacemaker, a calcium channel blocker, and a beta-blocker are not medical interventions that may extend the survival of the patient with left ventricular dysfunction.

The nurse is providing patient education prior to a patient's discharge home after treatment for HF. The nurse gives the patient a home care checklist as part of the discharge teaching. What should be included on this checklist? A) Know how to recognize and prevent orthostatic hypotension. B) Weigh yourself weekly at a consistent time of day. C) Measure everything you eat and drink until otherwise instructed. D) Limit physical activity to only those tasks that are absolutely necessary.

Ans: A) Know how to recognize and prevent orthostatic hypotension. Feedback: Patients with HF should be aware of the risks of orthostatic hypotension. Weight should be measured daily; detailed documentation of all forms of intake is not usually required. Activity should be gradually increased within the parameters of safety and comfort.

Cardiopulmonary resuscitation has been initiated on a patient who was found unresponsive. When performing chest compressions, the nurse should do which of the following? A) Perform at least 100 chest compressions per minute. B) Pause to allow a colleague to provide a breath every 10 compressions. C) Pause chest compressions to allow for vital signs monitoring every 4 to 5 minutes. D) Perform high-quality chest compressions as rapidly as possible.

Ans: A) Perform at least 100 chest compressions per minute. Feedback: During CPR, the chest is compressed 2 inches at a rate of at least 100 compressions per minute. This rate is the resuscitator's goal; the aim is not to give compressions as rapidly as possible. Compressions are not stopped after 10 compressions to allow for a breath or for full vital signs monitoring.

A cardiovascular patient with a previous history of pulmonary embolism (PE) is experiencing a sudden onset of dyspnea, rapid breathing, and chest pain. The nurse recognizes the characteristic signs and symptoms of a PE. What is the nurse's best action? A) Rapidly assess the patient's cardiopulmonary status. B) Arrange for an ECG. C) Increase the height of the patient's bed. D) Manage the patient's anxiety.

Ans: A) Rapidly assess the patient's cardiopulmonary status. Feedback: Patient management in the event of a PE begins with cardiopulmonary assessment and intervention. This is a priority over ECG monitoring, management of anxiety, or repositioning of the patient, even though each of these actions may be appropriate and necessary.

A nurse in the CCU is caring for a patient with HF who has developed an intracardiac thrombus. This creates a high risk for what sequela? A) Stroke B) Myocardial infarction (MI) C) Hemorrhage D) Peripheral edema

Ans: A) Stroke Feedback: Intracardiac thrombi can become lodged in the cerebral vasculature, causing stroke. There is no direct risk of MI, hemorrhage, or peripheral edema.

23. The nurse is teaching a patient about some of the health consequences of uncontrolled hypertension. What health problems should the nurse describe? Select all that apply. A) Transient ischemic attacks B) Cerebrovascular accident C) Retinal hemorrhage D) Venous insufficiency E) Right ventricular hypertrophy

Ans: A, B, C Feedback: Potential complications of hypertension include the following: left ventricular hypertrophy; MI; heart failure; transient ischemic attacks (TIAs); cerebrovascular accident; renal insufficiency and failure; and retinal hemorrhage. Venous insufficiency and right ventricular hypertrophy are not potential complications of uncontrolled hypertension.

8. The nurse is providing care for a patient with a new diagnosis of hypertension. How can the nurse best promote the patient's adherence to the prescribed therapeutic regimen? A) Screen the patient for visual disturbances regularly. B) Have the patient participate in monitoring his or her own BP. C) Emphasize the dire health outcomes associated with inadequate BP control. D) Encourage the patient to lose weight and exercise regularly.

Ans: B Feedback: Adherence to the therapeutic regimen increases when patients actively participate in self-care, including self-monitoring of BP and diet. Dire warnings may motivate some patients, but for many patients this is not an appropriate or effective strategy. Screening for vision changes and promoting healthy lifestyle are appropriate nursing actions, but do not necessarily promote adherence to a therapeutic regimen.

13. The nursing lab instructor is teaching student nurses how to take blood pressure. To ensure accurate measurement, the lab instructor would teach the students to avoid which of the following actions? A) Measuring the BP after the patient has been seated quietly for more than 5 minutes B) Taking the BP at least 10 minutes after nicotine or coffee ingestion C) Using a cuff with a bladder that encircles at least 80% of the limb D) Using a bare forearm supported at heart level on a firm surface

Ans: B Feedback: Blood pressures should be taken with the patient seated with arm bare, supported, and at heart level. The patient should not have smoked tobacco or taken caffeine in the 30 minutes preceding the measurement. The patient should rest quietly for 5 minutes before the reading is taken. The cuff bladder should encircle at least 80% of the limb being measured and have a width of at least 40% of limb circumference. Using a cuff that is too large results in a lower BP and a cuff that is too small will give a higher BP measurement.

22. The hospital nurse cares for many patients who have hypertension. What nursing diagnosis is most common among patients who are being treated for this health problem? A) Deficient knowledge regarding the lifestyle modifications for management of hypertension B) Noncompliance with therapeutic regimen related to adverse effects of prescribed therapy C) Deficient knowledge regarding BP monitoring D) Noncompliance with treatment regimen related to medication costs

Ans: B Feedback: Deviation from the therapeutic program is a significant problem for people with hypertension and other chronic conditions requiring lifetime management. For many patients, this is related to adverse effects of medications. Medication cost is relevant for many patients, but adverse effects are thought to be a more significant barrier. Many patients are aware of necessary lifestyle modification, but do not adhere to them. Most patients are aware of the need to monitor their BP.

15. The nurse is caring for an older adult with a diagnosis of hypertension who is being treated with a diuretic and beta-blocker. Which of the following should the nurse integrate into the management of this client's hypertension? A) Ensure that the patient receives a larger initial dose of antihypertensive medication due to impaired absorption. B) Pay close attention to hydration status because of increased sensitivity to extracellular volume depletion. C) Recognize that an older adult is less likely to adhere to his or her medication regimen than a younger patient. D) Carefully assess for weight loss because of impaired kidney function resulting from normal aging.

Ans: B Feedback: Elderly people have impaired cardiovascular reflexes and thus are more sensitive to extracellular volume depletion caused by diuretics. The nurse needs to assess hydration status, low BP, and postural hypotension carefully. Older adults may have impaired absorption, but they do not need a higher initial dose of an antihypertensive than a younger person. Adherence to treatment is not necessarily linked to age. Kidney function and absorption decline with age; less, rather than more antihypertensive medication is prescribed. Weight gain is not necessarily indicative of kidney function decline.

27. During an adult patient's last two office visits, the nurse obtained BP readings of 122/84 mm Hg and 130/88 mm Hg, respectively. How would this patient's BP be categorized? A) Normal B) Prehypertensive C) Stage 1 hypertensive D) Stage 2 hypertensive

Ans: B Feedback: Prehypertension is defined systolic BP of 120 to 139 mm Hg or diastolic BP of 80 to 89 mm Hg.

6. A 40-year-old male newly diagnosed with hypertension is discussing risk factors with the nurse. The nurse talks about lifestyle changes with the patient and advises that the patient should avoid tobacco use. What is the primary rationale behind that advice to the patient? A) Quitting smoking will cause the patient's hypertension to resolve. B) Tobacco use increases the patient's concurrent risk of heart disease. C) Tobacco use is associated with a sedentary lifestyle. D) Tobacco use causes ventricular hypertrophy.

Ans: B Feedback: Smoking increases the risk for heart disease, for which a patient with hypertension is already at an increased risk. Quitting will not necessarily cause hypertension to resolve and smoking does not directly cause ventricular hypertrophy. The association with a sedentary lifestyle is true, but this is not the main rationale for the nurse's advice; the association with heart disease is more salient.

11. The nurse is planning the care of a patient who has been diagnosed with hypertension, but who otherwise enjoys good health. When assessing the response to an antihypertensive drug regimen, what blood pressure would be the goal of treatment? A) 156/96 mm Hg or lower B) 140/90 mm Hg or lower C) Average of 2 BP readings of 150/80 mm Hg D) 120/80 mm Hg or lower

Ans: B Feedback: The goal of antihypertensive drug therapy is a BP of 140/90 mm Hg or lower. A pressure of 130/80 mm Hg is the goal for patients with diabetes or chronic kidney disease.

39. A patient's medication regimen for the treatment of hypertension includes hydrochlorothiazide. Following administration of this medication, the nurse should anticipate what effect? A) Drowsiness or lethargy B) Increased urine output C) Decreased heart rate D) Mild agitation

Ans: B Feedback: Thiazide diuretics lower BP by reducing circulating blood volume; this results in a short-term increase in urine output. These drugs do not cause bradycardia, agitation, or drowsiness.

29. A patient has been diagnosed as being prehypertensive. What should the nurse encourage this patient to do to aid in preventing a progression to a hypertensive state? A) Avoid excessive potassium intake. B) Exercise on a regular basis. C) Eat less protein and more vegetables. D) Limit morning activity.

Ans: B Feedback: To prevent or delay progression to hypertension and reduce risk, JNC 7 urged health care providers to encourage people with blood pressures in the prehypertension category to begin lifestyle modifications, such as nutritional changes and exercise. There is no need for patients to limit their activity in the morning or to avoid potassium and protein intake.

A patient has returned to the cardiac care unit after having a permanent pacemaker implantation. For which potential complication should the nurse most closely assess this patient? A) Chest pain B) Bleeding at the implantation site C) Malignant hyperthermia D) Bradycardia

Ans: B Feedback: Bleeding, hematomas, local infections, perforation of the myocardium, and tachycardia are complications of pacemaker implantations. The nurse should monitor for chest pain and bradycardia, but bleeding is a more common immediate complication. Malignant hyperthermia is unlikely because it is a response to anesthesia administration.

A patient is scheduled for catheter ablation therapy. When describing this procedure to the patient's family, the nurse should address what aspect of the treatment? A) Resetting of the heart's contractility B) Destruction of specific cardiac cells C) Correction of structural cardiac abnormalities D) Clearance of partially occluded coronary arteries

Ans: B Feedback: Catheter ablation destroys specific cells that are the cause or central conduction route of a tachydysrhythmia. It does not ìresetî the heart's contractility and it does not address structural or vascular abnormalities.

When planning the care of a patient with an implanted pacemaker, what assessment should the nurse prioritize? A) Core body temperature B) Heart rate and rhythm C) Blood pressure D) Oxygen saturation level

Ans: B Feedback: For patients with pacemakers, close monitoring of the heart rate and rhythm is a priority, even though each of the other listed vital signs must be assessed.

The nurse caring for a patient whose sudden onset of sinus bradycardia is not responding adequately to atropine. What might be the treatment of choice for this patient? A) Implanted pacemaker B) Trancutaneous pacemaker C) ICD D) Asynchronous defibrillator

Ans: B Feedback: If a patient suddenly develops a bradycardia, is symptomatic but has a pulse, and is unresponsive to atropine, emergency pacing may be started with transcutaneous pacing, which most defibrillators are now equipped to perform. An implanted pacemaker is not a time-appropriate option. An asynchronous defibrillator or ICD would not provide relief.

The ED nurse is caring for a patient who has gone into cardiac arrest. During external defibrillation, what action should the nurse perform? A) Place gel pads over the apex and posterior chest for better conduction. B) Ensure no one is touching the patient at the time shock is delivered. C) Continue to ventilate the patient via endotracheal tube during the procedure. D) Allow at least 3 minutes between shocks.

Ans: B Feedback: In external defibrillation, both paddles may be placed on the front of the chest, which is the standard paddle placement. Whether using pads, or paddles, the nurse must observe two safety measures. First, maintain good contact between the pads or paddles and the patient's skin to prevent leaking. Second, ensure that no one is in contact with the patient or with anything that is touching the patient when the defibrillator is discharged, to minimize the chance that electrical current will be conducted to anyone other than the patient. Ventilation should be stopped during defibrillation.

During a CPR class, a participant asks about the difference between cardioversion and defibrillation. What would be the instructor's best response? A) "Cardioversion is done on a beating heart; defibrillation is not." B) "The difference is the timing of the delivery of the electric current." C) "Defibrillation is synchronized with the electrical activity of the heart, but cardioversion is not." D) "Cardioversion is always attempted before defibrillation because it has fewer risks."

Ans: B Feedback: One major difference between cardioversion and defibrillation is the timing of the delivery of electrical current. In cardioversion, the delivery of the electrical current is synchronized with the patient's electrical events; in defibrillation, the delivery of the current is immediate and unsynchronized. Both can be done on beating heart (i.e., in a dysrhythmia). Cardioversion is not necessarily attempted first.

The nurse is analyzing a rhythm strip. What component of the ECG corresponds to the resting state of the patient's heart? A) P wave B) T wave C) U wave D) QRS complex

Ans: B Feedback: The T wave specifically represents ventricular muscle depolarization, also referred to as the resting state. Ventricular muscle depolarization does not result in the P wave, U wave, or QRS complex.

New nurses on the telemetry unit have been paired with preceptors. One new nurse asks her preceptor to explain depolarization. What would be the best answer by the preceptor? A) "Depolarization is the mechanical contraction of the heart muscles." B) "Depolarization is the electrical stimulation of the heart muscles." C) "Depolarization is the electrical relaxation of the heart muscles." D) "Depolarization is the mechanical relaxation of the heart muscles."

Ans: B Feedback: The electrical stimulation of the heart is called depolarization, and the mechanical contraction is called systole. Electrical relaxation is called repolarization, and mechanical relaxation is called diastole.

The nurse is caring for a patient with refractory atrial fibrillation who underwent the maze procedure several months ago. The nurse reviews the result of the patient's most recent cardiac imaging, which notes the presence of scarring on the atria. How should the nurse best respond to this finding? A) Recognize that the procedure was unsuccessful. B) Recognize this as a therapeutic goal of the procedure. C) Liaise with the care team in preparation for repeating the maze procedure. D) Prepare the patient for pacemaker implantation.

Ans: B Feedback: The maze procedure is an open heart surgical procedure for refractory atrial fibrillation. Small transmural incisions are made throughout the atria. The resulting formation of scar tissue prevents reentry conduction of the electrical impulse. Consequently, scar formation would constitute a successful procedure. There is no indication for repeating the procedure or implanting a pacemaker.

A patient converts from normal sinus rhythm at 80 bpm to atrial fibrillation with a ventricular response at 166 bpm. Blood pressure is 162/74 mm Hg. Respiratory rate is 20 breaths per minute with normal chest expansion and clear lungs bilaterally. IV heparin and Cardizem are given. The nurse caring for the patient understands that the main goal of treatment is what? A) Decrease SA node conduction B) Control ventricular heart rate C) Improve oxygenation D) Maintain anticoagulation

Ans: B Feedback: Treatment for atrial fibrillation is to terminate the rhythm or to control ventricular rate. This is a priority because it directly affects cardiac output. A rapid ventricular response reduces the time for ventricular filling, resulting in a smaller stroke volume. Control of rhythm is the initial treatment of choice, followed by anticoagulation with heparin and then Coumadin.

The staff educator is teaching a CPR class. Which of the following aspects of defibrillation should the educator stress to the class? A) Apply the paddles directly to the patient's skin. B) Use a conducting medium between the paddles and the skin. C) Always use a petroleum-based gel between the paddles and the skin. D) Any available liquid can be used between the paddles and the skin.

Ans: B Feedback: Use multifunction conductor pads or paddles with a conducting medium between the paddles and the skin (the conducting medium is available as a sheet, gel, or paste). Do not use gels or pastes with poor electrical conductivity.

The nurse is addressing exercise and physical activity during discharge education with a patient diagnosed with HF. What should the nurse teach this patient about exercise? A) "Do not exercise unsupervised." B) "Eventually aim to work up to 30 minutes of exercise each day." C) "Slow down if you get dizzy or short of breath." D) "Start your exercise program with high-impact activities."

Ans: B) "Eventually aim to work up to 30 minutes of exercise each day." Feedback: Eventually, a total of 30 minutes of physical activity every day should be encouraged. Supervision is not necessarily required and the emergence of symptoms should prompt the patient to stop exercising, not simply to slow the pace. Low-impact activities should be prioritized.

A patient with HF is placed on a low-sodium diet. Which statement by the patient indicates that the nurse's nutritional teaching plan has been effective? A) "I will have a ham and cheese sandwich for lunch." B) "I will have a baked potato with broiled chicken for dinner." C) "I will have a tossed salad with cheese and croutons for lunch." D) "I will have chicken noodle soup with crackers and an apple for lunch."

Ans: B) "I will have a baked potato with broiled chicken for dinner." Feedback: The patient's choice of a baked potato with broiled chicken indicates that the teaching plan has been effective. Potatoes and chicken are relatively low in sodium. Ham, cheese, and soup are often high in sodium.

A patient presents to the ED complaining of increasing shortness of breath. The nurse assessing the patient notes a history of left-sided HF. The patient is agitated and occasionally coughing up pink-tinged, foamy sputum. The nurse should recognize the signs and symptoms of what health problem? A) Right-sided heart failure B) Acute pulmonary edema C) Pneumonia D) Cardiogenic shock

Ans: B) Acute pulmonary edema Feedback: Because of decreased contractility and increased fluid volume and pressure in patients with HF, fluid may be driven from the pulmonary capillary beds into the alveoli, causing pulmonary edema and signs and symptoms described. In right-sided heart failure, the patient exhibits hepatomegaly, jugular vein distention, and peripheral edema. In pneumonia, the patient would have a temperature spike, and sputum that varies in color. Cardiogenic shock would show signs of hypotension and tachycardia.

The nurse is assessing a patient who is known to have right-sided HF. What assessment finding is most consistent with this patient's diagnosis? A) Pulmonary edema B) Distended neck veins C) Dry cough D) Orthopnea

Ans: B) Distended neck veins Feedback: Right-sided HF may manifest by distended neck veins, dependent edema, hepatomegaly, weight gain, ascites, anorexia, nausea, nocturia, and weakness. The other answers do not apply.

Which assessment would be most appropriate for a patient who is receiving a loop diuretic for HF? A) Monitor liver function studies B) Monitor for hypotension C) Assess the patient's vitamin D intake D) Assess the patient for hyperkalemia

Ans: B) Monitor for hypotension Feedback: Diuretic therapy increases urine output and decreases blood volume, which places the patient at risk of hypotension. Patients are at risk of losing potassium with loop diuretic therapy and need to continue with potassium in their diet; hypokalemia is a consequent risk. Liver function is rarely compromised by diuretic therapy and vitamin D intake is not relevant.

The nurse is caring for a 68-year-old patient the nurse suspects has digoxin toxicity. In addition to physical assessment, the nurse should collect what assessment datum? A) Skin turgor B) Potassium level C) White blood cell count D) Peripheral pulses

Ans: B) Potassium level Feedback: The serum potassium level is monitored because the effect of digoxin is enhanced in the presence of hypokalemia and digoxin toxicity may occur. Skin turgor, white cell levels, and peripheral pulses are not normally affected in cases of digitalis toxicity.

The nurse's comprehensive assessment of a patient who has HF includes evaluation of the patient's hepatojugular reflux. What action should the nurse perform during this assessment? A) Elevate the patient's head to 90 degrees. B) Press the right upper abdomen. C) Press above the patient's symphysis pubis. D) Lay the patient flat in bed.

Ans: B) Press the right upper abdomen. Feedback: Hepatojugular reflux, a sign of right-sided heart failure, is assessed with the head of the bed at a 45-degree angle. As the right upper abdomen (the area over the liver) is compressed for 30 to 40 seconds, the nurse observes the internal jugular vein. If the internal jugular vein becomes distended, a patient has positive hepatojugular reflux.

The nurse is educating an 80-year-old patient diagnosed with HF about his medication regimen. What should the nurse to teach this patient about the use of oral diuretics? A) Avoid drinking fluids for 2 hours after taking the diuretic. B) Take the diuretic in the morning to avoid interfering with sleep. C) Avoid taking the medication within 2 hours consuming dairy products. D) Take the diuretic only on days when experiencing shortness of breath.

Ans: B) Take the diuretic in the morning to avoid interfering with sleep. Feedback: Oral diuretics should be administered early in the morning so that diuresis does not interfere with the patient's nighttime rest. Discussing the timing of medication administration is especially important for elderly patients who may have urinary urgency or incontinence. The nurse would not teach the patient about the timing of fluid intake. Fluid intake does not need to be adjusted and dairy products are not contraindicated.

The nurse overseeing care in the ICU reviews the shift report on four patients. The nurse recognizes which patient to be at greatest risk for the development of cardiogenic shock? A) The patient admitted with acute renal failure B) The patient admitted following an MI C) The patient admitted with malignant hypertension D) The patient admitted following a stroke

Ans: B) The patient admitted following an MI Feedback: Cardiogenic shock may occur following an MI when a large area of the myocardium becomes ischemic, necrotic, and hypokinetic. It also can occur as a result of end-stage heart failure, cardiac tamponade, pulmonary embolism, cardiomyopathy, and dysrhythmias. While patients with acute renal failure are at risk for dysrhythmias and patients experiencing a stroke are at risk for thrombus formation, the patient admitted following an MI is at the greatest risk for development of cardiogenic shock when compared with the other listed diagnoses.

36. The nurse is reviewing the medication administration record of a patient who takes a variety of medications for the treatment of hypertension. What potential therapeutic benefits of antihypertensives should the nurse identify? Select all that apply. A) Increased venous return B) Decreased peripheral resistance C) Decreased blood volume D) Decreased strength and rate of myocardial contractions E) Decreased blood viscosity

Ans: B, C, D Feedback: The medications used for treating hypertension decrease peripheral resistance, blood volume, or the strength and rate of myocardial contraction. Antihypertensive medications do not increase venous return or decrease blood viscosity.

17. The nurse is assessing a patient new to the clinic. Records brought to the clinic with the patient show the patient has hypertension and that her current BP readings approximate the readings from when she was first diagnosed. What contributing factor should the nurse first explore in an effort to identify the cause of the client's inadequate BP control? A) Progressive target organ damage B) Possibility of medication interactions C) Lack of adherence to prescribed drug therapy D) Possible heavy alcohol use or use of recreational drugs

Ans: C Feedback: Deviation from the therapeutic program is a significant problem for people with hypertension and other chronic conditions requiring lifetime management. An estimated 50% of patients discontinue their medications within 1 year of beginning to take them. Consequently, this is a more likely problem than substance use, organ damage, or adverse drug interactions.

1. An older adult is newly diagnosed with primary hypertension and has just been started on a beta-blocker. The nurse's health education should include which of the following? A) Increasing fluids to avoid extracellular volume depletion from the diuretic effect of the beta-blocker B) Maintaining a diet high in dairy to increase protein necessary to prevent organ damage C) Use of strategies to prevent falls stemming from postural hypotension D) Limiting exercise to avoid injury that can be caused by increased intracranial pressure

Ans: C Feedback: Elderly people have impaired cardiovascular reflexes and are more sensitive to postural hypotension. The nurse teaches patients to change positions slowly when moving from lying or sitting positions to a standing position, and counsels elderly patients to use supportive devices as necessary to prevent falls that could result from dizziness. Lifestyle changes, such as regular physical activity/exercise, and a diet rich in fruits, vegetables, and low-fat dairy products, is strongly recommended. Increasing fluids in elderly patients may be contraindicated due to cardiovascular disease. Increased intracranial pressure is not a risk and activity should not normally be limited.

19. The nurse is providing care for a patient with a diagnosis of hypertension. The nurse should consequently assess the patient for signs and symptoms of which other health problem? A) Migraines B) Atrial-septal defect C) Atherosclerosis D) Thrombocytopenia

Ans: C Feedback: Hypertension is both a sign and a risk factor for atherosclerotic heart disease. It is not associated with structural cardiac defects, low platelet levels, or migraines.

5. A group of student nurses are practicing taking blood pressure. A 56-year-old male student has a blood pressure reading of 146/96 mm Hg. Upon hearing the reading, he exclaims, "My pressure has never been this high. Do you think my doctor will prescribe medication to reduce it?" Which of the following responses by the nursing instructor would be best? A) "Yes. Hypertension is prevalent among men; it is fortunate we caught this during your routine examination." B) "We will need to reevaluate your blood pressure because your age places you at high risk for hypertension." C) "A single elevated blood pressure does not confirm hypertension. You will need to have your blood pressure reassessed several times before a diagnosis can be made." D) "You have no need to worry. Your pressure is probably elevated because you are being tested."

Ans: C Feedback: Hypertension is confirmed by two or more readings with systolic pressure of at least 140 mm Hg and diastolic pressure of at least 90 mm Hg. An age of 56 does not constitute a risk factor in and of itself. The nurse should not tell the student that there is no need to worry.

12. A patient in a hypertensive emergency is admitted to the ICU. The nurse anticipates that the patient will be treated with IV vasodilators, and that the primary goal of treatment is what? A) Lower the BP to reduce onset of neurologic symptoms, such as headache and vision changes. B) Decrease the BP to a normal level based on the patient's age. C) Decrease the mean arterial pressure between 20% and 25% in the first hour of treatment. D) Reduce the BP to 120/75 mm Hg as quickly as possible.

Ans: C Feedback: Initially, the treatment goal in hypertensive emergencies is to reduce the mean arterial pressure by 25% in the first hour of treatment, with further reduction over the next 24 hours. Lowering the BP too fast may cause hypotension in a patient whose body has adjusted to hypertension and could cause a stroke, MI, or visual changes. Neurologic symptoms should be addressed, but this is not the primary focus of treatment planning.

24. The nurse is collaborating with the dietitian and a patient with hypertension to plan dietary modifications. These modifications should include which of the following? A) Reduced intake of protein and carbohydrates B) Increased intake of calcium and vitamin D C) Reduced intake of fat and sodium D) Increased intake of potassium, vitamin B12 and vitamin D

Ans: C Feedback: Lifestyle modifications usually include restricting sodium and fat intake, increasing intake of fruits and vegetables, and implementing regular physical activity. There is no need to increase calcium, potassium, and vitamin intake. Calorie restriction may be required for some patients, but a specific reduction in protein and carbohydrates is not normally indicated.

34. A student nurse is taking care of an elderly patient with hypertension during a clinical experience. The instructor asks the student about the relationships between BP and age. What would be the best answer by the student? A) "Because of reduced smooth muscle tone in blood vessels, blood pressure tends to go down with age, not up." B) "Decreases in the strength of arteries and the presence of venous insufficiency cause hypertension in the elderly." C) "Structural and functional changes in the cardiovascular system that occur with age contribute to increases in blood pressure." D) "The neurologic system of older adults is less efficient at monitoring and regulating blood pressure."

Ans: C Feedback: Structural and functional changes in the heart and blood vessels contribute to increases in BP that occur with aging. Venous insufficiency does not cause hypertension, however. Increased BP is not primarily a result of neurologic changes.

25. The critical care nurse is caring for a patient just admitted in a hypertensive emergency. The nurse should anticipate the administration of what medication? A) Warfarin (Coumadin) B) Furosemide (Lasix) C) Sodium nitroprusside (Nitropress) D) Ramipril (Altace)

Ans: C Feedback: The medications of choice in hypertensive emergencies are those that have an immediate effect. IV vasodilators, including sodium nitroprusside (Nitropress), nicardipine hydrochloride (Cardene), clevidipine (Cleviprex), fenoldopam mesylate (Corlopam), enalaprilat, and nitroglycerin, have immediate actions that are short lived (minutes to 4 hours), and they are therefore used for initial treatment. Ramipril is administered orally and would not meet the patient's immediate need for BP management. Diuretics, such as Lasix, are not used as initial treatments and there is no indication for anticoagulants such as Coumadin.

37. A newly diagnosed patient with hypertension is prescribed Diuril, a thiazide diuretic. What patient education should the nurse provide to this patient? A) "Eat a banana every day because Diuril causes moderate hyperkalemia." B) "Take over-the-counter potassium pills because Diuril causes your kidneys to lose potassium." C) "Diuril can cause low blood pressure and dizziness, especially when you get up suddenly." D) "Diuril increases sodium levels in your blood, so cut down on your salt."

Ans: C Feedback: Thiazide diuretics can cause postural hypotension, which may be potentiated by alcohol, barbiturates, opioids, or hot weather. Diuril does not cause either moderate hyperkalemia or severe hypokalemia and it does not result in hypernatremia.

3. A nurse is performing blood pressure screenings at a local health fair. While obtaining subjective assessment data from a patient with hypertension, the nurse learns that the patient has a family history of hypertension and she herself has high cholesterol and lipid levels. The patient says she smokes one pack of cigarettes daily and drinks "about a pack of beer" every day. The nurse notes what nonmodifiable risk factor for hypertension? A) Hyperlipidemia B) Excessive alcohol intake C) A family history of hypertension D) Closer adherence to medical regimen

Ans: C Feedback: Unlike cholesterol levels, alcohol intake and adherence to treatment, family history is not modifiable.

A patient is admitted to the cardiac care unit for an electrophysiology (EP) study. What goal should guide the planning and execution of the patient's care? A) Ablate the area causing the dysrhythmia. B) Freeze hypersensitive cells. C) Diagnose the dysrhythmia. D) Determine the nursing plan of care.

Ans: C Feedback: A patient may undergo an EP study in which electrodes are placed inside the heart to obtain an intracardiac ECG. This is used not only to diagnose the dysrhythmia but also to determine the most effective treatment plan. However, because an EP study is invasive, it is performed in the hospital and may require that the patient be admitted.

The nurse is caring for a patient who has just had an implantable cardioverter defibrillator (ICD) placed. What is the priority area for the nurse's assessment? A) Assessing the patient's activity level B) Facilitating transthoracic echocardiography C) Vigilant monitoring of the patient's ECG D) Close monitoring of the patient's peripheral perfusion

Ans: C Feedback: After a permanent electronic device (pacemaker or ICD) is inserted, the patient's heart rate and rhythm are monitored by ECG. This is a priority over peripheral circulation and activity. Echocardiography is not indicated.

During a patient's care conference, the team is discussing whether the patient is a candidate for cardiac conduction surgery. What would be the most important criterion for a patient to have this surgery? A) Angina pectoris not responsive to other treatments B) Decreased activity tolerance related to decreased cardiac output C) Atrial and ventricular tachycardias not responsive to other treatments D) Ventricular fibrillation not responsive to other treatments

Ans: C Feedback: Cardiac conduction surgery is considered in patients who do not respond to medications and antitachycardia pacing. Angina, reduced activity tolerance, and ventricular fibrillation are not criteria.

The nurse is writing a plan of care for a patient with a cardiac dysrhythmia. What would be the most appropriate goal for the patient? A) Maintain a resting heart rate below 70 bpm. B) Maintain adequate control of chest pain. C) Maintain adequate cardiac output. D) Maintain normal cardiac structure.

Ans: C Feedback: For patient safety, the most appropriate goal is to maintain cardiac output to prevent worsening complications as a result of decreased cardiac output. A resting rate of less than 70 bpm is not appropriate for every patient. Chest pain is more closely associated with acute coronary syndrome than with dysrhythmias. Nursing actions cannot normally influence the physical structure of the heart.

The nurse is planning discharge teaching for a patient with a newly inserted permanent pacemaker. What is the priority teaching point for this patient? A) Start lifting the arm above the shoulder right away to prevent chest wall adhesion. B) Avoid cooking with a microwave oven. C) Avoid exposure to high-voltage electrical generators. D) Avoid walking through store and library antitheft devices.

Ans: C Feedback: High-output electrical generators can reprogram pacemakers and should be avoided. Recent pacemaker technology allows patients to safely use most household electronic appliances and devices (e.g., microwave ovens). The affected arm should not be raised above the shoulder for 1 week following placement of the pacemaker. Antitheft alarms may be triggered so patients should be taught to walk through them quickly and avoid standing in or near these devices. These alarms generally do not interfere with pacemaker function.

An adult patient with third-degree AV block is admitted to the cardiac care unit and placed on continuous cardiac monitoring. What rhythm characteristic will the ECG most likely show? A) PP interval and RR interval are irregular. B) PP interval is equal to RR interval. C) Fewer QRS complexes than P waves D) PR interval is constant.

Ans: C Feedback: In third-degree AV block, no atrial impulse is conducted through the AV node into the ventricles. As a result, there are impulses stimulating the atria and impulses stimulating the ventricles. Therefore, there are more P waves than QRS complexes due to the difference in the natural pacemaker (nodes) rates of the heart. The other listed ECG changes are not consistent with this diagnosis.

A patient has undergone diagnostic testing and received a diagnosis of sinus bradycardia attributable to sinus node dysfunction. When planning this patient's care, what nursing diagnosis is most appropriate? A) Acute pain B) Risk for unilateral neglect C) Risk for activity intolerance D) Risk for fluid volume excess

Ans: C Feedback: Sinus bradycardia causes decreased cardiac output that is likely to cause activity intolerance. It does not typically cause pain, fluid imbalances, or neglect of a unilateral nature.

The nursing educator is presenting a case study of an adult patient who has abnormal ventricular depolarization. This pathologic change would be most evident in what component of the ECG? A) P wave B) T wave C) QRS complex D) U wave

Ans: C Feedback: The QRS complex represents the depolarization of the ventricles and, as such, the electrical activity of that ventricle.

The nurse is caring for a patient who has had a dysrhythmic event. The nurse is aware of the need to assess for signs of diminished cardiac output (CO). What change in status may signal to the nurse a decrease in cardiac output? A) Increased blood pressure B) Bounding peripheral pulses C) Changes in level of consciousness D) Skin flushing

Ans: C Feedback: The nurse conducts a physical assessment to confirm the data obtained from the history and to observe for signs of diminished cardiac output (CO) during the dysrhythmic event, especially changes in level of consciousness. Blood pressure tends to decrease with lowered CO and bounding peripheral pulses are inconsistent with this problem. Pallor, not skin flushing, is expected.

The triage nurse in the ED is assessing a patient with chronic HF who has presented with worsening symptoms. In reviewing the patient's medical history, what is a potential primary cause of the patient's heart failure? A) Endocarditis B) Pleural effusion C) Atherosclerosis D) Atrial-septal defect

Ans: C) Atherosclerosis Feedback: Atherosclerosis of the coronary arteries is the primary cause of HF. Pleural effusion, endocarditis, and an atrial-septal defect are not health problems that contribute to the etiology of HF.

The nurse is caring for an 84-year-old man who has just returned from the OR after inguinal hernia repair. The OR report indicates that the patient received large volumes of IV fluids during surgery and the nurse recognizes that the patient is at risk for left-sided heart failure. What signs and symptoms would indicate left-sided heart failure? A) Jugular vein distention B) Right upper quadrant pain C) Bibasilar fine crackles D) Dependent edema

Ans: C) Bibasilar fine crackles Feedback: Bibasilar fine crackles are a sign of alveolar fluid, a sequela of left ventricular fluid, or pressure overload. Jugular vein distention, right upper quadrant pain (hepatomegaly), and dependent edema are caused by right-sided heart failure, usually a chronic condition.

Diagnostic imaging reveals that the quantity of fluid in a client's pericardial sac is dangerously increased. The nurse should collaborate with the other members of the care team to prevent the development of what complication? A) Pulmonary edema B) Pericardiocentesis C) Cardiac tamponade D) Pericarditis

Ans: C) Cardiac tamponade Feedback: An increase in pericardial fluid raises the pressure within the pericardial sac and compresses the heart, eventually causing cardiac tamponade. Pericardiocentesis is the treatment for this complication. Pericarditis and pulmonary edema do not result from this pathophysiological process.

The nurse is caring for a patient with systolic HF whose previous adverse reactions preclude the safe use of ACE inhibitors. The nurse should anticipate that the prescriber may choose what combination of drugs? A) Loop diuretic and antiplatelet aggregator B) Loop diuretic and calcium channel blocker C) Combination of hydralazine and isosorbide dinitrate D) Combination of digoxin and normal saline

Ans: C) Combination of hydralazine and isosorbide dinitrate Feedback: A combination of hydralazine and isosorbide dinitrate may be an alternative for patients who cannot take ACE inhibitors. Antiplatelet aggregators, calcium channel blockers, and normal saline are not typically prescribed.

The nurse is providing discharge education to a patient diagnosed with HF. What should the nurse teach this patient to do to assess her fluid balance in the home setting? A) Monitor her blood pressure daily B) Assess her radial pulses daily C) Monitor her weight daily D) Monitor her bowel movements

Ans: C) Monitor her weight daily Feedback: To assess fluid balance at home, the patient should monitor daily weights at the same time every day. Assessing radial pulses and monitoring the blood pressure may be done, but these measurements do not provide information about fluid balance. Bowel function is not indicative of fluid balance.

The nurse notes that a patient has developed a cough productive for mucoid sputum, is short of breath, has cyanotic hands, and has noisy, moist-sounding, rapid breathing. These symptoms and signs are suggestive of what health problem? A) Pericarditis B) Cardiomyopathy C) Pulmonary edema D) Right ventricular hypertrophy

Ans: C) Pulmonary edema Feedback: As a result of decreased cerebral oxygenation, the patient with pulmonary edema becomes increasingly restless and anxious. Along with a sudden onset of breathlessness and a sense of suffocation, the patient's hands become cold and moist, the nail beds become cyanotic (bluish), and the skin turns ashen (gray). The pulse is weak and rapid, and the neck veins are distended. Incessant coughing may occur, producing increasing quantities of foamy sputum. Pericarditis, ventricular hypertrophy, and cardiomyopathy do not involve wet breath sounds or mucus production.

The nurse is caring for a patient who has developed obvious signs of pulmonary edema. What is the priority nursing action? A) Lay the patient flat. B) Notify the family of the patient's critical state. C) Stay with the patient. D) Update the physician.

Ans: C) Stay with the patient. Feedback: Because the patient has an unstable condition, the nurse must remain with the patient. The physician must be updated promptly, but the patient should not be left alone in order for this to happen. Supine positioning is unlikely to relieve dyspnea. The family should be informed, but this is not the priority action.

31. A community health nurse teaching a group of adults about preventing and treating hypertension. The nurse should encourage these participants to collaborate with their primary care providers and regularly monitor which of the following? A) Heart rate B) Sodium levels C) Potassium levels D) Blood lipid levels

Ans: D Feedback: Hypertension often accompanies other risk factors for atherosclerotic heart disease, such as dyslipidemia (abnormal blood fat levels), obesity, diabetes, metabolic syndrome, and a sedentary lifestyle. Individuals with hypertension need to monitor their sodium intake, but hypernatremia is not a risk factor for hypertension. In many patients, heart rate does not correlate closely with BP. Potassium levels do not normally relate to BP.

35. A 55-year-old patient comes to the clinic for a routine check-up. The patient's BP is 159/100 mm Hg and the physician diagnoses hypertension after referring to previous readings. The patient asks why it is important to treat hypertension. What would be the nurse's best response? A) "Hypertension can cause you to develop dangerous blood clots in your legs that can migrate to your lungs." B) "Hypertension puts you at increased risk of type 1 diabetes and cancer in your age group." C) "Hypertension is the leading cause of death in people your age." D) "Hypertension greatly increases your risk of stroke and heart disease."

Ans: D Feedback: Hypertension, particularly elevated systolic BP, increases the risk of death, stroke, and heart failure in people older than 50 years. Hypertension is not a direct precursor to pulmonary emboli, and it does not put older adults at increased risk of type 1 diabetes or cancer. It is not the leading cause of death in people 55 years of age.

4. The staff educator is teaching ED nurses about hypertensive crisis. The nurse educator should explain that hypertensive urgency differs from hypertensive emergency in what way? A) The BP is always higher in a hypertensive emergency. B) Vigilant hemodynamic monitoring is required during treatment of hypertensive emergencies. C) Hypertensive urgency is treated with rest and benzodiazepines to lower BP. D) Hypertensive emergencies are associated with evidence of target organ damage.

Ans: D Feedback: Hypertensive emergencies are acute, life-threatening BP elevations that require prompt treatment in an intensive care setting because of the serious target organ damage that may occur. Blood pressures are extremely elevated in both urgency and emergencies, but there is no evidence of target organ damage in hypertensive urgency. Extremely close hemodynamic monitoring of the patient's BP is required in both situations. The medications of choice in hypertensive emergencies are those with an immediate effect, such as IV vasodilators. Oral doses of fast-acting agents, such as beta-adrenergic blocking agents, angiotensin-converting enzyme inhibitors, or alpha-agonists, are recommended for the treatment of hypertensive urgencies.

26. A patient in hypertensive emergency is being cared for in the ICU. The patient has become hypovolemic secondary to natriuresis. What is the nurse's most appropriate action? A) Add sodium to the patient's IV fluid, as ordered. B) Administer a vasoconstrictor, as ordered. C) Promptly cease antihypertensive therapy. D) Administer normal saline IV, as ordered.

Ans: D Feedback: If there is volume depletion secondary to natriuresis caused by the elevated BP, then volume replacement with normal saline can prevent large, sudden drops in BP when antihypertensive medications are administered. Sodium administration, cessation of antihypertensive therapy, and administration of vasoconstrictors are not normally indicated.

38. A patient in hypertensive urgency is admitted to the hospital. The nurse should be aware of what goal of treatment for a patient in hypertensive urgency? A) Normalizing BP within 2 hours B) Obtaining a BP of less than 110/70 mm Hg within 36 hours C) Obtaining a BP of less than 120/80 mm Hg within 36 hours D) Normalizing BP within 24 to 48 hours

Ans: D Feedback: In cases of hypertensive urgency, oral agents can be administered with the goal of normalizing BP within 24 to 48 hours. For patients with this health problem, a BP of 120/80 mm Hg may be unrealistic.

30. The nurse is screening a number of adults for hypertension. What range of blood pressure is considered normal? A) Less than 140/90 mm Hg B) Less than 130/90 mm Hg C) Less than 129/89 mm Hg D) Less than 120/80 mm Hg

Ans: D Feedback: JNC 7 defines a blood pressure of less than 120/80 mm Hg as normal, 120 to 129/80 to 89 mm Hg as prehypertension, and 140/90 mm Hg or higher as hypertension.

28. A patient comes to the walk-in clinic complaining of frequent headaches. While assessing the patient's vital signs, the nurse notes the BP is 161/101 mm Hg. According to JNC 7, how would this patient's BP be defined if a similar reading were obtained at a subsequent office visit? A) High normal B) Normal C) Stage 1 hypertensive D) Stage 2 hypertensive

Ans: D Feedback: JNC 7 defines stage 2 hypertension as a reading 160/100 mm Hg.

40. A patient's recently elevated BP has prompted the primary care provider to prescribe furosemide (Lasix). The nurse should closely monitor which of the following? A) The client's oxygen saturation level B) The patient's red blood cells, hematocrit, and hemoglobin C) The patient's level of consciousness D) The patient's potassium level

Ans: D Feedback: Loop diuretics can cause potassium depletion. They do not normally affect level of consciousness, erythrocytes, or oxygen saturation.

32. A community health nurse is planning an educational campaign addressing hypertension. The nurse should anticipate that the incidence and prevalence of hypertension are likely to be highest among members of what ethnic group? A) Pacific Islanders B) African Americans C) Asian-Americans D) Hispanics

Ans: D Feedback: The prevalence of uncontrolled hypertension varies by ethnicity, with Hispanics and African Americans having the highest prevalence at approximately 63% and 57%, respectively.

33. The home health nurse is caring for a patient who has a comorbidity of hypertension. What assessment question most directly addresses the possibility of worsening hypertension? A) "Are you eating less salt in your diet?" B) "How is your energy level these days?" C) "Do you ever get chest pain when you exercise?" D) "Do you ever see spots in front of your eyes?"

Ans: D Feedback: To identify complications or worsening hypertension, the patient is questioned about blurred vision, spots in front of the eyes, and diminished visual acuity. The heart, nervous system, and kidneys are also carefully assessed, but angina pain and decreased energy are not normally suggestive of worsening hypertension. Sodium limitation is a beneficial lifestyle modification, but nonadherence to this is not necessarily a sign of worsening symptoms.

21. A patient with newly diagnosed hypertension has come to the clinic for a follow-up visit. The patient asks the nurse why she has to come in so often. What would be the nurse's best response? A) "We do this so you don't suffer a stroke." B) "We do this to determine how your blood pressure changes throughout the day." C) "We do this to see how often you should change your medication dose." D) "We do this to make sure your health is stable. We'll then monitor it at routinely scheduled intervals."

Ans: D Feedback: When hypertension is initially detected, nursing assessment involves carefully monitoring the BP at frequent intervals and then at routinely scheduled intervals. The reference to stroke is frightening and does not capture the overall rationale for the monitoring regimen. Changes throughout the day are not a clinical priority for most patients. The patient must not change his or her medication doses unilaterally.

An ECG has been ordered for a newly admitted patient. What should the nurse do prior to electrode placement? A) Clean the skin with providone-iodine solution. B) Ensure that the area for electrode placement is dry. C) Apply tincture of benzoin to the electrode sites and wait for it to become ìtacky.î D) Gently abrade the skin by rubbing the electrode sites with dry gauze or cloth.

Ans: D Feedback: An ECG is obtained by slightly abrading the skin with a clean dry gauze pad and placing electrodes on the body at specific areas. The abrading of skin will enhance signal transmission. Disinfecting the skin is unnecessary and conduction gel is used.

A nurse is providing health education to a patient scheduled for cryoablation therapy. The nurse should describe what aspect of this treatment? A) Peeling away the area of endocardium responsible for the dysrhythmia B) Using electrical shocks directly to the endocarduim to eliminate the source of dysrhythmia C) Using high-frequency sound waves to eliminate the source of dysrhythmia D) Using a cooled probe to eliminate the source of dysrhythmia

Ans: D Feedback: Cryoablation therapy involves using a cooled probe to create a small scar on the endocardium to eliminate the source of the dysrhythmias. Endocardium resection involves peeling away a specified area of the endocardium. Electrical ablation involves using shocks to eliminate the area causing the dysrhythmias. Radio frequency ablation uses high-frequency sound waves to destroy the area causing the dysrhythmias.

The nurse is assessing a patient who had a pacemaker implanted 4 weeks ago. During the patient's most recent follow-up appointment, the nurse identifies data that suggest the patient may be socially isolated and depressed. What nursing diagnosis is suggested by these data? A) Decisional conflict related to pacemaker implantation B) Deficient knowledge related to pacemaker implantation C) Spiritual distress related to pacemaker implantation D) Ineffective coping related to pacemaker implantation

Ans: D Feedback: Depression and isolation may be symptoms of ineffective coping with the implantation. These psychosocial symptoms are not necessarily indicative of issues related to knowledge or decisions. Further data would be needed to determine a spiritual component to the patient's challenges.

The nurse is caring for a patient on telemetry. The patient's ECG shows a shortened PR interval, slurring of the initial QRS deflection, and prolonged QRS duration. What does this ECG show? A) Sinus bradycardia B) Myocardial infarction C) Lupus-like syndrome D) Wolf-Parkinson-White (WPW) syndrome

Ans: D Feedback: In WPW syndrome there is a shortened PR interval, slurring (called a delta wave) of the initial QRS deflection, and prolonged QRS duration. These characteristics are not typical of the other listed cardiac anomalies.

A patient calls his cardiologist's office and talks to the nurse. He is concerned because he feels he is being defibrillated too often. The nurse tells the patient to come to the office to be evaluated because the nurse knows that the most frequent complication of ICD therapy is what? A) Infection B) Failure to capture C) Premature battery depletion D) Oversensing of dysrhythmias

Ans: D Feedback: Inappropriate delivery of ICD therapy, usually due to oversensing of atrial and sinus tachycardias with a rapid ventricular rate response, is the most frequent complication of ICD. Infections, failure to capture, and premature battery failure are less common.

The nurse is providing care to a patient who has just undergone an electrophysiologic (EP) study. The patient states that she is nervous about ìthings going wrongî during the procedure. What is the nurse's best response? A) "This is basically a risk-free procedure." B) "Thousands of patients undergo EP every year." C) "Remember that this is a step that will bring you closer to enjoying good health." D) "The whole team will be monitoring you very closely for the entire procedure."

Ans: D Feedback: Patients who are to undergo an EP study may be anxious about the procedure and its outcome. A detailed discussion involving the patient, the family, and the electrophysiologist usually occurs to ensure that the patient can give informed consent and to reduce the patient's anxiety about the procedure. It is inaccurate to state that EP is ìrisk-freeî and stating that it is common does not necessarily relieve the patient's anxiety. Characterizing EP as a step toward good health does not directly address the patient's anxiety.

A group of nurses are participating in orientation to a telemetry unit. What should the staff educator tell this class about ST segments? A) They are the part of an ECG that reflects systole. B) They are the part of an ECG used to calculate ventricular rate and rhythm. C) They are the part of an ECG that reflects the time from ventricular depolarization through repolarization. D) They are the part of an ECG that represents early ventricular repolarization.

Ans: D Feedback: ST segment is the part of an ECG that reflects the end of the QRS complex to the beginning of the T wave. The part of an ECG that reflects repolarization of the ventricles is the T wave. The part of an ECG used to calculate ventricular rate and rhythm is the RR interval. The part of an ECG that reflects the time from ventricular depolarization through repolarization is the QT interval.

A cardiac care nurse is aware of factors that result in positive chronotropy. These factors would affect a patient's cardiac function in what way? A) Exacerbating an existing dysrhythmia B) Initiating a new dysrhythmia C) Resolving ventricular tachycardia D) Increasing the heart rate

Ans: D Feedback: Stimulation of the sympathetic system increases heart rate. This phenomenon is known as positive chronotropy. It does not influence dysrhythmias.

Following cardiac resuscitation, a patient has been placed in a state of mild hypothermia before being transferred to the cardiac intensive care unit. The nurse's assessment reveals that the patient is experiencing neuromuscular paralysis. How should the nurse best respond? A) Administer hypertonic IV solution. B) Administer a bolus of warned normal saline. C) Reassess the patient in 15 minutes. D) Document this as an expected assessment finding.

Ans: D Feedback: The nurse caring for a patient with hypothermia (passive or induced) needs to monitor for appropriate level of cooling, sedation, and neuromuscular paralysis to prevent seizures; myoclonus; and shivering. Neuromuscular paralysis is an expected finding and does not necessitate further interventions.

When assessing the patient with pericardial effusion, the nurse will assess for pulsus paradoxus. Pulsus paradoxus is characterized by what assessment finding? A) A diastolic blood pressure that is lower during exhalation B) A diastolic blood pressure that is higher during inhalation C) A systolic blood pressure that is higher during exhalation D) A systolic blood pressure that is lower during inhalation

Ans: D) A systolic blood pressure that is lower during inhalation Feedback: Systolic blood pressure that is markedly lower during inhalation is called pulsus paradoxus. The difference in systolic pressure between the point that is heard during exhalation and the point that is heard during inhalation is measured. Pulsus paradoxus exceeding 10 mm Hg is abnormal.

The nurse has entered a patient's room and found the patient unresponsive and not breathing. What is the nurse's next appropriate action? A) Palpate the patient's carotid pulse. B) Illuminate the patient's call light. C) Begin performing chest compressions. D) Activate the Emergency Response System (ERS).

Ans: D) Activate the Emergency Response System (ERS). Feedback: After checking for responsiveness and breathing, the nurse should activate the ERS. Assessment of carotid pulse should follow and chest compressions may be indicated. Illuminating the call light is an insufficient response.

The cardiac monitor alarm alerts the critical care nurse that the patient is showing no cardiac rhythm on the monitor. The nurse's rapid assessment suggests cardiac arrest. In providing cardiac resuscitation documentation, how will the nurse describe this initial absence of cardiac rhythm? A) Pulseless electrical activity (PEA) B) Ventricular fibrillation C) Ventricular tachycardia D) Asystole

Ans: D) Asystole Feedback: Cardiac arrest occurs when the heart ceases to produce an effective pulse and circulate blood. It may be caused by a cardiac electrical event such as ventricular fibrillation, ventricular tachycardia, profound bradycardia, or when there is no heart rhythm at all (asystole). Cardiac arrest may also occur when electrical activity is present, but there is ineffective cardiac contraction or circulating volume, which is PEA. Asystole is the only condition that involves the absolute absence of a heart rhythm.

The nurse is reviewing a newly admitted patient's electronic health record, which notes a history of orthopnea? What nursing action is most clearly indicated? A) Teach the patient deep breathing and coughing exercises. B) Administer supplemental oxygen at all times. C) Limit the patient's activity level. D) Avoid positioning the patient supine.

Ans: D) Avoid positioning the patient supine. Feedback: Orthopnea is defined as difficulty breathing while lying flat. This is a possible complication of HF and, consequently, the nurse should avoid positioning the patient supine. Oxygen supplementation may or may not be necessary and activity does not always need to be curtailed. Deep breathing and coughing exercises do not directly address this symptom.

The triage nurse in the ED is performing a rapid assessment of a man with complaints of severe chest pain and shortness of breath. The patient is diaphoretic, pale, and weak. When the patient collapses, what should the nurse do first? A) Check for a carotid pulse. B) Apply supplemental oxygen. C) Give two full breaths. D) Gently shake and shout, "Are you OK?"

Ans: D) Gently shake and shout, "Are you OK?" Feedback: Assessing responsiveness is the first step in basic life support. Opening the airway and checking for respirations should occur next. If breathing is absent, two breaths should be given, usually accompanied by supplementary oxygen. Circulation is checked by palpating the carotid artery.

An older adult patient with HF is being discharged home on an ACE inhibitor and a loop diuretic. The patient's most recent vital signs prior to discharge include oxygen saturation of 93% on room air, heart rate of 81 beats per minute, and blood pressure of 94/59 mm Hg. When planning this patient's subsequent care, what nursing diagnosis should be identified? A) Risk for ineffective tissue perfusion related to dysrhythmia B) Risk for fluid volume excess related to medication regimen C) Risk for ineffective breathing pattern related to hypoxia D) Risk for falls related to hypotension

Ans: D) Risk for falls related to hypotension Feedback: The combination of low BP, diuretic use, and ACE inhibitor use constitute a risk for falls. There is no evidence, or heightened risk, of dysrhythmia. The patient's medications create a risk for fluid deficit, not fluid excess. Hypoxia is a risk for all patients with HF, but this is not in evidence for this patient at this time.

The nurse is assessing an older adult patient with numerous health problems. What assessment datum indicates an increase in the patient's risk for heart failure (HF)? A) The patient takes Lasix (furosemide) 20 mg/day. B) The patient's potassium level is 4.7 mEq/L. C) The patient is an African American man. D) The patient's age is greater than 65.

Ans: D) The patient's age is greater than 65. Feedback: HF is the most common reason for hospitalization of people older than 65 years of age and is the second most common reason for visits to a physician's office. A potassium level of 4.7 mEq/L is within reference range and does not indicate an increased risk for HF. The fact that the patient takes Lasix 20 mg/day does not indicate an increased risk for HF, although this drug is often used in the treatment of HF. The patient being an African American man does not indicate an increased risk for HF.

25. A 16-year-old being treated for hypertension has laboratory values of hemoglobin B 16 g/dL, hematocrit level 43%, sodium 139 mEq/L, potassium 4.4 mEq/L, and total cholesterol of 220 mg/dL. Which drug does the nurse suspect the patient takes based on the total cholesterol? 1. Beta blockers. 2. Calcium channel blockers. 3. ACE inhibitors. 4. Diuretics.

Answer: 1. Beta blockers. Rationale: 1. Beta blockers are used with caution in patients with hyperlipidemia, hyper- glycemia, and impotence. 2. Calcium channel blockers do not affect these blood levels. 3. ACE inhibitors do not affect these blood levels. 4. Diuretics do not affect these blood levels. TEST-TAKING HINT: The test taker needs to know side effects of drugs.

39. Exposure to which illness should be a cause to discontinue therapy and substitute dipyridamole (Persantine) in a child receiving aspirin therapy for Kawasaki disease (KD)? 1. Chickenpox or influenza. 2. E. coli or staphylococcus. 3. Mumps or streptococcus A. 4. Streptococcus A or staphylococcus.

Answer: 1. Chickenpox or influenza. Rationale: 1. Both chickenpox and influenza are viral in nature, so consider stopping the aspirin because of the danger of Reye syndrome. 2. E. coli and staphylococcus are not viral, so Reye syndrome is not a factor. 3. Mumps is caused by a virus, so aspirin should not be used to treat fever. Streptococcus A is a bacterium; Reye syndrome is not a factor. 4. Streptococcus A and staphylococcus are not viral, so Reye syndrome is not a factor. TEST-TAKING HINT: Consider Reye syndrome when the patient is taking aspirin and has a viral infection.

36. A child born with Down syndrome should be evaluated for which associated cardiac manifestation? 1. Congenital heart defect (CHD). 2. Systemic hypertension. 3. Hyperlipidemia. 4. Cardiomyopathy.

Answer: 1. Congenital heart defect (CHD). Rationale: 1. CHD is found often in children with Down syndrome. 2. This is not associated with Down syndrome. 3. This is not associated with Down syndrome. 4. This is not associated with Down syndrome. TEST-TAKING HINT: A child with a syndrome, such as Down, is likely to have other abnormalities.

42. A nursing action that promotes ideal nutrition in an infant with congestive heart failure (CHF) is: 1. Feeding formula that is supplemented with additional calories. 2. Allowing the infant to nurse at each breast for 20 minutes. 3. Providing large feedings every 5 hours. 4. Using firm nipples with small openings to slow feedings.

Answer: 1. Feeding formula that is supplemented with additional calories. Rationale: 1. Formula can be supplemented with extra calories, either from a commer- cial supplement, such as Polycose, or from corn syrup. Calories in formula could increase from 20 kcal/oz to 30 kcal/oz or more. 2. The infant would get too tired while feeding, which increases cardiac demand. Limit breastfeeding to a half hour, or 15 minutes per side. 3. Smaller feedings more often, such as every 2 to 3 hours, would decrease cardiac demand. 4. Soft nipples that are easy for the infant to suck would make for less work getting nutrition. TEST-TAKING HINT: Allow the child to get the most nutrition most effectively.

30. A child diagnosed with congestive heart failure (CHF) is receiving maintenance doses of digoxin and furosemide. She is rubbing her eyes when she is looking at the lights in the room, and her HR is 70 beats per minute. The nurse expects which laboratory finding? 1. Hypokalemia. 2. Hypomagnesemia. 3. Hypocalcemia. 4. Hypophosphatemia.

Answer: 1. Hypokalemia. Rationale: 1. The rubbing of the child's eyes may mean that she is seeing halos around the lights, indicating digoxin toxicity. The HR is slow for her age and also indicates digoxin toxicity. A decrease in serum potassium because of the furosemide can increase the risk for digoxin toxicity. 2. Hypomagnesemia does not affect digoxin and is not related to the child rubbing her eyes. 3. Hypocalcemia does not affect digoxin and is not related to the child rubbing her eyes. 4. Hypophosphatemia does not affect digoxin and is not related to the child rubbing her eyes. TEST-TAKING HINT: The test taker knows that furosemide causes the loss of potassium and can cause digoxin toxicity.

16. The nurse is caring for a child with Kawasaki disease (KD). A student nurse who is on the unit asks if there are medications to treat this disease. The nurse's response to the student nurse is: 1. Immunoglobulin G and aspirin. 2. Immunoglobulin G and ACE inhibitors. 3. Immunoglobulin E and heparin. 4. Immunoglobulin E and ibuprofen.

Answer: 1. Immunoglobulin G and aspirin. Rationale: 1. High-dose immunoglobulin G and salicylate therapy for inflammation are the current treatment for KD. 2. Immunoglobulin G is correct, but ACE inhibitors are incorrect for treatment. 3. Heparin may be used for the child with an aneurysm, but not immuno-globulin E. 4. Immunoglobulin E and ibuprofen are not correct. TEST-TAKING HINT: Consider anti-inflammatory medications for treatment of KD.

53. Which patient could require feeding by gavage? 1. Infant with congestive heart failure (CHF). 2. Toddler with repair of transposition of the great vessels. 3. Toddler with Kawasaki disease (KD) in the acute phase. 4. School-age child with rheumatic fever (RF) and chorea.

Answer: 1. Infant with congestive heart failure (CHF). Rationale: 1. The child may experience increased cardiac demand while feeding. Feed- ings by gavage eliminate that work and still provide high-calorie intake for growth. 2. Transposition of the great vessels should be repaired before the toddler years, so that child would not need to be gavage-fed. 3. A toddler with KD in the acute phase does not need to be gavage-fed. 4. An RF patient with St. Vitus' dance (chorea) does not need to be gavage-fed. Most of these children do not have CHF. TEST-TAKING HINT: The test taker should consider how gavage feedings would affect the work of the heart.

54. Which physiological changes occur as a result of hypoxemia in congestive heart failure (CHF)? 1. Polycythemia and clubbing. 2. Anemia and barrel chest. 3. Increased white blood cells and low platelets. 4. Elevated erythrocyte sedimentation rate and peripheral edema.

Answer: 1. Polycythemia and clubbing. Rationale: 1. The hypoxemia stimulates erythro- poiesis, which causes polycythemia, in an attempt to increase oxygen by having more red blood cells carry oxygen. Clubbing of the fingers is a result of the polycythemia and hypoxemia. 2. Anemia and barrel chest do not occur as a result of hypoxemia. Hypoxemia stimulates the production of erythropoietin to increase the number of red blood cells to carry more oxygen. The barrel chest is the result of air trapping. 3. Increased white blood cells occur as the result of an infection, not hypoxemia. Hypoxemia does not cause a decreased number of platelets. 4. An elevated erythrocyte sedimentation rate is the result of inflammation in the body. Peripheral edema can be caused by CHF. TEST-TAKING HINT: The test taker could eliminate answers 2, 3, and 4 by knowing that they do not cause hypoxemia in CHF.

46. The parents of a 3-month-old ask why their baby will not have an operation to correct a ventricular septal defect (VSD). The nurse's best response is: 1. "It is always helpful to get a second opinion about any serious condition like this." 2. "Your baby's defect is small and will likely close on its own by 1 year of age." 3. "It is common for physicians to wait until an infant develops respiratory distress before they do the surgery." 4. "With a small defect like this, they wait until the child is 10 years old to do the surgery."

Answer: 2. "Your baby's defect is small and will likely close on its own by 1 year of age." Rationale: 1. This is not a collegial response, and the nurse should explain to the parents why an operation is not necessary now. 2. Usually a VSD will close on its own within the first year of life. 3. It is not common for physicians to wait until respiratory distress develops because that puts the infant at greater risk for complications. The defect is small and will likely close on its own. 4. Small defects usually close on their own within the first year. TEST-TAKING HINT: Know the various treatments depending on size of the defect. VSD is the most common CHD.

44. Treatment for congestive heart failure (CHF) in an infant began 3 days ago and has included digoxin and furosemide. The child no longer has retractions, lungs are clear, and HR is 96 beats per minute while the child sleeps. The nurse is confident that the child has diuresed successfully and has good renal perfusion when the nurse notes the child's urine output is: 1. 0.5 cc/kg/hr. 2. 1 cc/kg/hr. 3. 30 cc/hr. 4. 1 oz/hr.

Answer: 2. 1 cc/kg/hr. Rationale: 1. This is incorrect because 0.5 cc/kg/hr is below the normal pediatric urine output. 2. Normal pediatric urine output is 1 cc/kg/hr. 3. This is incorrect because 30 cc/hr is above the normal pediatric urine output. 4. This is incorrect because 1 oz/hr is above the normal pediatric urine output. TEST-TAKING HINT: The test taker needs to know that normal urine output for a child is 1 cc/kg/hr.

22. BP screenings to detect end-organ damage should be done routinely beginning at what age? 1. Birth. 2. 3 years. 3. 8 years. 4. 13 years.

Answer: 2. 3 years. Rationale: 1. Birth is too early, and readings are often not reliable due to patient movement. 2. Age 3 years is the recommended age to establish a baseline BP in a normal healthy child. 3. Age 8 years is too late to detect early damage. 4. Age 13 years is too late to detect early damage. TEST-TAKING HINT: The test taker needs to know that 3 years of age is the recommended age to begin BP measurements in healthy children.

49. A child has been seen by the school nurse for dizziness since the start of the school term. It happens when standing in line for recess and homeroom. The child now reports that she would rather sit and watch her friends play hopscotch because she cannot count out loud and jump at the same time. When the nurse asks her if her chest ever hurts, she says yes. Based on this history, the nurse suspects that she has: 1. Ventricular septal defect (VSD). 2. Aortic stenosis (AS). 3. Mitral valve prolapse. 4. Tricuspid atresia.

Answer: 2. Aortic stenosis (AS). Rationale: 1. Murmur and CHF are often found in infancy. 2. AS can progress, and the child can develop exercise intolerance that can be better when resting. 3. Mitral valve prolapse causes a murmur and palpitations, usually in adulthood. 4. Tricuspid atresia causes hypoxemia in infancy. TEST-TAKING HINT: What does each of the last words of the defects mean, and what do those cause?

38. A child has a Glasgow Coma Scale of 3, HR of 88 beats per minute and regular, respiratory rate of 22, BP of 78/52, and blood sugar of 35 mg/dL. The nurse asks the caregiver about accidental ingestion of which drug? 1. Calcium channel blocker. 2. Beta blocker. 3. ACE inhibiter. 4. ARB.

Answer: 2. Beta blocker. Rationale: 1. Calcium channel blockers decrease the force of cardiac contraction and slow the electrical conduction of the heart, resulting in slowing of the HR. The HR is normal in this child. 2. The beta blocker not only affects the heart and lungs but also blocks the beta sites in the liver, reducing the amount of glycogen available for use, causing hypoglycemia. The lower HR and BP also suggest ingestion of a cardiac medication. 3. ACE inhibiters block the conversion of a protein from its inactive to its active form. The protein causes constriction of small blood vessels, which raises BP. By blocking this protein, BP is lowered. 4. Angiotensin receptor blockers relax blood vessels, which lowers BP and makes it easier for the heart to pump blood. TEST-TAKING HINT: Know the drug's side effects. In this case, the glucose is blocked.

50. The school nurse has been following a child who comes to the office frequently for vague complaints of dizziness and headache. Today, she is brought in after fainting in the cafeteria following a nosebleed. Her BP is 122/85, and her radial pulses are bounding. The nurse suspects she has: 1. Transposition of the great vessels. 2. Coarctation of the aorta (COA). 3. Aortic stenosis (AS). 4. Pulmonic stenosis (PS).

Answer: 2. Coarctation of the aorta (COA). Rationale: 1. Transposition of the great vessels does not cause these symptoms. 2. In the older child, COA causes dizzi- ness, headache, fainting, elevated blood pressure, and bounding radial pulses. 3. AS does not cause these symptoms. 4. PS does not cause these symptoms. TEST-TAKING HINT: The test taker should recognize that the child's BP is elevated and her pulses are bounding, which are symptoms of COA.

23. What associated manifestation might the nurse occasionally find in a child diagnosed with Wilms tumor? 1. Atrial fibrillation. 2. Hypertension. 3. Endocarditis. 4. Hyperlipidemia.

Answer: 2. Hypertension. Rationale: 1. Wilms tumor does not affect or cause this condition. 2. Because Wilms tumor sits on the kid- ney, it can be associated with secondary hypertension. It does not affect or cause the other conditions. 3. Wilms tumor does not affect or cause this condition. 4. Wilms tumor does not affect or cause this condition. TEST-TAKING HINT: Where is the Wilms tumor located?

37. The Norwood procedure is used to correct: 1. Transposition of the great vessels. 2. Hypoplastic left heart syndrome. 3. Tetralogy of Fallot (TOF). 4. Patent ductus arteriosus (PDA).

Answer: 2. Hypoplastic left heart syndrome. Rationale: 1. Transposition of the great vessels requires different surgical procedures. 2. The Norwood procedure is specific to hypoplastic left heart syndrome. 3. TOF requires different surgical procedures. 4. PDA requires different surgical procedures. TEST-TAKING HINT: Review surgical treatment of CHD.

43. An 18-month-old with a myelomeningocele is undergoing a cardiac catheterization. The mother expresses concern about the use of dye in the procedure. The child does not have any allergies. In addition to the concern for an iodine allergy, what other allergy should the nurse bring to the attention of the catheterization staff? 1. Soy. 2. Latex. 3. Penicillin. 4. Dairy

Answer: 2. Latex. Rationale: 1. Children with spina bifida (myelo -meningocele) often have a latex allergy. The catheter balloon is often made of latex, and all personnel caring for the patient should be made aware of the allergy. 2.Children with spina bifida (myelo -meningocele) often have a latex allergy. The catheter balloon is often made of latex, and all personnel caring for the patient should be made aware of the allergy. 3. Children with spina bifida (myelo -meningocele) often have a latex allergy. The catheter balloon is often made of latex, and all personnel caring for the patient should be made aware of the allergy. 4. Children with spina bifida (myelo -meningocele) often have a latex allergy. The catheter balloon is often made of latex, and all personnel caring for the patient should be made aware of the allergy. TEST-TAKING HINT: Material that composes the balloon catheter is made of latex, which is a common allergy in a child with a myelomeningocele

57. The mother of a toddler reports that the child's father has just had a myocardial infarction (MI). Because of this information, the nurse recommends the child have a(n): 1. Electrocardiogram. 2. Lipid profile. 3. Echocardiogram. 4. Cardiac catheterization.

Answer: 2. Lipid profile. Rationale: 1. Current recommendations are for a lipid profile in children over 2 years with a first- or second-degree relative with stroke, myocardial infarction, angina, or sudden cardiac death. Also screen if parent, sibling, or grandparent has cholesterol of 240 mg/dL or greater. 2. Current recommendations are for a lipid profile in children over 2 years with a first- or second-degree relative with stroke, myocardial infarction, angina, or sudden cardiac death. Also screen if parent, sibling, or grandpar- ent has cholesterol of 240 mg/dL or greater. 3. Current recommendations are for a lipid profile in children over 2 years with a first- or second-degree relative with stroke, myocardial infarction, angina, or sudden cardiac death. Also screen if parent, sibling, or grandparent has cholesterol of 240 mg/dL or greater. 4. Current recommendations are for a lipid profile in children over 2 years with a first- or second-degree relative with stroke, myocardial infarction, angina, or sudden cardiac death. Also screen if parent, sibling, or grandparent has cholesterol of 240 mg/dL or greater. TEST-TAKING HINT: Think about the cause of the father's MI.

13. While assessing a newborn with respiratory distress, the nurse auscultates a machine-like heart murmur. Other findings are a wide pulse pressure, periods of apnea, increased PaCO2, and decreased PO2. The nurse suspects that the newborn has: 1. Pulmonary hypertension. 2. Patent ductus arteriosus (PDA). 3. Ventricular septal defect (VSD). 4. Bronchopulmonary dysplasia.

Answer: 2. Patent ductus arteriosus (PDA). Rationale: 1. Pulmonary hypertension is a pulmonary condition, which does not create a heart murmur. 2. The main identifier in the stem is the machine-like murmur, which is the hallmark of a PDA. 3. A VSD does not produce a machine-like murmur. 4. Bronchopulmonary dysplasia is a pulmonary condition, which does not create a heart murmur. TEST-TAKING HINT: The test taker need to know common murmur sounds.

27. While looking through the chart of an infant with a congenital heart defect (CHD) of decreased pulmonary blood flow, the nurse would expect which laboratory finding? 1. Decreased platelet count. 2. Polycythemia. 3. Decreased ferritin level. 4. Shift to the left.

Answer: 2. Polycythemia. Rationale: 1. The nurse should expect a normal platelet count in an infant with a CHD of decreased pulmonary blood flow. 2. Polycythemia is the result of the body attempting to increase the oxygen sup- ply in the presence of hypoxia by in- creasing the total number of red blood cells to carry the oxygen. 3. Ferritin measures the amount of iron stored in the body and not affected by decreased pulmonary blood flow. 4. "Shift to the left" refers to an increase in the number of immature white blood cells. TEST-TAKING HINT: The test taker needs to know what laboratory values hypoxia can affect.

33. A 10-year-old has undergone a cardiac catheterization. At the end of the procedure, the nurse should first assess: 1. Pain. 2. Pulses. 3. Hemoglobin and hematocrit levels. 4. Catheterization report.

Answer: 2. Pulses. Rationale: 1. Pain needs to be assessed post procedure but is not the priority. 2. Checking for pulses, especially in the canulated extremity, would assure perfusion to that extremity and is the priority post procedure. 3. Hemoglobin and hematocrit levels would be checked post procedure if the child had bled very much during or after the procedure. 4. The catheterization report would be of interest to know what was determined from the procedure. This would also be good to check on the patient post procedure. TEST-TAKING HINT: The test taker would know that the priority is assessing the cannulated extremity, checking for adequate perfusion.

55. Aspirin has been ordered for the child with rheumatic fever (RF) in order to: 1. Keep the patent ductus arteriosus (PDA) open. 2. Reduce joint inflammation. 3. Decrease swelling of strawberry tongue. 4. Treat ventricular hypertrophy of endocarditis.

Answer: 2. Reduce joint inflammation. Rationale: 1. Aspirin is not used to treat this condition. A PDA does not occur with RF. 2. Joint inflammation is experienced in RF; aspirin therapy helps with inflammation and pain. 3. Strawberry tongue is manifested in KD; aspirin is not used to treat this disease. 4. Aspirin is not used to treat this condition. TEST-TAKING HINT: Know the manifesta- tions of RF.

7. Which finding might delay a cardiac catheterization procedure on a 1-year-old? 1. 30th percentile for weight. 2. Severe diaper rash. 3. Allergy to soy. 4. Oxygen saturation of 91% on room air.

Answer: 2. Severe diaper rash. Rationale: 1. This may be a reason the child needs the catheterization. 2. A child with severe diaper rash has potential for infection if the interven- tionist makes the standard groin approach. 3. Shellfish, not soy, is an allergy concern. 4. This may be a reason the child needs the catheterization. TEST-TAKING HINT: Consider the risk for infection as a delaying factor

40. The nurse is caring for an 8-year-old girl whose parents indicate she has developed spastic movements of her extremities and trunk, facial grimace, and speech disturbances. They state it seems worse when she is anxious and does not occur while sleeping. The nurse questions the parents about which recent illness? 1. Kawasaki disease (KD). 2. Strep throat. 3. Malignant hypertension. 4. Atrial fibrillation.

Answer: 2. Strep throat. Rationale: 1. KD does not result in this condition, called chorea or St. Vitus' dance. 2. Chorea can be a manifestation of RF, with a higher incidence in females. 3. Malignant hypertension does not result in this condition, called chorea or St. Vitus' dance. 4. Atrial fibrillation is not an illness. TEST-TAKING HINT: The test taker can eliminate answer 1 because KD can cause damage to coronary arteries.

60. Family discharge teaching has been effective when the parent of a toddler diagnosed with Kawasaki disease (KD) states: 1. "The arthritis in her knees is permanent. She will need knee replacements." 2. "I will give her diphenhydramine (Benadryl) for her peeling palms and soles of her feet." 3. "I know she will be irritable for 2 months after her symptoms started." 4. "I will continue with high doses of Tylenol for her inflammation."

Answer: 3. "I know she will be irritable for 2 months after her symptoms started." Rationale: 1. Arthritis in KD is always temporary. 2. Peeling palms and feet are painless. 3. Children can be irritable for 2 months after the symptoms of the disease start. 4. Tylenol is never given in high doses due to liver failure, and it is not an anti- inflammatory. Aspirin is given in high doses for KD. TEST-TAKING HINT: The test taker must know about KD to choose the best response.

12. On examination, a nurse hears a murmur at the left sternal border (LSB) in a child with diarrhea and fever. The parent asks why the pediatrician never said anything about the murmur. The nurse explains: 1. "The pediatrician is not a cardiologist." 2. "Murmurs are difficult to detect, especially in children." 3. "The fever increased the intensity of the murmur." 4. "We need to refer the child to an interventional cardiologist."

Answer: 3. "The fever increased the intensity of the murmur." Rationale: 1. This is not a collegial response. 2. The increased CO of the fever increases the intensity of the murmur, making it easier to hear. 3. The increased CO of the fever increases the intensity of the murmur, making it easier to hear. 4. This child does not need to see an interventional cardiologist. The murmur needs to be diagnosed first, and then a treatment plan would be developed. TEST-TAKING HINT: Consider the pathophysiology of fever.

61. Which assessment indicates that the parent of a 7-year-old is following the prescribed treatment for congestive heart failure (CHF)? 1. HR of 56 beats per minute. 2. Elevated red blood cell count. 3. 50th percentile height and weight for age. 4. Urine output of 0.5 cc/kg/hr.

Answer: 3. 50th percentile height and weight for age. Rationale: 1. HR of 56 beats per minute is likely due to digoxin toxicity. 2. Elevated count of red blood cells indicates polycythemia secondary to hypoxemia. 3. The 50th percentile height and weight for age shows good growth and development, indicating good nutrition and perfusion. 4. Urine output of 0.5 cc/kg/hr indicates that furosemide is not being given as ordered; the output is too low. TEST-TAKING HINT: The test taker should know the expected responses of medications used to treat CHF.

24. Which drug should not be used to control secondary hypertension in a sexually active adolescent female who uses intermittent birth control? 1. Beta blockers. 2. Calcium channel blockers. 3. ACE inhibitors. 4. Diuretics.

Answer: 3. ACE inhibitors. Rationale: 1. ACE inhibitors and angiotensin II receptor blockers can cause birth defects. The others are not teratogenic. 2. ACE inhibitors and angiotensin II receptor blockers can cause birth defects. The others are not teratogenic. 3.ACE inhibitors and angiotensin II receptor blockers can cause birth de- fects. The others are not teratogenic. 4. ACE inhibitors and angiotensin II receptors can cause birth defects. The others are not teratogenic. TEST-TAKING HINT: The test taker needs to know which of these drugs are teratogenic.

32. In which congenital heart defect (CHD) would the nurse need to take upper and lower extremity BPs? 1. Transposition of the great vessels. 2. Aortic stenosis (AS). 3. Coarctation of the aorta (COA). 4. Tetralogy of Fallot (TOF).

Answer: 3. Coarctation of the aorta (COA). Rationale: 1. BPs would not need to be taken in both the upper and lower extremities in trans- position of the great vessels. The aorta and pulmonary arteries are in opposite positions, which does not change the BP readings. 2. AS is a narrowing of the aortic valve, which does not affect the BP in the extremities. 3. With COA there is narrowing of the aorta, which increases pressure proxi- mal to the defect (upper extremities) and decreases pressure distal to the defect (lower extremities). There will be high BP and strong pulses in the upper extremities and lower- than-expected BP and weak pulses in the lower extremities. 4. TOF is a congenital cardiac problem with four defects that do not affect the BP in the extremities. TEST-TAKING HINT: The test taker must know the anatomy of the defects and what assessments are to be made in each one.

31. Which plan would be appropriate in helping to control congestive heart failure (CHF) in an infant? 1. Promoting fluid restriction. 2. Feeding a low-salt formula. 3. Feeding in semi-Fowler position. 4. Encouraging breast milk.

Answer: 3. Feeding in semi-Fowler position. Rationale: 1. The nurse would not need to restrict fluids, as the child likely would not be getting overloaded with oral fluids. 2. The infant likely will have sodium depletion because of the chronic diuretic use; the infant needs a normal source of sodium, so low-sodium formula would not be used. 3. The infant has a great deal of difficulty feeding with CHF, so even getting the maintenance fluids is a challenge. The infant is fed in the more upright position so fluid in the lungs can go to the base of the lungs, allowing better expansion. 4. Breast milk has slightly less sodium than formula, and the child needs a normal source of sodium because of the diuretic. TEST-TAKING HINT: Infants are not able to concentrate urine well and may have sodium depletion, so they need a normal source of sodium.

59. A heart transplant may be indicated for a child with severe heart failure and: 1. Patent ductus arteriosus (PDA). 2. Ventricular septal defect (VSD). 3. Hypoplastic left heart syndrome. 4. Pulmonic stenosis (PS).

Answer: 3. Hypoplastic left heart syndrome. Rationale: 1. Severe heart failure can be an indication for heart transplant if quality of life is decreased. 2. Severe heart failure can be an indication for heart transplant if quality of life is decreased. 3. Hypoplastic left heart syndrome is treated by the Norwood procedure, or heart transplant. 4. Severe heart failure can be an indication for heart transplant if quality of life is decreased. TEST-TAKING HINT: Consider severe heart failure and which complex of CHD.

51. Which medication should the nurse give to a child diagnosed with transposition of the great vessels? 1. Ibuprofen. 2. Betamethasone. 3. Prostaglandin E. 4. Indocin.

Answer: 3. Prostaglandin E. Rationale: 1. Ibuprofen blocks prostaglandins, which would speed up the closing of the PDA. 2. Betamethasone blocks prostaglandins, which would speed up the closing of the PDA. 3. Prostaglandin E inhibits closing of the PDA, which connects the aorta and pulmonary artery. 4. Indocin is used to treat osteoarthritis and gout. TEST-TAKING HINT: The test taker would know that children who have transposition of the great vessels also have another cardiac defect, and the common one is PDA.

58. During play, a toddler with a history of tetralogy of Fallot (TOF) might assume which position? 1. Sitting. 2. Supine. 3. Squatting. 4. Standing.

Answer: 3. Squatting. Rationale: 1. The toddler will naturally assume this position to decrease preload by occluding venous flow from the lower extremities and increasing afterload. Increasing SVR in this position increases pulmonary blood flow. This occurs with squatting. 2. The toddler will naturally assume this position to decrease preload by occluding venous flow from the lower extremities and increasing afterload. Increasing SVR in this position increases pulmonary blood flow. This occurs with squatting. 3. The toddler will naturally assume this position to decrease preload by occluding venous flow from the lower extremities and increasing afterload. Increasing SVR in this position increases pulmonary blood flow. 4. The toddler will naturally assume this position to decrease preload by occluding venous flow from the lower extremities and increasing afterload. Increasing SVR in this position increases pulmonary blood flow. TEST-TAKING HINT: The child self-assumes this position during the spell.

41. The most common cardiac dysrhythmia in pediatrics is: 1. Ventricular tachycardia. 2. Sinus bradycardia. 3. Supraventricular tachycardia. 4. First-degree heart block.

Answer: 3. Supraventricular tachycardia. Rationale: 1. Ventricular tachycardia is uncommon in children. 2. Sinus bradycardia is uncommon in children. 3. Supraventricular tachycardia is most common in children. 4. First-degree heart block is uncommon in children. TEST-TAKING HINT: Consider a tachycardiac rhythm in a pediatric patient.

6. Which statement by a parent of an infant with congestive heart failure (CHF) who is being sent home on digoxin indicates the need for further education? 1. "I will give the medication at regular 12-hour intervals." 2. "If he vomits, I will not give a make-up dose." 3. "If I miss a dose, I will not give an extra dose" 4. "I will mix the digoxin in some formula to make it taste better."

Answer: 4. "I will mix the digoxin in some formula to make it taste better." Rationale: 1. This is appropriate for digoxin administration. 2. This is appropriate for digoxin administration. 3. This is appropriate for digoxin administration. 4. If the medication is mixed in his formula, and he refuses to drink the entire amount, the digoxin dose will be inadequate. TEST-TAKING HINT: What if the child does not drink all the formula?

35. A child has been diagnosed with valvular disease following rheumatic fever (RF). During patient teaching, the nurse discusses the child's long-term prophylactic therapy with antibiotics for dental procedures, surgery, and childbirth. The parents indicate they understand when they say: 1. "She will need to take the antibiotics until she is 18 years old." 2. "She will need to take the antibiotics for 5 years after the last attack." 3. "She will need to take the antibiotics for 10 years after the last attack." 4. "She will need to take the antibiotics for the rest of her life."

Answer: 4. "She will need to take the antibiotics for the rest of her life." Rationale: 1. This could be true for a patient with a less severe form of RF. 2. This could be true for a patient with a less severe form of RF. 3. This could be true for a patient with a less severe form of RF. 4. Valvular involvement indicates significant damage, so antibiotics would be taken for the rest of her life. TEST-TAKING HINT: The test taker would know that the severity of the damage to the heart valves determines how long pro-phylaxic antibiotics will be administered

8. The nurse is caring for a child who has undergone a cardiac catheterization. During recovery, the nurse notices the dressing is saturated with bright red blood. The nurse's first action is to: 1. Call the interventional cardiologist. 2. Notify the cardiac catheterization laboratory that the child will be returning. 3. Apply a bulky pressure dressing over the present dressing. 4. Apply direct pressure 1 inch above the puncture site.

Answer: 4. Apply direct pressure 1 inch above the puncture site. Rationale: 1. This is not an appropriate action. 2. This is not an appropriate action. 3. This can be done after applying direct pressure 1 inch above the puncture site. 4. Applying direct pressure 1 inch above the puncture site will localize pressure over the vessel site. TEST-TAKING HINT: Consider the risk for volume depletion.

18. During a well-child checkup for an infant with tetralogy of Fallot (TOF), the child develops severe respiratory distress and becomes cyanotic. The nurse's first action should be to: 1. Lay the child flat to promote hemostasis. 2. Lay the child flat with legs elevated to increase blood flow to the heart. 3. Sit the child on the parent's lap, with legs dangling, to promote venous pooling. 4. Hold the child in knee-chest position to decrease venous blood return.

Answer: 4. Hold the child in knee-chest position to decrease venous blood return. Rationale: 1. Laying the child flat would increase pre-load, increasing blood to the heart, there-fore making respiratory distress worse. 2. Laying the child flat with legs elevated would increase preload, increasing blood to the heart, therefore making respiratory distress worse. 3. Sitting the child on the parent's lap with legs dangling might possibly help, but it would not be as effective as the knee-chest position in occluding the venous return. 4. The increase in the SVR would increase afterload and increase blood return to the pulmonary artery. TEST-TAKING HINT: The test taker should choose the response that decreases the preload in this patient.

21. A toddler who has been hospitalized for vomiting due to gastroenteritis is sleeping and difficult to wake up. Assessment reveals vital signs of a regular HR of 220 beats per minute, respiratory rate of 30 per minute, BP of 84/52, and capillary refill of 3 seconds. Which dysrhythmia does the nurse suspect in this child? 1. Rapid pulmonary flutter. 2. Sinus bradycardia. 3. Rapid atrial fibrillation. 4. Supraventricular tachycardia.

Answer: 4. Supraventricular tachycardia. Rationale: 1. This is not a real dysrhythmia. 2. Sinus bradycardia is a slow rate for the child's age. 3. Rapid atrial fibrillation is an irregular rhythm. 4. SVT is often above 200 and a result of dehydration, which a vomiting child could have. The rapid rate causes a low CO, resulting in low BP and prolonged capillary refill. TEST-TAKING HINT: The HR is regular and very rapid for a child of any age. The child has been vomiting, which can result in dehydration.

28. The nurse is caring for a 9-month-old who was born with a congenital heart defect (CHD). Assessment reveals a HR of 160, capillary refill of 4 seconds, bilateral crackles, and sweat on the scalp. These are signs of _____________________.

Answer: Congestive heart failure or CHF. Rationale: TEST-TAKING HINT: All of these are signs of pump failure. The infant is likely to have diaphoresis only on the scalp. The signs are not unlike those of an adult with this condition.

15. A child who has reddened eyes with no discharge; red, swollen, and peeling palms and soles of the feet; dry, cracked lips; and a "strawberry tongue" most likely has _____________________.

Answer: Kawasaki disease or KD. Rationale: TEST-TAKING HINT: Classic signs of KD include red eyes with no discharge; dry, cracked lips; strawberry tongue; and red, swollen, and peeling palms and soles of the feet. Incidence of KD is higher in males. The strongest indicator for this disease is the hallmark strawberry tongue.

48. Patent ductus arteriosus causes what type of shunt? _____________________.

Answer: Left to right Rationale: Blood flows from the higher pressure aorta to the lower pressure pulmonary artery, resulting in a left to right shunt. TEST-TAKING HINT: What is the CHD classification of PDA?

47. The flow of blood through the heart with an atrial septal defect (ASD) is _____________________.

Answer: Left to right. Rationale: The pressures in the left side of the heart are greater, causing the flow of blood to be from an area of higher pressure to lower pressure, or left to right, increasing the pulmonary blood flow with the extra blood. TEST-TAKING HINT: What is the CHD classification of ASD?

10. Indomethacin may be given to close which congenital heart defect (CHD) in newborns? _____________________

Answer: Patent ductus arteriosus or PDA Rationale: TEST-TAKING HINT: Prostaglandins allow the duct to remain open; thus, a prostaglandin inhibitor, such as Indocin or ibuprofen, can help close the duct.

11. For the child with hypoplastic left heart syndrome, which drug may be given to allow the patent ductus arteriosus (PDA) to remain open until surgery? _____________________

Answer: Prostaglandin E. Rationale: TEST-TAKING HINT: Prostaglandin E main- tains ductal patency to promote blood flow until the Norwood procedure is begun. Consider the opposite of wanting to close the PDA.

20. A 6-month-old who has episodes of cyanosis after crying could have the congenital heart defect (CHD) of decreased pulmonary blood flow called _____________________.

Answer: Tetralogy of Fallot or TOF. Rationale: "Tet" spells are characteristic of TOF. TEST-TAKING HINT: Know the congenital heart defect classifications.

5. What should the nurse assess prior to administering digoxin? Select all that apply. 1. Sclera. 2. Apical pulse rate. 3. Cough. 4. Liver function test.

Answer: 2. Apical pulse rate. Rationale: 1. The sclera has nothing to do with CHF. 2. The apical pulse rate is assessed because digoxin decreases the HR, and if the HR is <60, digoxin should not be administered. 3. Cough would not be assessed before adminis- tration. It is more commonly seen in patients who have been prescribed ACE inhibitors. 4. Liver function tests are not assessed before digoxin is administered. Digoxin can lower HR and cause dysrhythmias. TEST-TAKING HINT: The test taker should know that the sclera and liver function tests have nothing to do with digoxin. Cough could be associated with ACE inhibitors.

34. Which statement by the mother of a child with rheumatic fever (RF) shows she has good understanding of the care of her child? 1. "I will apply heat to his swollen joints to promote circulation." 2. "I will have him do gentle stretching exercises to prevent contractures." 3. "I will give him the aspirin that is ordered for pain and inflammation." 4. "I will apply cold packs to his swollen joints to reduce pain."

Answer: 3. "I will give him the aspirin that is ordered for pain and inflammation." Rationale: 1. During the acute phase, limit any manipulation of the joint, and avoid heat or cold. 2. During the acute phase, limit any manipulation of the joint, and avoid heat or cold. 3. Aspirin is the drug of choice for treatment of RF. 4. During the acute phase, limit any manipulation of the joint, and avoid heat or cold. TEST-TAKING HINT: The test taker should know that aspirin is the drug of choice and that manipulation of the joint should be limited during the acute phase.

26. The _____________________ serves as the septal opening between the atria of the fetal heart.

Answer: Foramen ovale. Rationale: TEST-TAKING HINT: The foramen ovale is the septal opening between the atria of the fetal heart. The test taker needs to know basic fetal circulation.

45. A 3-month-old has been diagnosed with a ventricular septal defect (VSD). The flow of blood through the heart is _____________________.

Answer: Left to right. Rationale: The pressures in the left side of the heart are greater, causing the flow of blood to be from an area of higher pressure to lower pressure, or left to right, increasing the pulmonary blood flow with the extra blood. TEST-TAKING HINT: The test taker should know that the classification for this defect is left to right.

14. Which are the most serious complications for a child with Kawasaki disease (KD)? Select all that apply. 1. Coronary thrombosis. 2. Coronary stenosis. 3. Coronary artery aneurysm. 4. Hypocoagulability. 5. Decreased sedimentation rate. 6. Hypoplastic left heart syndrome.

Answer: 1, 2, 3 1. Coronary thrombosis. 2. Coronary stenosis. 3. Coronary artery aneurysm. Rationale: 1. Thrombosis, stenosis, and aneurysm affect blood vessels. The child with KD has hypercoagulability and an increased sedimentation rate due to inflammation. 2. Thrombosis, stenosis, and aneurysm affect blood vessels. The child with KD has hypercoagulability and an increased sedimentation rate due to inflammation. 3. Thrombosis, stenosis, and aneurysm affect blood vessels. The child with KD has hypercoagulability and an increased sedimentation rate due to inflammation. 4. The child with KD has hypercoagulability and an increased sedimentation rate due to inflammation. 5. The child with KD has hypercoagulability and an increased sedimentation rate due to inflammation. 6. Hypoplastic left heart syndrome is a CHD and has no relation to KD. TEST-TAKING HINT: KD is an inflammation of small- and medium-sized blood vessels.

19. Hypoxic spells in the infant with a congenital heart defect (CHD) can cause which of the following? Select all that apply. 1. Polycythemia. 2. Blood clots. 3. Cerebrovascular accident. 4. Developmental delays. 5. Viral pericarditis. 6. Brain damage. 7. Alkalosis.

Answer: 1, 2, 3, 4, 6 1. Polycythemia. 2. Blood clots. 3. Cerebrovascular accident. 4. Developmental delays. 6. Brain damage. Rationale: 1. Hypoxia causes polycythemia, which can lead to increased blood viscosity, which can lead to blood clots and a stroke. 2. Hypoxia causes polycythemia, which can lead to increased blood viscosity, which can lead to blood clots and a stroke. 3. Hypoxia causes polycythemia, which can lead to increased blood viscosity, which can lead to blood clots and a stroke. 4. Developmental delays can be caused by multiple hospitalizations and surgeries. The child usually catches up to the appropriate level. 5. Hypoxia can increase the risk for bacterial endocarditis, not viral pericarditis. 6. Brain damage can be caused by hypoxia, blood clots, and stroke. 7. Hypoxic episodes cause acidosis, not alkalosis. TEST-TAKING HINT: Hypoxic episodes in a child with CHD ("tet spells") can cause polycythemia and strokes.

3. Tetralogy of Fallot (TOF) involves which defects? Select all that apply. 1. Ventricular septal defect (VSD). 2. Right ventricular hypertrophy. 3. Left ventricular hypertrophy. 4. Pulmonic stenosis (PS). 5. Pulmonic atresia. 6. Overriding aorta. 7. Patent ductus arteriosus (PDA).

Answer: 1, 2, 4, 6 1. Ventricular septal defect (VSD). 2. Right ventricular hypertrophy. 4. Pulmonic stenosis (PS). 6. Overriding aorta. Rationale: 1. TOF is a congenital defect with a ven- tricular septal defect, right ventricular hypertrophy, pulmonary valve stenosis, and overriding aorta. 2. TOF is a congenital defect with ventric- ular septal defect, right ventricular hypertrophy, pulmonary valve stenosis, and overriding aorta. 3. TOF is a congenital defect with ventricular septal defect, right ventricular hypertrophy, pulmonary valve stenosis, and overriding aorta. 4. TOF is a congenital defect with ventric- ular septal defect, right ventricular hypertrophy, pulmonary valve stenosis, and overriding aorta. 5. TOF is a congenital defect with ventricular septal defect, right ventricular hypertrophy, pulmonary valve stenosis, and overriding aorta. 6. TOF is a congenital defect with ventric- ular septal defect, right ventricular hypertrophy, pulmonary valve stenosis, and overriding aorta. 7. PDA is not one of the defects in tetralogy of Fallot. TEST-TAKING HINT: Tetralogy of Fallot has four defects. Pulmonary stenosis causes decreased pulmonary flow.

1.What can an electrocardiogram (ECG) detect? Select all that apply. 1. Ischemia. 2. Injury. 3. Cardiac output (CO). 4. Dysrhythmias. 5. Systemic vascular resistance (SVR). 6. Occlusion pressure. 7. Conduction delay.

Answer: 1, 2, 4, 7 1. Ischemia. 2. Injury. 4. Dysrhythmias. 7. Conduction delay. Rationale: 1. An electrocardiogram can indicate ischemia of the heart muscle. 2. An electrocardiogram can indicate injury to the heart muscle. 3. An electrocardiogram does not indicate CO. 4. An electrocardiogram can show dysrhythmias. 5. An electrocardiogram does not show SVR. 6. An electrocardiogram does not show occlusion pressures. 7. An electrocardiogram does show conduction delays. TEST-TAKING HINT: The electrocardiogram checks the electrical system of the heart, not the mechanical system. CO is mechanical; occlusion pressure does not have to do with the electrocardiogram; and SVR measures pressures in the peripheral system.

52. Which statement by the mother of a child with rheumatic fever (RF) shows she has an understanding of prevention for her other children? (Select all that apply) 1. "Whenever one of them gets a sore throat, I will give that child an antibiotic." 2. "There is no treatment. It must run its course." 3. "If their culture is positive for group A streptococcus, I will give them their antibiotic." 4. "If their culture is positive for staphylococcus A, I will give them their antibiotic."

Answer: 1, 3 1. "Whenever one of them gets a sore throat, I will give that child an antibiotic." 3. "If their culture is positive for group A streptococcus, I will give them their antibiotic." Rationale: 1. Do not use an antibiotic if the disease is not bacterial in origin. Most sore throats are viral. 2. RF is a bacterial infection caused by group A beta-hemolytic streptococcus, and the drug of choice is penicillin. 3. RF is caused by a streptococcus infection, not by staphylococcus. 4. RF is cause by a streptococcus infection, not by staphylococcus TEST-TAKING HINT: The test taker needs to know the cause of RF and how it is treated.

29. The following are examples of acquired heart disease. Select all that apply. 1. Infective endocarditis. 2. Hypoplastic left heart syndrome. 3. Rheumatic fever (RF). 4. Cardiomyopathy. 5. Kawasaki disease (KD). 6. Transposition of the great vessels.

Answer: 1, 3, 4, 5 1. Infective endocarditis. 3. Rheumatic fever (RF). 4. Cardiomyopathy. 5. Kawasaki disease (KD). Rationale: 1. Infective endocarditis is an example of an acquired heart problem. 2. Hypoplastic left heart syndrome is a CHD. 3. RF is an acquired heart problem. 4. Cardiomyopathy is an acquired heart problem. 5. KD is an acquired heart problem. 6. Transposition of the great vessels is a CHD. TEST-TAKING HINT: "Acquired" means occurring after birth and seen in an otherwise normal and healthy heart.

9. Which interventions decrease cardiac demands in an infant with congestive heart failure (CHF)? Select all that apply. 1. Allow parents to hold and rock their child. 2. Feed only when the infant is crying. 3. Keep the child uncovered to promote low body temperature. 4. Make frequent position changes. 5. Feed the child when sucking the fists. 6. Change bed linens only when necessary. 7. Organize nursing activities.

Answer: 1, 4, 5, 6, 7 1. Allow parents to hold and rock their child. 4. Make frequent position changes. 5. Feed the child when sucking the fists. 6. Change bed linens only when necessary. 7. Organize nursing activities. Rationale: 1. Rocking by the parents will comfort the infant and decrease demands. 2. The infant would not be fed when crying because crying increases cardiac demands. The infant might choke if the nipple is placed in the mouth and the child inhales when trying to swallow. 3. Keep the child normothermic to reduce metabolic demands. 4. Frequent position changes will de- crease the risk for infection by avoid- ing immobility with its potential for skin breakdown. 5. An infant sucking the fists could indicate hunger. 6. Change bed linens only when neces- sary to avoid disturbing the child. 7. Organize nursing activities to avoid disturbing the child. TEST-TAKING HINT: Do all that can be done to decrease demands on the child.

17. Congenital heart defects (CHDs) are classified by which of the following? Select all that apply. 1. Cyanotic defect. 2. Acyanotic defect. 3. Defects with increased pulmonary blood flow. 4. Defects with decreased pulmonary blood flow. 5. Mixed defects. 6. Obstructive defects. 7. Pansystolic murmurs.

Answer: 3, 4, 5, 6 3. Defects with increased pulmonary blood flow. 4. Defects with decreased pulmonary blood flow. 5. Mixed defects. 6. Obstructive defects. Rationale: 1. Heart defects are no longer classified as cyanotic or acyanotic. 2. Heart defects are no longer classified as cyanotic or acyanotic. 3. Heart defects are now classified as defects with increased or decreased pulmonary blood flow 4. Heart defects are now classified as defects with increased or decreased pulmonary blood flow. 5. Heart defects are now classified as defects with increased or decreased pulmonary blood flow. 6. Heart defects are now classified as defects with increased or decreased pulmonary blood flow. 7. A murmur may be heard with a CHD, but a murmur does not classify the defect. TEST-TAKING HINT: Know the new classifications, not the older ones.

56. Which vaccines must be delayed for 11 months after the administration of gamma globulin? Select all that apply. 1. Diphtheria, tetanus, and pertussis. 2. Hepatitis B. 3. Inactivated polio virus. 4. Measles, mumps, and rubella. 5. Varicella.

Answer: 4, 5 4. Measles, mumps, and rubella. 5. Varicella. Rationale: 1. Diphtheria, tetanus, and pertussis can be given following administration of gamma globulin. These are killed vaccines, and the only vaccines not administered would be live vaccines such as measles, mumps, rubella. 2. Hepatitis B can be administered following gamma globulin. Live vaccines are held for at least 11 months. 3. Inactivated polio virus can be given following gamma globulin administration. Live vaccines are held for 11 months. 4. The body might not produce the appropriate number of antibodies following gamma globulin infusion, so live virus vaccines should be delayed for 11 months. 5. The body might not produce the appropriate number of antibodies following gamma globulin infusion, so live virus vaccines should be delayed for 11 months. TEST-TAKING HINT: The test taker needs to know which vaccines are killed and which are live.

2. A newborn is diagnosed with a congenital heart defect (CHD). The test results reveal that the lumen of the duct between the aorta and pulmonary artery remains open. This defect is known as _____________________.

Answer: Patent ductus arteriosus or PDA. Rationale: TEST-TAKING HINT: This is a defect with increased pulmonary flow. It should close in the first few weeks of life.

4. A 10-year-old child is recovering from a severe sore throat. The parent states that the child complains of chest pain. The nurse observes that the child has swollen joints, nodules on the fingers, and a rash on the chest. The likely cause is _____________________.

Answer: Rheumatic fever or RF. Rationale: To make the diagnosis of RF, major and minor criteria are used. Major criteria include cardi- tis, subcutaneous nodules, erythema margina- tum, chorea, and arthritis. Minor criteria include fever and previous history of RF. TEST-TAKING HINT: It is an inflammatory disease caused by group A beta-hemolytic streptococcus.

To assess the dorsalis pedis artery, the nurse would use the tips of three fingers and apply light pressure to the:

Anterior surface of the foot near the ankle joint.

Which medication should the nurse expect the health-care provider to order for a client diagnosed with arterial occlusive disease? 3. An antiplatelet medication

Anti-platelet medications inhibit platelet aggregations in the arterial blood, such as aspirin or clopidogrel (Plavix).

Which would the nurse stress as a lifelong necessity for a client managing infectious endocarditis?

Antibiotic Therapy

During discharge teaching with a 68-year-old patient who had a mitral valve replacement with a mechanical valve, the nurse instructs the patient on the c. need for frequent laboratory blood testing.

Anticoagulation with warfarin (Coumadin) is needed for a patient with mechanical valves to prevent clotting on the valve.

Which client problem has priority for the client with a cardiac dysrhythmia? 2. Decreased cardiac output.

Any abnormal electrical activity of the heart causes decreased cardiac output.

Which assessment data would require immediate intervention by the nurse for the client who is six (6) hours post-operative abdominal aortic aneurysm repair? 1. Absent bilateral pedal pulses.

Any neurovascular abnormality in the client's lower extremities indicates the graft is occluded or possibly bleeding and requires immediate intervention by the nurse.

The client diagnosed with a myocardial infarction is six (6) hours post-right femoral percutaneous transluminal angioplasty (PTCA), also known as balloon surgery. Which assessment data would require immediate intervention by the nurse? 3. The client is complaining of numbness in the right foot.

Any neurovascular assessment data that are abnormal require intervention by the nurse; numbness may indicate decreased blood supply to the right foot.

The client is admitted to the telemetry unit diagnosed with acute exacerbation of congestive heart failure (CHF). Which signs/symptoms would the nurse expect to find when assessing this client?

Apical pulse rate of 110 and 4+ pitting edema of feet.

The client has just received a mechanical valve replacement. Which behavior by the client indicates the client needs more teaching? 3. The client takes an enteric-coated aspirin daily.

Aspirin and nonsteroidal anti-inflammatory drugs (NSAIDs) interfere with clotting and may potentiate the effects of the anticoagulant therapy, which the client with a mechanical valve will be prescribed. Therefore, the client should not take aspirin daily.

The nurse is caring for a client diagnosed with a myocardial infarction who is experiencing chest pain. Which interventions should the nurse implement? Select all that apply. 2. Administer an aspirin orally. 3. Apply oxygen via a nasal cannula.

Aspirin is an antiplatelet medication and should be administered orally. Oxygen will help decrease myocardial ischemia, thereby decreasing pain.

The client is one (1) day postoperative coronary artery bypass surgery. The client complains of chest pain. Which intervention should the nurse implement first?

Assess the client's chest dressing and vital signs. - if the chest pain is expected postop or if it is a complication

The client is scheduled for a right femoral cardiac catheterization. Which nursing intervention should the nurse implement after the procedure?

Assess the client's neurovascular status. - assess for pulses, paresthesia, paralysis, coldness and pallor - HOB elevated no more than 10 degrees, client kept in bedrest

Which intervention should the nurse implement when administering a loop diuretic to a client diagnosed with coronary artery disease?

Assess the client's serum potassium level. - assess apical pulse not radial pulse

The client had an abdominal aortic aneurysm repair two (2) days ago. Which intervention should the nurse implement first? 1. Assess the client's bowel sounds.

Assessment is the first part of the nursing process and is the first intervention the nurse should implement.

The nurse enters the room of the client diagnosed with congestive heart failure. The client is lying in bed gasping for breath, is cool and clammy, and has buccal cyanosis. Which intervention would the nurse implement first?

Assist the client to a sitting position THEN vital and check pulse ox then sponge the client's forehead

The nurse is developing a nursing care plan for a client diagnosed with congestive heart failure. A nursing diagnosis of "decreased cardiac output related to inability of the heart to pump effectively" is written. Which short-term goal would be best for the client? The client will: 2. Have an audible S1 and S2 with no S3 heard by end of shift.

Audible S1 and S2 sounds are normal for a heart with adequate output. An audible S3 sound might indicate left ventricular failure that could be life threatening.

Which of the following describes a valve used in replacement surgery that is made from the patient's own heart valve?

Autograft

14. The nurse identifies the collaborative problem of potential complication: pulmonary edema for a patient in ADHF. When assessing the patient, the nurse will be most concerned about a. an apical pulse rate of 106 beats/min. b. an oxygen saturation of 88% on room air. c. weight gain of 1 kg (2.2 lb) over 24 hours. d. decreased hourly patient urinary output.

B Rationale: A decrease in oxygen saturation to less than 92% indicates hypoxemia. The nurse should administer supplemental oxygen immediately to the patient. An increase in apical pulse rate, 1-kg weight gain, and decreases in urine output also indicate worsening heart failure and require rapid nursing actions, but the low oxygen saturation rate requires the most immediate nursing action. Cognitive Level: Analysis Text Reference: pp. 829-830 Nursing Process: Assessment NCLEX: Physiological Integrity

22. A patient who is receiving dobutamine (Dobutrex) for the treatment of ADHF has all of the following nursing actions included in the plan of care. Which action will be best for the RN to delegate to an experienced LPN/LVN? a. Teach the patient the reasons for remaining on bed rest. b. Monitor the patient's BP every hour. c. Adjust the drip rate to keep the systolic BP >90 mm Hg. d. Call the health care provider about a decrease in urine output.

B Rationale: An experienced LPN/LVN would be able to monitor BP and would know to report significant changes to the RN. Teaching patients and making adjustments to the drip rate for vasoactive medications are RN-level skills. Because the health care provider may order changes in therapy based on the decrease in urine output, the RN should call the health care provider about the decreased urine output. Cognitive Level: Application Text Reference: pp. 827-829 Nursing Process: Planning NCLEX: Safe and Effective Care Environment

5. When the nurse is developing a teaching plan to prevent the development of heart failure in a patient with stage 1 hypertension, the information that is most likely to improve compliance with antihypertensive therapy is that a. hypertensive crisis may lead to development of acute heart failure in some patients. b. hypertension eventually will lead to heart failure by overworking the heart muscle. c. high BP increases risk for rheumatic heart disease. d. high systemic pressure precipitates papillary muscle rupture.

B Rationale: Hypertension is a primary cause of heart failure because the increase in ventricular afterload leads to ventricular hypertrophy and dilation. Hypertensive crisis may precipitate acute heart failure is some patients, but this patient with stage 1 hypertension may not be concerned about a crisis that happens only to some patients. Hypertension does not directly cause rheumatic heart disease (which is precipitated by infection with group A -hemolytic streptococcus) or papillary muscle rupture (which is caused by myocardial infarction/necrosis of the papillary muscle). Cognitive Level: Application Text Reference: p. 822 Nursing Process: Planning NCLEX: Health Promotion and Maintenance

8. A patient admitted to the hospital with an exacerbation of chronic heart failure tells the nurse, "I felt fine when I went to bed, but I woke up in the middle of the night feeling like I was suffocating!" The nurse can best document this assessment information as a. pulsus alternans. b. paroxysmal nocturnal dyspnea. c. two-pillow orthopnea. d. acute bilateral pleural effusion.

B Rationale: Paroxysmal nocturnal dyspnea is caused by the reabsorption of fluid from dependent body areas when the patient is sleeping and is characterized by waking up suddenly with the feeling of suffocation. Pulsus alternans is the alternation of strong and weak peripheral pulses during palpation. Orthopnea indicates that the patient is unable to lie flat because of dyspnea. Pleural effusions develop over a longer time period. Cognitive Level: Comprehension Text Reference: p. 825 Nursing Process: Assessment NCLEX: Physiological Integrity

11. When developing a plan to decrease preload in the patient with heart failure, the nurse will include actions such as a. administering sedatives to promote rest and decrease myocardial oxygen demand. b. positioning the patient in a high-Fowler's position with the feet horizontal in the bed. c. administering oxygen per mask or nasal cannula. d. encouraging leg exercises to improve venous return.

B Rationale: Positioning the patient in a high-Fowler's position with the legs dependent will reduce preload by decreasing venous return to the right atrium. The other interventions may also be appropriate for patients with heart failure but will not help in decreasing preload. Cognitive Level: Application Text Reference: pp. 827-828 Nursing Process: Planning NCLEX: Physiological Integrity

7. Intravenous sodium nitroprusside (Nipride) is ordered for a patient with acute pulmonary edema. During the first hours of administration, the nurse will need to adjust the Nipride rate if the patient develops a. a drop in heart rate to 54 beats/min. b. a systolic BP <90 mm Hg. c. any symptoms indicating cyanide toxicity. d. an increased amount of ventricular ectopy.

B Rationale: Sodium nitroprusside is a potent vasodilator, and the major adverse effect is severe hypotension. After 48 hours of continuous use, cyanide toxicity is a possible (though rare) adverse effect. Reflex tachycardia (not bradycardia) is another adverse effect of this medication. Nitroprusside does not cause increased ventricular ectopy. Cognitive Level: Application Text Reference: p. 828 Nursing Process: Evaluation NCLEX: Physiological Integrity

10. The nurse working in the heart failure clinic will know that teaching for a 74-year-old patient with newly diagnosed heart failure has been effective when the patient a. says that the nitroglycerin patch will be used for any chest pain that develops. b. calls when the weight increases from 124 to 130 pounds in a week. c. tells the home care nurse that furosemide (Lasix) is taken daily at bedtime. d. makes an appointment to see the doctor at least once yearly.

B Rationale: Teaching for a patient with heart failure includes information about the need to weigh daily and notify the health care provider about an increase of 3 pounds in 2 days or 5 pounds in a week. Nitroglycerin patches are used primarily to reduce preload (not to prevent chest pain) in patients with heart failure and should be used daily, not on an "as necessary" basis. Diuretics should be taken earlier in the day to avoid nocturia and sleep disturbance. Heart failure is a chronic condition that will require frequent follow-up rather than an annual health care provider examination. Cognitive Level: Application Text Reference: pp. 826, 833-834, 838 Nursing Process: Evaluation NCLEX: Health Promotion and Maintenance

15. While admitting an 80-year-old patient with heart failure to the medical unit, the nurse obtains the information that the patient lives alone and sometimes confuses the "water pill" with the "heart pill." The nurse makes a note that discharge planning for the patient will need to include a. transfer to a dementia care service. b. referral to a home health care agency. c. placement in a long-term-care facility. d. arrangements for around-the-clock care.

B Rationale: The data about the patient suggest that assistance in developing a system for taking medications correctly at home is needed. A home health nurse will assess the patient's home situation and help the patient to develop a method for taking the two medications as directed. There is no evidence that the patient requires services such as dementia care, long-term-care, or around-the-clock home care. Cognitive Level: Application Text Reference: pp. 836-837 Nursing Process: Assessment NCLEX: Health Promotion and Maintenance

After a patient returns from cardiac catheterization, the nurse assesses that the pulse distal to the catheter insertion site is weaker. The nurse should: A. Elevate the affected extremity. B. Record the data on the nurse's notes. C. Notify the physician of the observation. D. Apply warm compresses to the insertion site.

B. Record the data on the nurse's notes. Elevation is not necessary; the extremity is kept straight. The pulse distal to the catheter insertion site may be weaker for the first few hours after catheterization. It should gradually increase in strength. Because a weaker pulse is an expected finding, the nurse should document this and continue to monitor. The insertion site is kept dry.

The nurse is assessing the client diagnosed with congestive heart failure. Which laboratory data would indicate that the client is in severe congestive heart failure? 1. An elevated B-type natriuretic peptide (BNP).

BNP is a specific diagnostic test. Levels higher than normal indicate congestive heart failure, with the higher the number, the more severe the CHF.

Which potential complication should the nurse assess for in the client with infective endocarditis who has embolization of vegetative lesions from the mitral valve? 2. Cerebrovascular accident.

Bacteria enter the bloodstream from invasive procedures and sterile platelet-fibrin vegetation forms on heart valves. The mitral valve is on the left side of the heart and, if the vegetation breaks off, it will go through the left ventricle into the systemic circulation and may lodge in the brain, kidneys, or peripheral tissues.

The nurse knows the client understands the teaching concerning a low-fat, lowcholesterol diet when the client selects which meal? 3. Baked chicken, baked potato, and skim milk.

Baked, broiled, or grilled meats are recommended; a plain baked potato is appropriate; and skim milk is low in fat—so this meal is appropriate for a low-fat, lowcholesterol diet.

The nurse is caring for a client who is diagnosed with aortic stenosis, but is reluctant to have surgery for valve replacement. The nurse is present when the physician talks to the client about a treatment that is less invasive than surgery, but that will likely relieve some of the client's symptoms. Later, the client asks the nurse to review with her spouse the treatment offered. What does the nurse talk about with the spouse?

Balloon percutaneous valvuloplasty

When admitting a patient with a non-ST-segment-elevation myocardial infarction (NSTEMI) to the intensive care unit, which action should the nurse perform first? b. Attach the cardiac monitor.

Because dysrhythmias are the most common complication of myocardial infarction (MI), the first action should be to place the patient on a cardiac monitor

When developing a teaching plan for a 61-year-old man with the following risk factors for coronary artery disease (CAD), the nurse should focus on the. d. elevation of the patient's low-density lipoprotein (LDL) level.

Because family history, gender, and age are nonmodifiable risk factors, the nurse should focus on the patient's LDL level.

To improve the physical activity level for a mildly obese 71-year-old patient, which action should the nurse plan to take? b. Determine what kind of physical activities the patient usually enjoys.

Because patients are more likely to continue physical activities that they already enjoy, the nurse will plan to ask the patient about preferred activities.

Nadolol (Corgard) is prescribed for a patient with chronic stable angina and left ventricular dysfunction. To determine whether the drug is effective, the nurse will monitor for d. the ability to do daily activities without chest pain.

Because the medication is ordered to improve the patient's angina, effectiveness is indicated if the patient is able to accomplish daily activities without chest pain.

Which action by the nurse will determine if the therapies ordered for a patient with chronic constrictive pericarditis are effective? c. Assess for the presence of jugular venous distention (JVD).

Because the most common finding on physical examination for a patient with chronic constrictive pericarditis is jugular venous distention, a decrease in JVD indicates improvement.

The nurse is assessing the client diagnosed with congestive heart failure. Which signs/ symptoms would indicate that the medical treatment has been effective? 3. The client is able to perform activities of daily living without dyspnea.

Being able to perform activities of daily living (ADLs) without shortness of breath (dyspnea) would indicate the client's condition is improving. The client's heart is a more effective pump and can oxygenate the body better without increasing fluid in the lungs.

The nurse, reviewing a patient's laboratory results, recognizes what result as most indicative of myocardial infarction?

Biochemical markers such as creatine phosphokinase (CPK), CPK-MB, and troponin are released specifically by myocardial cells when injured and are detectable in the blood. Myoglobin, although one of the first markers to increase after a myocardial infarction (MI), does not have as high of a cardiac specificity as others. C-reactive protein is increased after an MI as a result of the inflammation caused by tissue damage; however it is also not as highly specific to cardiac tissue. An increased white blood cell count may be present after an MI but is due to a generalized inflammatory response.

The nurse is teaching a class on valve replacements. Which statement identifies a disadvantage of having a biologic tissue valve replacement? 4. The valve has to be replaced frequently.

Biologic valves deteriorate and need to be replaced frequently; this is a disadvantage of them. Mechanical valves do not deteriorate and do not have to be replaced often.

A nurse is caring for a patient who had an aortic balloon valvuloplasty. The nurse would inspect the surgical insertion site closely for which of the following complications?

Bleeding and infection

The client with infective endocarditis is admitted to the medical department. Which health-care provider's order should be implemented first?. 2. Obtain blood cultures times two (2).

Blood cultures must be done before administering antibiotics so that an adequate number of organisms can be obtained to culture and identify.

Abnormally slow heart rate

Bradydysrhythmias

The nurse recognizes which of the following lab tests is a key diagnostic indicator of heart failure?

Brain natriuretic peptide (BNP)

2. A patient with chronic heart failure who has been following a low-sodium diet tells the nurse at the clinic about a 5-pound weight gain in the last 3 days. The nurse's first action will be to a. ask the patient to recall the dietary intake for the last 3 days because there may be hidden sources of sodium in the patient's diet. b. instruct the patient in a low-calorie, low-fat diet because the weight gain has likely been caused by excessive intake of inappropriate foods. c. assess the patient for clinical manifestations of acute heart failure because an exacerbation of the chronic heart failure may be occurring. d. educate the patient about the use of diuretic therapy because it is likely that the patient will need medications to reduce the hypervolemia.

C Rationale: The 5-pound weight gain over 3 days indicates that the patient's chronic heart failure may be worsening; it is important that the patient be immediately assessed for other clinical manifestations of decompensation, such as lung crackles. A dietary recall to detect hidden sodium in the diet and teaching about diuretic therapy are appropriate interventions but are not the first nursing actions indicated. There is no evidence that the patient's weight gain is caused by excessive dietary intake of fat or calories, so the answer beginning "instruct the patient in a low-calorie, low-fat diet" describes an inappropriate action. Cognitive Level: Application Text Reference: p. 826 Nursing Process: Assessment NCLEX: Physiological Integrity

20. An elderly patient with a 40-pack-year history of smoking and a recent myocardial infarction is admitted to the medical unit with acute shortness of breath; the nurse need to rule out pneumonia versus heart failure. The diagnostic test that the nurse will monitor to help in determining whether the patient has heart failure is a. 12-lead electrocardiogram (ECG). b. arterial blood gases (ABGs). c. B-type natriuretic peptide (BNP). d. serum creatine kinase (CK).

C Rationale: BNP is secreted when ventricular pressures increase, as with heart failure, and elevated BNP indicates a probable or very probable diagnosis of heart failure. 12-lead ECGs, ABGs, and CK may also be used in determining the causes or effects of heart failure but are not as clearly diagnostic of heart failure as BNP. Cognitive Level: Application Text Reference: p. 827 Nursing Process: Assessment NCLEX: Physiological Integrity

16. A home health care patient has recently started taking oral digoxin (Lanoxin) and furosemide (Lasix) for control of heart failure. The patient data that will require the most immediate action by the nurse is if the patient's a. weight increases from 120 pounds to 122 pounds over 3 days. b. liver is palpable 2 cm below the ribs on the right side. c. serum potassium level is 3.0 mEq/L after 1 week of therapy. d. has 1 to 2+ edema in the feet and ankles in the morning.

C Rationale: Hypokalemia potentiates the actions of digoxin and increases the risk for digoxin toxicity, which can cause life-threatening dysrhythmias. The other data indicate that the patient's heart failure requires more effective therapies, but they do not require nursing action as rapidly as the low serum potassium level. Cognitive Level: Application Text Reference: pp. 832-833 Nursing Process: Assessment NCLEX: Physiological Integrity

3. During assessment of a 72-year-old with ankle swelling, the nurse notes jugular venous distention (JVD) with the head of the patient's bed elevated 45 degrees. The nurse knows this finding indicates a. decreased fluid volume. b. incompetent jugular vein valves. c. elevated right atrial pressure. d. jugular vein atherosclerosis.

C Rationale: The jugular veins empty into the superior vena cava and then into the right atrium, so JVD with the patient sitting at a 45-degree angle reflects elevated right atrial pressure. JVD is an indicator of excessive fluid volume (increased preload), not decreased fluid volume; it is not caused by incompetent jugular vein valves or atherosclerosis. Cognitive Level: Comprehension Text Reference: p. 825 Nursing Process: Assessment NCLEX: Physiological Integrity

9. During a visit to an elderly patient with chronic heart failure, the home care nurse finds that the patient has severe dependent edema and that the legs appear to be weeping serous fluid. Based on these data, the best nursing diagnosis for the patient is a. activity intolerance related to venous congestion. b. disturbed body image related to massive leg swelling. c. impaired skin integrity related to peripheral edema. d. impaired gas exchange related to chronic heart failure.

C Rationale: The patient's findings of severe dependent edema and weeping serous fluid from the legs support the nursing diagnosis of impaired skin integrity. There is less evidence for the nursing diagnoses of activity intolerance, disturbed body image, and impaired gas exchange, although the nurse will further assess the patient to determine whether there are other clinical manifestations of heart failure to indicate that these diagnoses are appropriate. Cognitive Level: Application Text Reference: p. 836 Nursing Process: Diagnosis NCLEX: Physiological Integrity

A diagnosis of rheumatic fever is being ruled out for a child. Which lab test(s) is/are the most reliable? (Select all that apply.) A. Throat culture B. C-reactive protein (CRP) C. Antistreptolysin-O titer (ASO) titer D. Elevated white blood cell count (WBC) E. Erythrocyte sedimentation rate (ESR)

C. Antistreptolysin-O titer (ASO) titer The most reliable and best standardized lab for antistreptococcal antibodies is an Antistreptolysin-O (ASO) titer. A throat culture indicates a current streptococcal infection. C-reactive protein (CRP) laboratory test indicates inflammation. An elevated white blood cell (WBC) may indicate a possible infection but does not indicate a causative agent. An erythrocyte sedimentation rate (ESR) indicates inflammation.

An important nursing responsibility when a dysrhythmia is suspected is to: A. Order an immediate electrocardiogram. B. Count the radial rate at 1-minute intervals 5 times in a row. C. Count the apical rate for 1 full minute and compare it with the radial rate. D. Have someone else take the radial rate while the nurse simultaneously checks the apical rate.

C. Count the apical rate for 1 full minute and compare it with the radial rate. This may be indicated after conferring with the practitioner. The radial pulse needs to be compared with the apical. This is the nurse's first action. If an arrhythmia is occurring, the radial pulse may be lower than the apical rate. It is the responsibility of the nurse to check both rates.

When assessing for hypertension in an infant, the nurse will expect the infant to exhibit which signs? (Select all that apply.) A. Dizziness B. Changes in vision C. Irritability D. Head rubbing E. Waking up screaming in the night

C. Irritability D. Head rubbing E. Waking up screaming in the night Clinical manifestations of hypertension are: For adolescents and older children: · Frequent headaches · Dizziness · Changes in vision For infants or young children: · Irritability · Head banging or head rubbing · Waking up screaming in the night

Nursing care of the infant or child with congestive heart failure would include: A. Forcing fluids appropriate to age. B. Monitoring respirations during active periods. C. Organizing activities to allow for uninterrupted sleep. D. Giving larger feedings less often to conserve energy.

C. Organizing activities to allow for uninterrupted sleep. The child who has congestive heart failure has an excess of fluid. Monitoring vital signs is appropriate, but minimizing energy expenditure is a priority. The child needs to be well rested before feeding. The child's needs should be met as quickly as possible to minimize crying. The nurse must organize care to facilitate a decrease in his or her energy expenditure. The child often cannot tolerate larger feedings.

The client who has had a myocardial infarction is admitted to the telemetry unit from intensive care. Which referral would be most appropriate for the client?

CARDIAC REHABILITATION

Which of the following lab values would be seen in the patient diagnosed with infective endocarditis? Select all that apply

CRP, WBC, Nonstemi

The client shows ventricular fibrillation on the telemetry at the nurse's station. Which action should the telemetry nurse implement first?

Call a STAT code. - treatment choice defibrillator but not action

A patient in severe pulmonary edema is being intubated by the respiratory therapist. What priority action by the nurse will assist in the confirmation of tube placement in the proper position in the trachea?

Call for a chest x-ray.

The nurse hears the alarm sound on the telemetry monitor and observes a flat line. The patient is found unresponsive, without a pulse, and no respiratory effort. What is the first action by the nurse?

Call for help and begin chest compressions.

When evaluating a patient's knowledge regarding a low-sodium, low-fat cardiac diet, the nurse recognizes additional teaching is needed when the patient selects which food choice? d) Canned chicken noodle soup

Canned soups are very high in sodium content.

When evaluating a patient's knowledge regarding a low-sodium, low-fat cardiac diet, the nurse recognizes additional teaching is needed when the patient selects which food choice?

Canned soups are very high in sodium content. Patients need to be taught to read food labels for sodium and fat content. Baked flounder, angel food cake, and baked potato with margarine are all low-sodium and low-fat and would be appropriate for this diet.

Procedure in which a radiopaque catheter is inserted through a peripheral blood vessel into the heart; contrast material injected; films taken of the dilution and circulation of the contrast material; used for diagnosis, treatment of valves/vessels, and electrophysiology studies

Cardiac Catheterization

The nurse is providing care to a patient with chronic stable angina that is scheduled for a cardiac catheterization. What finding associated with myocardial ischemia could be obtained by this diagnostic procedure?

Cardiac catheterization is an invasive diagnostic procedure to find out the location and severity of blockages in the coronary circulation. ST segment depression is an important diagnostic finding for the presence of myocardial ischemia, which is obtained by electrocardiography (ECG). Cardiac enlargement is a sign of heart failure that can be seen on an x-ray. Echocardiography is used to detect the presence of abnormal wall motion due to myocardial ischemia.

The client is diagnosed with a myocardial infarction. Which referral would be most appropriate for the client? 3. Cardiac rehabilitation.

Cardiac rehabilitation is the most appropriate referral. The client can start rehabilitation in the hospital and then attend an outpatient cardiac rehabilitation, which includes progressive exercise, diet teaching, and classes on modifying risk factors.

The nurse is caring for a patient diagnosed with pericarditis. What serious complication should this patient be monitored for?

Cardiac tamponade

A 55-year-old female patient develops acute pericarditis after a myocardial infarction. It is most important for the nurse to assess for which clinical manifestation of a possible complication? D. Decreased blood pressure with tachycardia

Cardiac tamponade is a serious complication of acute pericarditis. Signs and symptoms indicating cardiac tamponade include narrowed pulse pressure, tachypnea, tachycardia, a decreased cardiac output, and decreased blood pressure.

Refers to abnormalities of the myocardium in which the cardiac muscles' ability to contract is impaired; relatively rare in children.

Cardiomyopathy

Which of the following nursing interventions should a nurse perform when a patient with cardiomyopathy receives a diuretic?

Check for dependent edema regularly

When evaluating a patient suspected of having pericarditis, the nurse documents the description of which indicator that is considered the most characteristic symptom?

Chest pain

The client is diagnosed with pericarditis. Which are the most common signs/symptoms the nurse would expect to find when assessing the client? 4. Increased chest pain with inspiration.

Chest pain is the most common symptom of pericarditis, usually has an abrupt onset, and is aggravated by respiratory movements (deep inspiration, coughing), changes in body position, and swallowing.

Which assessment data collected by the nurse who is admitting a patient with chest pain suggest that the pain is caused by an acute myocardial infarction (AMI)? b. The pain has lasted longer than 30 minutes.

Chest pain that lasts for 20 minutes or more is characteristic of AMI.

The nurse is caring for a 78-year-old patient with aortic stenosis. Which assessment data obtained by the nurse would be most important to report to the health care provider? a. The patient complains of chest pressure when ambulating.

Chest pressure (or pain) occurring with aortic stenosis is caused by cardiac ischemia, and reporting this information would be a priority

Which information given by a patient admitted with chronic stable angina will help the nurse confirm this diagnosis? d. The patient states that the pain "goes away" with one sublingual nitroglycerin tablet.

Chronic stable angina is typically relieved by rest or nitroglycerin administration.

Which assessment data would the nurse expect to find in the client diagnosed with chronic venous insufficiency? Brown discolored skin

Chronic venous insufficiency leads to chronic edema that, in turn, causes a brownish pigmentation to the skin.

Which client would the nurse suspect of having a mitral valve prolapse? 2. A 23-year-old male with Marfan syndrome.

Clients with Marfan syndrome have lifethreatening cardiovascular problems, including mitral valve prolapse,

A thickening and flattening of the tips of the fingers and toes; thought to be a result of chronic tissue hypoxemia and polycythemia.

Clubbing

While doing an admission assessment, the nurse notes clubbing of the patient's fingers. Based on this finding, the nurse will question the patient about which disease process? A. Endocarditis

Clubbing of the fingers is a loss of the normal angle between the base of the nail and the skin. This finding can be found in endocarditis, congenital defects, and/or prolonged oxygen deficiency.

The nurse is teaching the client diagnosed with arterial occlusive disease. Which interventions should the nurse include in the teaching? Select all that apply. 1. Wash legs and feet daily in warm water. 2. Apply moisturizing cream to feet. 4. Do not wear any type of knee stocking. 5. Wear clean white cotton socks.

Cold water causes vasoconstriction and hot water may burn the client's feet; therefore, warm tepid water should be recommended. 2. Moisturizing prevents drying of the feet. 4. This will further decrease circulation to the legs. 5. Colored socks have dye and dirty socks may cause foot irritation that may lead to breaks in the skin.

The nurse is discussing the importance of exercising with a client who is diagnosed with CAD. Which statement best describes the scientific rationale for encouraging 30 minutes of walking daily to help prevent complications of atherosclerosis? 1. Exercise promotes the development of collateral circulation.

Collateral circulation is the development of blood supply around narrowed arteries; it helps prevent complications of atherosclerosis, including myocardial infarction, cerebrovascular accidents, and peripheral vascular disease. Exercise promotes the development of collateral circulation.

A nurse is teaching a patient about an upcoming surgery to separate fused cardiac leaflets. Which of the following is the correct term used to describe this surgery?

Commissurotomy

Vital organ function is maintained by intrinsic compensatory mechanism; blood flow is usually normal or increased, but generally uneven or maldistributed in the microcirculation

Compensated shock

D. Problematic because children with acyanotic heart defects may develop cyanosis. The classification does not reflect the path of blood flow within the heart. Children with cyanosis may be easily identified, but that does not help with the diagnosis. Cyanosis is present when children have defects in which oxygenated blood and unoxygenated blood are mixed. This classification is problematic. Children with traditionally named acyanotic defects may be cyanotic, and children with traditionally classified cyanotic defects may appear pink.

Congenital heart defects have traditionally been divided into acyanotic or cyanotic defects. The nurse should recognize that in clinical practice this system is: A. Helpful because it explains the hemodynamics involved. B. Helpful because children with cyanotic defects are easily identified. C. Problematic because cyanosis is rarely present in children. D. Problematic because children with acyanotic heart defects may develop cyanosis.

Includes primarily anatomic abnormalities present at birth that result in abnormal cardiac function, the consequences of which are hypoxemia and heart failure.

Congenital heart disease

The inability of the heart to pump an adequate amount of blood to meet the metabolic demands of the body; not a disease; in children, most common in infants; usually secondary to increases in blood volume and pressure from anomalies; result of an excessive workload imposed on normal myocardium.

Congestive heart failure

Which information about a patient who has been receiving thrombolytic therapy for an acute myocardial infarction (AMI) is most important for the nurse to communicate to the health care provider? a. No change in the patient's chest pain

Continued chest pain suggests that the thrombolytic therapy is not effective and that other interventions such as percutaneous coronary intervention (PCI) may be needed.

The term for a diagnostic test that involves injection of a contrast media into the venous system through a dorsal vein in the foot is which of the following?

Contrast phlebography

Which of the following is a diagnostic test that involves injection of a contrast media into the venous system through a dorsal vein in the foot?

Contrast phlebography

The nurse is caring for a 64-year-old patient admitted with mitral valve regurgitation. Which information obtained by the nurse when assessing the patient should be communicated to the health care provider immediately? a. The patient has bilateral crackles.

Crackles that are audible throughout the lungs indicate that the patient is experiencing severe left ventricular failure with pulmonary congestion and needs immediate interventions such as diuretics.

The nurse will plan discharge teaching about the need for prophylactic antibiotics when having dental procedures for which patient?. c. Patient who had a mitral valve replacement with a mechanical valve.

Current American Heart Association guidelines recommend the use of prophylactic antibiotics before dental procedures for patients with prosthetic valves to prevent infective endocarditis (IE).

A blue discoloration in the mucous membranes, skin, and nail beds of the child with reduced oxygen saturation; results from the presence of deoxygenated hemoglobin (hemoglobin not bound to oxygen); determined subjectively.

Cyanosis

17. Following an acute myocardial infarction, a previously healthy 67-year-old patient develops clinical manifestations of heart failure. The nurse anticipates discharge teaching will include information about a. digitalis preparations, such as digoxin (Lanoxin). b. calcium-channel blockers, such as diltiazem (Cardizem). c. -adrenergic agonists, such as dobutamine (Dobutrex). d. angiotensin-converting enzyme (ACE) inhibitors, such as captopril (Capoten).

D Rationale: ACE-inhibitor therapy is currently recommended to prevent the development of heart failure in patients who have had a myocardial infarction and as a first-line therapy for patients with chronic heart failure. Digoxin therapy for heart failure is no longer considered a first-line measure, and digoxin is added to the treatment protocol when therapy with other medications such as ACE-inhibitors, diuretics, and -adrenergic blockers is insufficient. Calcium-channel blockers are not generally used in the treatment of heart failure. The -adrenergic agonists such as dobutamine are administered through the IV route and are not used as initial therapy for heart failure. Cognitive Level: Application Text Reference: p. 832 Nursing Process: Implementation NCLEX: Physiological Integrity

4. The nurse is caring for a patient receiving IV furosemide (Lasix) 40 mg and enalapril (Vasotec) 5 mg PO bid for ADHF with severe orthopnea. When evaluating the patient response to the medications, the best indicator that the treatment has been effective is a. weight loss of 2 pounds overnight. b. improvement in hourly urinary output. c. reduction in systolic BP. d. decreased dyspnea with the head of the bed at 30 degrees.

D Rationale: Because the patient's major clinical manifestation of ADHF is orthopnea (caused by the presence of fluid in the alveoli), the best indicator that the medications are effective is a decrease in crackles. The other assessment data also may indicate that diuresis or improvement in cardiac output have occurred but are not as useful in evaluating this patient's response. Cognitive Level: Application Text Reference: p. 825 Nursing Process: Evaluation NCLEX: Physiological Integrity

23. A hospitalized patient with heart failure has a new order for captopril (Capoten) 12.5 mg PO. After administering the first dose and teaching the patient about captopril, which statement by the patient indicates that teaching has been effective? a. "I will need to include more high-potassium foods in my diet." b. "I will expect to feel more short of breath for the next few days." c. "I will be sure to take the medication after eating something." d. "I will call for help when I need to get up to the bathroom."

D Rationale: Captopril can cause hypotension, especially after the initial dose, so it is important that the patient not get up out of bed without assistance until the nurse has had a chance to evaluate the effect of the first dose. The ACE inhibitors are potassium sparring, and the nurse should not teach the patient to increase sources of dietary potassium. Increased shortness of breath is expected with initiation of -blocker therapy for heart failure, not for ACE-inhibitor therapy. ACE inhibitors are best absorbed when taken an hour before eating. Cognitive Level: Application Text Reference: p. 832 Nursing Process: Evaluation NCLEX: Physiological Integrity

13. The nurse plans discharge teaching for a patient with chronic heart failure who has prescriptions for digoxin (Lanoxin), hydrochlorothiazide (HydroDIURIL), and a potassium supplement. Appropriate instructions for the patient include a. avoid dietary sources of potassium because too much can cause digitalis toxicity. b. take the pulse rate daily and never take digoxin if the pulse is below 60 beats/min. c. take the hydrochlorothiazide before bedtime to maximize activity level during the day. d. notify the health care provider immediately if nausea or difficulty breathing occurs.

D Rationale: Difficulty breathing is an indication of acute decompensated heart failure and suggests that the medications are not achieving the desired effect. Nausea is an indication of digoxin toxicity and should be reported so that the provider can assess the patient for toxicity and adjust the digoxin dose, if necessary. Digoxin toxicity is potentiated by hypokalemia, rather than hyperkalemia. Patients should be taught to check their pulse daily before taking the digoxin and, if the pulse is less than 60, to call their provider before taking the digoxin. Diuretics should be taken early in the day to avoid sleep disruption. Cognitive Level: Application Text Reference: p. 835 Nursing Process: Implementation NCLEX: Health Promotion and Maintenance

18. A 55-year-old patient with inoperable coronary artery disease and end-stage heart failure asks the nurse whether heart transplant is a possible therapy. The nurse's response to the patient will be based on the knowledge that a. heart transplants are experimental surgeries that are not covered by most insurance. b. the patient is too old to be placed on the transplant list. c. the diagnoses and symptoms indicate that the patient is not an appropriate candidate. d. candidacy for heart transplant depends on many factors.

D Rationale: Indications for a heart transplant include inoperable coronary artery disease and refractory end-stage heart failure, but other factors such as coping skills, family support, and patient motivation to follow the rigorous post-transplant regimen are also considered. Heart transplants are not considered experimental; rather, transplantation has become the treatment of choice for patients who meet the criteria. The patient is not too old for a transplant. The patient's diagnoses and symptoms indicate that the patient may be an appropriate candidate for a heart transplant. Cognitive Level: Comprehension Text Reference: p. 837 Nursing Process: Planning NCLEX: Health Promotion and Maintenance

12. When teaching the patient with heart failure about a 2000-mg sodium diet, the nurse explains that foods to be restricted include a. eggs and other high-cholesterol foods. b. canned and frozen fruits. c. fresh or frozen vegetables. d. milk, yogurt, and other milk products.

D Rationale: Milk and yogurt naturally contain a significant amount of sodium, and intake of these should be limited for patients on a diet that limits sodium to 2000 mg daily. Other milk products, such as processed cheeses, have very high levels of sodium and are not appropriate for a 2000-mg sodium diet. The other foods listed have minimal levels of sodium and can be eaten without restriction. Cognitive Level: Application Text Reference: p. 833 Nursing Process: Implementation NCLEX: Health Promotion and Maintenance

What should the nurse recognize as an early clinical sign of compensated shock in a child? A. Confusion B. Sleepiness C. Hypotension D. Apprehension

D. Apprehension Confusion indicates uncompensated shock. Sleepiness is not an indication of shock. Hypotension is a symptom of irreversible shock. Apprehension indicates compensated shock.

What should the nurse recognize as an early clinical sign of compensated shock in a child? A. Confusion B. Sleepiness C. Hypotension D. Apprehension

D. Apprehension Confusion indicates uncompensated shock. Sleepiness is not an indication of shock. Hypotension is a symptom of irreversible shock. Apprehension indicates compensated shock.

Nurses counseling parents regarding the home care of the child with a cardiac defect before corrective surgery should stress the: A. Importance of reducing caloric intake to decrease cardiac demands. B. Importance of relaxing discipline and limit-setting to prevent crying. C. Need to be extremely concerned about cyanotic spells. D. Desirability of promoting normalcy within the limits of the child's condition.

D. Desirability of promoting normalcy within the limits of the child's condition. Child needs increased caloric intake. Child needs discipline and appropriate limits. Because cyanotic spells occur in children with some defects, the parents need to be taught how to manage these. The child needs to have social interactions, discipline, and appropriate limit-setting. Parents need to be encouraged to promote as normal a life as possible for their child.

The school nurse is called to the cafeteria because a child "has eaten something he is allergic to." The child is in severe respiratory distress. FIRST the nurse should: A. Determine what the child has eaten. B. Administer diphenhydramine (Benadryl). C. Move the child to the nurse's office or hallway. D. Have someone call for an ambulance/paramedic rescue squad.

D. Have someone call for an ambulance/paramedic rescue squad. Because severe respiratory distress is occurring, treatment of the response is indicated first. The cause of the response can be determined later. Diphenhydramine will not be effective for this type of allergic reaction. The child should not be moved unless the child is in a place that puts the child at greater hazard. Because the child is in severe respiratory distress, the nurse should remain with the child while someone else calls for the rescue squad.

The doctor suggests that surgery be performed for patent ductus arteriosus (PDA) to prevent: A. Pulmonary infection. B. Right-to-left shunt of blood. C. Decreased workload on left side of heart. D. Increased pulmonary vascular congestion.

D. Increased pulmonary vascular congestion. The increased pulmonary vascular congestion is the primary complication. The shunt of blood is left to right. The increased pulmonary vascular congestion is the primary complication. A PDA allows blood to flow from the aorta (high pressure) to the pulmonary artery (low pressure). If the PDA stays open, increased pulmonary congestion can occur.

Congenital heart defects have traditionally been divided into acyanotic or cyanotic defects. The nurse should recognize that in clinical practice this system is: A. Helpful because it explains the hemodynamics involved. B. Helpful because children with cyanotic defects are easily identified. C. Problematic because cyanosis is rarely present in children. D. Problematic because children with acyanotic heart defects may develop cyanosis.

D. Problematic because children with acyanotic heart defects may develop cyanosis. The classification does not reflect the path of blood flow within the heart. Children with cyanosis may be easily identified, but that does not help with the diagnosis. Cyanosis is present when children have defects in which oxygenated blood and unoxygenated blood are mixed. This classification is problematic. Children with traditionally named acyanotic defects may be cyanotic, and children with traditionally classified cyanotic defects may appear pink.

A young child with tetralogy of Fallot may assume a posturing position as a compensatory mechanism. The position automatically assumed by the child is: A. Low Fowler's. B. Prone. C. Supine. D. Squatting.

D. Squatting. Low Fowler's would assist with respiratory issues but would not assist with the need for cardiac compensation. Prone does not offer any advantage to the child. Supine does not offer any advantage to the child. The squatting or knee-chest position decreases the amount of blood returning to the heart and allows the child time to compensate.

Which is considered a mixed cardiac defect? A. Pulmonic stenosis B. Atrial septal defect C. Patent ductus arteriosus D. Transposition of the great arteries

D. Transposition of the great arteries Pulmonic stenosis is classified as an obstructive defect. Atrial septal defect is classified as a defect with increased pulmonary blood flow. Patent ductus arteriosus is classified as a defect with increased pulmonary blood flow. Transposition of the great arteries allows the mixing of blood in the heart.

The primary therapy for secondary hypertension in children is: A. Weight reduction. B. Low-salt diet. C. Increased exercise and fitness. D. Treatment of underlying cause.

D. Treatment of underlying cause. These therapies are usually effective for essential hypertension. These therapies are usually effective for essential hypertension. These therapies are usually effective for essential hypertension. Secondary hypertension is a result of an underlying disease process or structural abnormality. It is usually necessary to treat the problem before the hypertension will be resolved.

Which of the following body system responses correlates with systolic heart failure (HF)?

Decrease in renal perfusion

A client seeks medical attention for dyspnea, chest pain, syncope, fatigue, and palpitations. A thorough physical examination reveals an apical systolic thrill and heave, along with a fourth heart sound (S4) and a systolic murmur. Diagnostic tests reveal that the client has hypertrophic cardiomyopathy (HCM). Which nursing diagnosis may be appropriate?

Decreased Cardiac Output

During assessment of a client admitted for cardiomyopathy, the nurse notes the following symptoms: dyspnea on exertion, fatigue, fluid retention, and nausea. The initial appropriate nursing diagnosis is which of the following?

Decreased Cardiac output

For a client with cardiomyopathy, the most important nursing diagnosis is:

Decreased cardiac output related to reduced myocardial contractility.

What is the primary underlying disorder of pulmonary edema?

Decreased left ventricular pumping

The nurse identifies the nursing diagnosis of decreased cardiac output related to valvular insufficiency for the patient with infective endocarditis (IE) based on which assessment finding(s)? b. Urine output less than 30 mL/hr

Decreased renal perfusion caused by inadequate cardiac output will lead to decreased urine output.

The most important reason for a nurse to encourage a client with peripheral vascular disease to initiate a walking program is that this form of exercise:

Decreases venous congestion

Tetralogy of Fallot

Defects with decreased pulmonary blood flow

Tricuspid atresia

Defects with decreased pulmonary blood flow

Atrial septal defect

Defects with increased pulmonary blood flow

Atrioventricular canal defect

Defects with increased pulmonary blood flow

Patent ductus arteriosus

Defects with increased pulmonary blood flow

Ventricular septal defect

Defects with increased pulmonary blood flow

The nurse obtains a health history from a 65-year-old patient with a prosthetic mitral valve who has symptoms of infective endocarditis (IE). Which question by the nurse is most appropriate? "Have you had dental work done recently?

Dental procedures place the patient with a prosthetic mitral valve at risk for infective endocarditis (IE).

20. When caring for a patient who is recovering from a sudden cardiac death (SCD) event and has no evidence of an acute myocardial infarction (AMI), the nurse will anticipate teaching the patient that b. additional diagnostic testing will be required.

Diagnostic testing (e.g., stress test, Holter monitor, electrophysiologic studies, cardiac catheterization) is used to determine the possible cause of the SCD and treatment options. SCD is likely to recur.

Along with persistent, crushing chest pain, which signs/symptoms would make the nurse suspect that the client is experiencing a myocardial infarction?

Diaphoresis and cool clammy skin. - these are systemic reaction

The nurse is caring for a patient diagnosed with myocarditis. The nurse must be aware that patients with myocarditis are sensitive to which of the following medications?

Digoxin

A patient is exhibiting digitalis toxicity. Which of the following medications would the nurse expect to be ordered for this patient?

Digoxin immune FAB (Digibind)

A nurse is caring for a client who was admitted yesterday with myocarditis. The client takes insulin, digoxin, and metformin at home and continues these medications during his hospitalization. During the morning assessment, the client reports anorexia, nausea, vomiting, headache, and disturbed color vision. His ECG strip shows a prolonged PR interval. The nurse suspects which of the following?

Digoxin toxicity

The nurse performing an assessment on a patient who has arterial insufficiency of the legs and an ulcer on the left great toe would expect to find which of the following characteristics?

Diminished or absent pulses

Which of the following are the first symptoms of cardiac tamponade? Select all that apply.

Dizziness, Chest tightness, SOB

The nurse is discussing the importance of exercise with the client diagnosed with coronary artery disease. Which intervention should the nurse implement?

Do not walk outside if it is less than 40˚F. - when it is cold outside, it causes vasoconstriction and lower oxygen to heart muscles - perform isotonic exercises not isometric - walk at least 30 minutes not 15 minutes for 3-4 times - wear supportive shoes not sandals!

The nurse is caring for a client following cardiac valve replacement. Which nursing action is correct when obtaining a Homan's sign to screen for thrombophlebitis?

Dorsiflexion of the foot with calf pain

The nurse would assess a patient with complaints of chest pain for which clinical manifestations associated with a myocardial infarction (MI) (select all that apply)? b) Ashen skin c) Diaphoresis d) Nausea and vomiting e) S3 or S4 heart sounds

During the initial phase of an MI, catecholamines are released from the ischemic myocardial cells, causing increased sympathetic nervous system (SNS) stimulation. This results in the release of glycogen, diaphoresis, and vasoconstriction of peripheral blood vessels. The patient's skin may be ashen, cool, and clammy (not flushed) as a result of this response. Nausea and vomiting may result from reflex stimulation of the vomiting center by severe pain. Ventricular dysfunction resulting from the MI may lead to the presence of the abnormal S3 and S4 heart sounds.

Frequently, what is the earliest symptom of left-sided heart failure?

Dyspnea on exertion

A client is being seen in the clinic to R/O mitral valve stenosis. Which assessment data would be most significant? 1. The client complains of shortness of breath when walking.

Dyspnea on exertion (DOE) is typically the earliest manifestation of mitral valve stenosis.

A client with aortic regurgitation is admitted to the hospital. Which of the following assessment findings would indicate left ventricular failure?

Dyspnea, orthopnea, paroxysmal nocturnal dyspnea (PND)

A client is admitted to the hospital with aortic stenosis. Which of the following assessment findings would indicate the development of left ventricular failure?

Dyspnea, orthopnea, pulmonary edema

The nurse is caring for a patient who was admitted to the coronary care unit following an acute myocardial infarction (AMI) and percutaneous coronary intervention the previous day. Teaching for this patient would include a. when cardiac rehabilitation will begin.

Early after an AMI, the patient will want to know when resumption of usual activities can be expected.

A patient is admitted to the hospital with possible acute pericarditis and pericardial effusion. The nurse knows to prepare the patient for which diagnostic test used to confirm the patient's diagnosis?

Echocardiogram

A patient is admitted with suspected cardiomyopathy. What diagnostic test would be most helpful with the identification of this disorder?

Echocardiogram

A patient is admitted to the hospital with possible acute pericarditis. The nurse should plan to teach the patient about the purpose of a. echocardiography.

Echocardiograms are useful in detecting the presence of the pericardial effusions associated with pericarditis

A client with venous insufficiency asks the nurse what they can do to decrease their risk of complications. What advice should the nurse provide to clients with venous insufficiency?

Elevate the legs periodically for at least 15 to 20 minutes.

The nurse is caring for the client with chronic venous insufficiency. Which statement indicates that the client understands the discharge teaching? 2. "I need to elevate the foot of my bed while sleeping."

Elevating the foot of the bed while sleeping helps the venous blood return to the heart and decreases pressure in the lower extremity.

A patient complaining of heart palpitations is diagnosed with atrial fibrillation caused by mitral valve prolapse. In order to relieve the symptoms, the nurse should teach the patient which of the following dietary interventions?

Eliminate caffeine and alcohol

When caring for a patient with infective endocarditis of the tricuspid valve, the nurse should monitor the patient for the development of c. shortness of breath.

Embolization from the tricuspid valve would cause symptoms of pulmonary embolus.

Which of the following statements is accurate regarding Reynaud's disease?

Episodes may be triggered by unusual sensitivity to cold

The client diagnosed with essential hypertension asks the nurse, "I don't know why the doctor is worried about my blood pressure. I feel just great." Which statement by the nurse would be the most appropriate response? 1. "Damage can be occurring to your heart and kidneys even if you feel great."

Even if the client feels great, the blood pressure can be elevated, causing damage to the heart, kidney, and blood vessels.

The female client is diagnosed with rheumatic fever and prescribed penicillin, an antibiotic. Which statement indicates the client needs more teaching concerning the discharge teaching? 3. "I will have no problems as long as I take my medication."

Even with antibiotic treatment for rheumatic fever, the client may experience bacterial endocarditis in later years and should know this may occur.

Which antilipemic medications should the nurse question for a patient with cirrhosis of the liver (select all that apply)? b) Ezetimibe (Zetia) d) Atorvastatin (Lipitor)

Ezetimibe (Zetia) should not be used by patients with liver impairment. Adverse effects of atorvastatin (Lipitor), a statin drug, include liver damage and myopathy.

Which signs and symptoms accompany a diagnosis of pericarditis?

Fever, chest discomfort, and elevated erythrocyte sedimentation rate (ESR)

Which actions should the surgical scrub nurse take to prevent from personally developing a DVT? 2. Flex the leg muscles and change the leg positions frequently.

Flexing the leg muscles and changing positions assist the blood to return to the heart and move out of the peripheral vessels.

The health-care provider ordered a femoral angiogram for the client diagnosed with arterial occlusive disease. Which intervention should the nurse implement? 4. Inform the client that fluids will be increased after the procedure.

Fluids will help flush the contrast dye out of the body and help prevent kidney damage.

A nurse is assessing a 53-year-old obese female patient who has been postmenopausal for three years. The nurse suspects that the patient may have coronary artery disease and asks the patient to test her lipid profile. What abnormality in the lipid profile would the nurse expect to find?

Following menopause, there is a significant increase in the low-density lipoprotein levels and a consequent reduction in the high-density lipoprotein levels. This occurs due to hormonal changes in the body. This makes postmenopausal women more susceptible to developing coronary artery disease. Obese postmenopausal women may have increased levels of both high-density lipoproteins and low-density lipoproteins (HDLs and LDLs). As the patient is obese, the low-density lipoproteins (LDL) levels would be high and as she is postmenopausal, there is a marked reduction in high-density lipoprotein levels (HDLs). Before menopause, high estrogen levels cause an increase in high-density lipoproteins and lowering of low-density lipoprotein levels.

A male patient who has coronary artery disease (CAD) has serum lipid values of LDL cholesterol 98 mg/dL and HDL cholesterol 47 mg/dL. What should the nurse include in the patient teaching? d) The lipid levels are normal.

For men, the recommended LDL is less than 100 mg/dL, and the recommended level for HDL is greater than 40mg/dL.

A nurse is instructing a client about using antiembolism stockings. Antiembolism stockings help prevent deep vein thrombosis (DVT) by:

Forcing blood into the deep venous system.

You are caring for a client with suspected right-sided heart failure. What would you know that clients with suspected right-sided heart failure may experience?

Gradual unexplained weight gain

The client is employed in a job that requires extensive standing. Which intervention should the nurse include when discussing how to prevent varicose veins? 4. Wear graduated compression hose.

Graduated compression hose help decrease edema and increase the circulation back to the heart; this helps prevent varicose veins.

The nurse is discharging a client diagnosed with DVT from the hospital. Which discharge instructions should be provided to the client? 3. Avoid green leafy vegetables and notify the HCP of red or brown urine.

Green leafy vegetables contain vitamin K, which is the antidote for warfarin. These foods will interfere with the action of warfarin. Red or brown urine may indicate bleeding.

A patient with a prosthetic heart valve is diagnosed with subacute infective endocarditis. The nurse knows that the majority of these infections are caused by:

Group A, beta-hemolytic Streptococcus

A patient is diagnosed with rheumatic endocarditis. What bacterium is the nurse aware causes this inflammatory response?

Group A, beta-hemolytic streptococcus

The nurse is developing a nursing care plan for a client diagnosed with congestive heart failure. A nursing diagnosis of "decreased cardiac output related to inability of the heart to pump effectively" is written. Which short-term goal would be best for the client? The client will:

Have an audible S1 and S2 with no S3 heard by end of shift.

The client diagnosed with rule-out myocardial infarction is experiencing chest pain while walking to the bathroom. Which action should the nurse implement first?

Have the client sit down immediately. - decreases the need for oxygen, may help decrease the chest pain

The nurse obtains a health history from a patient with a prosthetic heart valve and new symptoms of infective endocarditis. Which question by the nurse is most appropriate to ask?

Have you seen the dentist lately?

The nurse has been teaching a patient about ways to decrease risk factors for coronary artery disease (CAD). Which statement by the patient indicates an adequate understanding?

Health-promoting behaviors for those at risk for CAD include: improving physical activity such as brisk walking (three to four miles/hour for at least 30 minutes five or more times a week); reducing total fat and saturated fat intake; stopping all tobacco use, and altering patterns that are conducive to stress.

A nursing student is caring for a client with end-stage cardiomyopathy. The client's spouse asks the student to clarify one of the last treatment options available that the physician mentioned. After checking with the primary nurse, the student would most likely discuss which of the following?

Heart Transplantation

Which of the following therapies are for patient who have advanced heart failure (HF) after all other therapies have failed?

Heart transplant

The nurse and an unlicensed assistive personnel (UAP) are caring for four clients on a telemetry unit. Which nursing task would be best for the nurse to delegate to the UAP?

Help position the client who is having a portable x-ray done. - unstable patient from ICU is not delegated - teaching is not delegated

Heparin is ordered for a patient with a non-ST-segment-elevation myocardial infarction (NSTEMI). What is the purpose of the heparin? c. Heparin prevents the development of new clots in the coronary arteries.

Heparin helps prevent the conversion of fibrinogen to fibrin and decreases coronary artery thrombosis.

A white male, age 43, with a tentative diagnosis of infective endocarditis is admitted to an acute care facility. His medical history reveals diabetes mellitus, hypertension, and pernicious anemia; he underwent an appendectomy 20 years earlier and an aortic valve replacement 2 years before this admission. Which history finding is a major risk factor for infective endocarditis?

History of aortic valve replacement

A client who underwent total hip replacement exhibits a red, painful area on the calf of the affected leg. What test validates presence of thromboembolism?

Homans'

An infant with an unrepaired tetralogy of Fallot defect is becoming extremely cyanotic during a routine blood draw. Which interventions should the nurse implement? Place in order from the highest-priority intervention to the lowest-priority intervention. a. Administer 100% oxygen by blow-by. b. Place infant in knee-chest position. c. Remain calm. d. Give morphine subcutaneously or by an existing intravenous line. 59. First priority 60. Second priority 61. Third priority 62. Fourth priority

Hypercyanotic spells, also referred to as blue spells or tet spells because they are often seen in infants with tetralogy of Fallot, may occur in any child whose heart defect includes obstruction to pulmonary blood flow and communication between the ventricles. The infant becomes acutely cyanotic and hyperpneic because sudden infundibular spasm decreases pulmonary blood flow and increases right-to-left shunting. Because profound hypoxemia causes cerebral hypoxia, hypercyanotic spells require prompt assessment and treatment to prevent brain damage or possibly death. The infant should first be placed in the knee-chest position to reduce blood returning to the heart. Next, 100% oxygen is given to alleviate the hypoxemia. Morphine is next administered to reduce infundibular spasms. Last, the nurse should remain calm.

A general term for excessive lipids (fats) and fatlike substances; believed to play an important role in atherosclerosis development.

Hyperlipidemia

The consistent elevation of blood pressure beyond values considered to be the upper limits of normal.

Hypertension

A nurse reviewing a patient's echocardiogram report reads the following statements: "The heart muscle is asymmetrically thickened and has an increase in overall size and mass, especially along the septum. The ventricular walls are thickened reducing the size of the ventricular cavities. Several areas of the myocardium have evidence of scaring." The nurse knows these manifestations are indicative of which type of cardiomyopathy?

Hypertrophic

In which type of cardiomyopathy does the heart muscle actually increase in size and mass weight, especially along the septum?

Hypertrophic

After receiving report on the following patients, which patient should the nurse assess first? b. Patient with acute aortic regurgitation whose blood pressure is 86/54 mm Hg

Hypotension in patients with acute aortic regurgitation may indicate cardiogenic shock.

Refers to an arterial oxygen tension (or pressure) that is less than normal and can be identified by a decreased arterial saturation or a decreased PaO2.

Hypoxemia

A reduction in tissue oxygenation that results from low oxygen saturation and PaO2 and results in impaired cellular processes.

Hypoxia

A 45-year-old male client with a confirmed DVT is being discharged from the ED. Which of the following client statements indicates that the client has received proper nursing instruction and understands how to manage his condition?

I need to do my leg exercises....

A patient with endocarditis is being discharged home. In evaluating the effectiveness of patient teaching about how to prevent recurrence of the infection, the student nurse would expect the patient to state:

I will ask for antibiotics whenever I have dental work done

Which New York Heart Association classification of heart failure has a poor prognosis and includes symptoms of cardiac insufficiency at rest?

IV

Which intervention should the nurse implement when defibrillating a client who is in ventricular fibrillation? 4. Shout "all clear" prior to defibrillating the client.

If any member of the health-care team is touching the client or the bed during defibrillation, that person could possibly be shocked. Therefore, the nurse should shout "all clear."

A female patient who has type 1 diabetes mellitus has chronic stable angina that is controlled with rest. She states that over the past few months she has required increasing amounts of insulin. What goal should the nurse use to plan care that should help prevent cardiovascular disease progression? d) Keep Hgb A1C less than 7%.

If the Hgb A1C is kept below 7%, this means that the patient has had good control of her blood glucose over the past 3 months.

The telemetry nurse is unable to read the telemetry monitor at the nurse's station. Which intervention should the telemetry nurse implement first?. 3. Contact the client on the client call system.

If the client answers the call light and is not experiencing chest pain, then there is probably a monitor artifact, which is not a life-threatening emergency. After talking with the client, send a nurse to the room to check the monitor.

The client has just returned from a cardiac catheterization. Which assessment data would warrant immediate intervention from the nurse? 3. The client refuses to keep the leg straight.

If the client bends the leg, it could cause the insertion site to bleed. This is arterial blood and the client could bleed to death very quickly, so this requires immediate intervention.

The nurse is caring for a patient who has started anticoagulant therapy with warfarin (Coumadin). When does the nurse understand that therapeutic benefits will begin?

In 3 to 5 days

The patient is being dismissed from the hospital after ACS and will be attending rehabilitation. What information does the patient need to be taught about the early recovery phase of rehabilitation? d) Activity level is gradually increased under cardiac rehabilitation team supervision and with ECG monitoring.

In the early recovery phase after the patient is dismissed from the hospital, the activity level is gradually increased under supervision and with ECG monitoring.

The client who has just had a percutaneous balloon valvuloplasty is in the recovery room. Which intervention should the recovery room nurse implement? 4. Keep the client's affected leg straight

In this invasive procedure, performed in a cardiac catheterization laboratory, the client has a catheter inserted into the femoral artery. Therefore, the client must keep the leg straight to prevent hemorrhaging at the insertion site.

A nurse is developing a nursing care plan for a client with peripheral arterial disease. Which of the following will be the priority nursing diagnosis?

Ineffective peripheral tissue perfusion

A client is diagnosed with deep vein thrombosis (DVT). Which nursing diagnosis should receive highest priority at this time?

Ineffective peripheral tissue perfusion related to venous congestion

A patient seen in the clinic has been diagnosed with stage A heart failure (according to the staging classification of the American College of Cardiology [ACC]). What education will the nurse provide to this patient?

Information about ACE inhibitors and risk factor reduction

The client with coronary artery disease is prescribed a Holter monitor. Which intervention should the nurse implement?

Instruct client to keep a diary of activity, especially when having chest pain. - important, so physician can compare ECG recordings with different level of activity - it should not be removed for any reason - does not affect meds regimen - perform all activities as usual while wearing Holter

A nurse is teaching a patient newly diagnosed with arterial insufficiency. Which of the following terms should the nurse use to refer to leg pain that occurs when the patient is walking?

Intermittent claudication

Which of the following is the hallmark symptom for peripheral arterial disease (PAD) in the lower extremity?

Intermittent claudication

Which of the following terms refers to a muscular, cramplike pain in the extremities consistently reproduced with the same degree of exercise and relieved by rest?

Intermittent claudication

Which assessment data would cause the nurse to suspect the client has atherosclerosis? 3. Intermittent claudication.

Intermittent claudication is a sign of generalized atherosclerosis and is a marker of atherosclerosis.

Condition in which actual damage to vital organs occurs and disruption/death occurs even if homeostasis returns to normal with therapy.

Irreversible shock

One of the most common causes of mitral valve regurgitation in people living in developed countries is

Ischemia of the left ventricle

The nurse identifies which of the following symptoms as a characteristic of right-sided heart failure?

Jugular vein distention (JVD)

A new client has been admitted with right-sided heart failure. The nurse knows to look for which of the following assessment findings when assessing this client?

Jugular venous distention

The client is experiencing multifocal premature ventricular contractions. Which antidysrhythmic medication would the nurse expect the health-care provider to order for this client?

LIDOCAINE - suppresses ventricular ectopy and is the drug of choice for ventricular dysrhythmias ATROPINE - decreases vagal stimulation, for asystole DIGOXIN - slows heart rate, increases contractility, for atrial fibrillation ADENOSINE - choice drug for supraventricular tachycardia

A patient had a non-ST-segment-elevation myocardial infarction (NSTEMI) 3 days ago. Which nursing intervention included in the plan of care is most appropriate for the registered nurse (RN) to delegate to an experienced licensed practical/vocational nurse (LPN/LVN)? d. Reinforcement of teaching about the purpose of prescribed medications

LPN/LVN education and scope of practice include reinforcing education that has previously been done by the RN. Evaluating the patient response to exercise after a NSTEMI requires more education and should be done by the RN.

A client who suffered blunt chest trauma in a motor vehicle accident complains of chest pain, which is exacerbated by deep inspiration. On auscultation, the nurse detects a pericardial friction rub — a classic sign of acute pericarditis. The physician confirms acute pericarditis and begins appropriate medical intervention. To relieve chest pain associated with pericarditis, which position should the nurse encourage the client to assume?

Leaning forward while sitting

A patient was diagnosed with mitral regurgitation. The nurse is aware that the patient should be observed for indicators of left- and right-sided heart failure. This understanding is based on the mechanics of cardiac hemodynamics. With mitral regurgitation, due to thickened valve leaflets, blood flows backward from the:

Left ventricle into the left atrium during systole

A nursing student is caring for a client with end-stage cardiomyopathy. The client's spouse asks the nurse to clarify one of the last treatment options available that the physician mentioned earlier. After checking with the primary nurse, the nursing student would most likely discuss which of the following?

Left ventricular assist device

For patients diagnosed with aortic stenosis, digoxin would be ordered for which of the following clinical manifestations?

Left ventricular dysfunction

The client is experiencing multifocal premature ventricular contractions. Which antidysrhythmic medication would the nurse expect the health-care provider to order for this client? 1. Lidocaine.

Lidocaine suppresses ventricular ectopy and is the drug of choice for ventricular dysrhythmias.

Which nursing intervention will be most effective when assisting the patient with coronary artery disease (CAD) to make appropriate dietary changes? c. Help the patient modify favorite high-fat recipes by using monosaturated oils when possible.

Lifestyle changes are more likely to be successful when consideration is given to the patient's values and preferences.

A 21-year-old woman is scheduled for percutaneous transluminal balloon valvuloplasty to treat mitral stenosis. Which information should the nurse include when explaining the advantages of valvuloplasty over valve replacement to the patient? c. Lifelong anticoagulant therapy will be needed after mechanical valve replacement.

Long-term anticoagulation therapy is needed after mechanical valve replacement, and this would restrict decisions about career and childbearing in this patient

Which intervention should the nurse implement when administering a loop diuretic to a client diagnosed with coronary artery disease? 2. Assess the client's serum potassium level.

Loop diuretics cause potassium to be lost in the urine output. Therefore, the nurse should assess the client's potassium level, and if the client is hypokalemic, the nurse should question administering this medication.

The client is admitted for surgical repair of an 8-cm abdominal aortic aneurysm. Which sign/symptom would make the nurse suspect the client has an expanding AAA? 1. Complaints of low back pain.

Low back pain is present because of the pressure of the aneurysm on the lumbar nerves; this is a serious symptom, usually indicating that the aneurysm is expanding rapidly and about to rupture.

A 52-year-old male client in the hospital unit where you practice nursing is being treated for myocarditis. Which of the following nursing interventions should you perform to reduce cardiac workload in a client with myocarditis?

Maintain the client on bedrest

Which of the following nursing interventions should a nurse perform to reduce cardiac workload in a patient diagnosed with myocarditis?

Maintain the patient on bedrest

Which statement by a patient diagnosed with stable angina indicates understanding of the disease process?

Mismatch between oxygen demand of cardiac muscles and supply of oxygen leads to myocardial ischemia that is represented by pain. Decreased oxygenation level of blood indicates respiratory problems. Myocardial ischemia is completely reversible. The patient will start to experience symptoms of ischemia when the coronary artery is blocked by 75% or more.

A 6-year-old female client is admitted to the pediatrics unit due to suspected rheumatic fever. Aggressive antibiotic therapy and comfort measures have been instituted to minimize the long-lasting effects of the systemic inflammation. If the client were to develop rheumatic carditis, which cardiac structure would most likely be affected?

Mitral Valve.

Hypoplastic left heart syndrome

Mixed defects

Transposition of the great vessels

Mixed defects

Truncus arteriosus

Mixed defects

The client diagnosed with congestive heart failure is complaining of leg cramps at night. Which nursing interventions should be implemented?

Monitor the client's potassium level and assess the client's intake of bananas and orange juice. - most probable cause of the pain is hypokalemia - peripheral edema may cause calf tightness not cramping

A client develops cardiogenic pulmonary edema and is extremely apprehensive. What medication can the nurse administer with physician orders that will relieve anxiety and slow respiratory rate?

Morphine sulfate

Which patient would the nurse identify being at higher risk for developing coronary artery disease (CAD)?

Multiple risk factors increase the risk of CAD, and this patient has three risk factors: age over 55, African-American ethnic background, and cholesterol level greater than 240 mg/dL. Middle-aged nonsmoking African-American male has only two risk factors: middle age and male gender: Caucasian middle-age males are more prone to develop CAD. The Hispanic patient has only two risk factors for CAD: male gender and smoking. Middle-aged white male has two risk factors only (age and gender) because systolic blood pressure is less than 160 mm Hg.

A patient who has recently started taking pravastatin (Pravachol) and niacin (Nicobid) reports the following symptoms to the nurse. Which is most important to communicate to the health care provider? a. Generalized muscle aches and pains

Muscle aches and pains may indicate myopathy and rhabdomyolysis, which have caused acute kidney injury and death in some patients who have taken the statin medications.

The nurse is assessing a patient with myocarditis before administering the scheduled dose of digoxin (Lanoxin). Which finding is most important for the nurse to communicate to the health care provider? b. Irregular pulse

Myocarditis predisposes the heart to digoxin-associated dysrhythmias and toxicity. T

The client with pericarditis is prescribed a nonsteroidal anti-inflammatory drug (NSAID). Which teaching instruction should the nurse discuss with the client? 3. Instruct the client to take the medication with food.

NSAIDs must be taken with food, milk, or antacids to help decrease gastric distress. NSAIDs reduce fever, inflammation, and pericardial pain.

During the assessment of a 25-year-old patient with infective endocarditis (IE), the nurse would expect to find a new regurgitant murmur.

New regurgitant murmurs occur in IE because vegetations on the valves prevent valve closure.

A nurse is advising a 24-year-old obese female about smoking cessation, since smoking can lead to coronary artery disease. Why does the nurse discourage cigarette smoking? Select all that apply.

Nicotine present in tobacco smoke stimulates the release of catecholamines. Catecholamines have a stimulatory effect on the sympathetic nervous system that causes an increase in heart rate and blood pressure. Carbon monoxide present in tobacco smoke has a greater affinity to hemoglobin than oxygen. Therefore carbon monoxide reduces the oxygen carrying capacity of blood. Tobacco smoke is known to increase the level of low-density lipoproteins and subsequently decrease in high-density lipoproteins. All these factors can lead to atherosclerosis. Tobacco smoke is known to decrease estrogen levels in premenopausal women thereby increasing their susceptibility to get coronary artery disease.

A patient with chronic stable angina has received a prescription for nitroglycerin (Nitrolingual). The nurse tells the patient that orthostatic hypotension is an important side effect of this drug. To prevent complications associated with the side effect, what should the nurse include in the patient's teaching?

Nitrates cause vasodilatation, which in turn causes peripheral pooling of blood. On changing position suddenly (from recumbent to erect posture), there is a sudden drop of blood pressure, which is known as orthostatic hypotension. This decreases the blood supply to the brain, which may lead to dizziness and falling. Headaches and flushing are not associated with orthostatic hypotension. If there is no tingling sensation on administering the drug, then the medication may be expired or ineffective.

The patient with pericarditis is complaining of chest pain. After assessment, which intervention should the nurse expect to implement to provide pain relief? D. Nonsteroidal antiinflammatory drugs

Nonsteroidal antiinflammatory drugs (NSAIDs) will control pain and inflammation.

The intensive care department nurse is assessing the client who is 12 hours post-myocardial infarction. The nurse assesses an S3 heart sound. Which intervention should the nurse implement?

Notify the health-care provider immediately. - means left ventricular failure, should be reported ASAP

Which assessment data would warrant immediate intervention in the client diagnosed with arterial occlusive disease? 3. The client has numbness and tingling.

Numbness and tingling are paresthesia, which is a sign of a severely decreased blood supply to the lower extremities.

A nurse is assessing a client's right lower leg, which is wrapped with an elastic bandage. Which signs and symptoms suggest circulatory impairment?

Numbness, cool skin temperature, and pallor

D. Desirability of promoting normalcy within the limits of the child's condition. Child needs increased caloric intake. Child needs discipline and appropriate limits. Because cyanotic spells occur in children with some defects, the parents need to be taught how to manage these. The child needs to have social interactions, discipline, and appropriate limit-setting. Parents need to be encouraged to promote as normal a life as possible for their child.

Nurses counseling parents regarding the home care of the child with a cardiac defect before corrective surgery should stress the: A. Importance of reducing caloric intake to decrease cardiac demands. B. Importance of relaxing discipline and limit-setting to prevent crying. C. Need to be extremely concerned about cyanotic spells. D. Desirability of promoting normalcy within the limits of the child's condition.

C. Organizing activities to allow for uninterrupted sleep. The child who has congestive heart failure has an excess of fluid. Monitoring vital signs is appropriate, but minimizing energy expenditure is a priority. The child needs to be well rested before feeding. The child's needs should be met as quickly as possible to minimize crying. The nurse must organize care to facilitate a decrease in his or her energy expenditure. The child often cannot tolerate larger feedings.

Nursing care of the infant or child with congestive heart failure would include: A. Forcing fluids appropriate to age. B. Monitoring respirations during active periods. C. Organizing activities to allow for uninterrupted sleep. D. Giving larger feedings less often to conserve energy.

Which of the following are risk factors for venous disorders of the lower extremities?

Obesity

Aortic stenosis

Obstructive defects

Coarctation of the aorta

Obstructive defects

Pulmonic stenosis

Obstructive defects

Subvalvular aortic stenosis

Obstructive defects

Valvular aortic stenosis

Obstructive defects

You are working in a long-term care facility with a group of older adults with cardiac disorders. Why would it be important for you to closely monitor an older adult receiving digitalis preparations for cardiac disorders?

Older adults are at increased risk for toxicity.

Which statement by an 84-year-old patient with coronary artery disease (CAD) indicates understanding of discharge teaching about physical activity?

Older adults have to use longer rest periods between exercise sessions because of decreased endurance and ability to tolerate stress. The older adults have decreased sweating and therefore shouldn't exercise in extremes of temperature outside. The older adults have to perform low level activity exercise for longer periods of time. The elderly adults have to change their lifestyle to accommodate a physical activity program, even though they are more prone to make such changes during hospitalization or when experiencing symptoms of CAD.

The male client is diagnosed with Guillain Barré syndrome (GB) and is in the intensive care unit on a ventilator. Which cardiovascular rationale explains implementing passive range of motion (ROM) exercises? 4. They help to prevent DVTs by movement of the blood through the veins.

One reason for performing range of motion exercises is to assist the blood vessels in the return of blood to the heart, preventing DVT.

The client is diagnosed with an abdominal aortic aneurysm. Which statement would the nurse expect the client to make during the admission assessment? 2. "I don't have any abdominal pain or any type of problems."

Only about two-fifths of clients with AAA have symptoms; the remainder are asymptomatic.

The male client diagnosed with essential hypertension has been prescribed an alphaadrenergic blocker. Which intervention should the nurse discuss with the client? Change position slowly when going from lying to sitting position.

Orthostatic hypotension may occur when the blood pressure is decreasing and may lead to dizziness and light-headedness so the client should change position slowly.

The health-care provider has ordered an angiotensin-converting enzyme (ACE) inhibitor for the client diagnosed with congestive heart failure. Which discharge instructions should the nurse include? 2. Teach the client how to prevent orthostatic hypotension.

Orthostatic hypotension may occur with ACE inhibitors as a result of vasodilation. Therefore, the nurse should instruct the client to rise slowly and sit on the side of the bed until equilibrium is restored.

When caring for a patient with infective endocarditis, the nurse will assess the patient for which vascular manifestations (select all that apply)? A. Osler's nodes B. Janeway's lesions C. Splinter hemorrhages

Osler's nodes, Janeway's lesions, and splinter hemorrhages are all vascular manifestations of infective endocarditis.

For which problem is percutaneous coronary intervention (PCI) most clearly indicated? d) Acute myocardial infarction

PCI is indicated to restore coronary perfusion in cases of myocardial infarction.

The client is being admitted with Coumadin (warfarin), an anticoagulant, toxicity. Which laboratory data should the nurse monitor?. 3. International Normalized Ratio (INR).

PT/INR is a test to monitor warfarin (Coumadin) action in the body.

Which signs/symptoms should the nurse assess in any client who has a long-term valvular heart disease? Select all that apply. 1. Paroxysmal nocturnal dyspnea. 2. Orthopnea. 3. Cough.

Paroxysmal nocturnal dyspnea is a sudden attack of respiratory distress usually occurring at night because of the reclining position and occurs in valvular disorders. This is an abnormal condition in which a Client must sit or stand to breathe comfortably and occurs in valvular disorders. Coughing occurs when the client with long-term valvular disease has difficulty breathing when walking or performing any type of activity.

The nurse will suspect that the patient with stable angina is experiencing a side effect of the prescribed metoprolol (Lopressor) if the b. blood pressure is 90/54 mm Hg.

Patients taking β-adrenergic blockers should be monitored for hypotension and bradycardia.

After the nurse teaches the patient about the use of carvedilol (Coreg) in preventing anginal episodes, which statement by a patient indicates that the teaching has been effective?b. "It is important not to suddenly stop taking the carvedilol."

Patients who have been taking β-adrenergic blockers can develop intense and frequent angina if the medication is suddenly discontinued.

The nurse establishes the nursing diagnosis of ineffective health maintenance related to lack of knowledge regarding long-term management of rheumatic fever when a 30-year-old recovering from rheumatic fever without carditis says which of the following? d. "I will be immune to further episodes of rheumatic fever after this infection."

Patients with a history of rheumatic fever are more susceptible to a second episode.

What nursing action should the nurse prioritize during the care of a patient who has recently recovered from rheumatic fever? C. Teach the patient about the need for continuous antibiotic prophylaxis.

Patients with a history of rheumatic fever frequently require ongoing antibiotic prophylaxis, an intervention that necessitates education

The nurse performs discharge teaching for a 68-year-old man who is newly diagnosed with infective endocarditis with a history of IV substance abuse. Which statement by the patient indicates to the nurse that teaching was successful? B. "I will inform my dentist about my hospitalization for infective endocarditis."

Patients with infective endocarditis should inform their dental providers of their health history.

Which statement by a patient with restrictive cardiomyopathy indicates that the nurse's discharge teaching about self-management has been most effective? c. "I will take antibiotics when my teeth are cleaned at the dental office."

Patients with restrictive cardiomyopathy are at risk for infective endocarditis and should use prophylactic antibiotics for any procedure that may cause bacteremia.

To assess the patient with pericarditis for evidence of a pericardial friction rub, the nurse should b. auscultate by placing the diaphragm of the stethoscope on the lower left sternal border.

Pericardial friction rubs are heard best with the diaphragm at the lower left sternal border.

A patient comes into the emergency room complaining about chest pain that gets worse when taking deep breaths and lying down. After ruling out a myocardial infarction, a nurse would assess for which of the following diagnoses?

Pericarditis

While admitting a patient with pericarditis, the nurse will assess for what manifestations of this disorder? A. Pulsus paradoxus

Pericarditis can lead to cardiac tamponade, an emergency situation. Pulsus paradoxus greater than 10 mm Hg is a sign of cardiac tamponade that should be assessed at least every 4 hours in a patient with pericarditis.

A nurse is caring for a client following an arterial vascular bypass graft in the leg. Over the next 24 hours, what should the nurse plan to assess?

Peripheral pulses every 15 minutes following surgery

The patient with cardiac failure is taught to report which of the following symptoms to the physician or clinic immediately?

Persistent cough

An increased number of red blood cells; increases the oxygen carrying capacity of the blood

Polycythemia

Postoperative care of a patient undergoing coronary artery bypass graft (CABG) surgery includes monitoring for what common complication? c) Atrial dysrhythmias

Postoperative dysrhythmias, specifically atrial dysrhythmias, are common in the first 3 days following CABG surgery.

Postoperative care of a patient undergoing coronary artery bypass graft (CABG) surgery includes monitoring for what common complication?

Postoperative dysrhythmias, specifically atrial dysrhythmias, are common in the first three days following CABG surgery. Although dehydration, paralytic ileus, and acute respiratory distress syndrome could occur, they are not common complications.

A patient is prescribed digitalis preparations. Which of the following conditions should the nurse closely monitor when caring for the patient?

Potassium levels

The instructor is talking with a nursing student who is caring for a client with pericarditis. The instructor asks the student to name the main characteristic of pericarditis. What should be the student's answer?

Precordial pain

Which of the following describes the amount of blood presented to the ventricle just before systole?

Preload

The client is exhibiting sinus bradycardia, is complaining of syncope and weakness, and has a BP of 98/60. Which collaborative treatment should the nurse anticipate being implemented?

Prepare for insertion of a pacemaker. - client is symptomatic and will need a pacemaker - PAY ATTENTION IT SAYS COLLABORATIVE, NOT INDEPENDENT! - assessment is independent action

A patient with pericarditis is experiencing cardiac tamponade. Which collaborative intervention should the nurse anticipate for this patient?

Prepare for pericardiocentesis

The client with varicose veins is six (6) hours post-operative vein ligation. Which nursing intervention should the nurse implement first? 2. Assess and maintain pressure bandages on the affected leg.

Pressure bandages are applied for up to six (6) weeks after vein ligation to help prevent bleeding and to help venous return from the lower extremities when in the standing or sitting position.

Diltiazem (Cardizem) is ordered for a patient with newly diagnosed Prinzmetal's (variant) angina. When teaching the patient, the nurse will include the information that diltiazem will b. decrease spasm of the coronary arteries.

Prinzmetal's angina is caused by coronary artery spasm. Calcium channel blockers (e.g., diltiazem, amlodipine [Norvasc

Which of the following mitral valve conditions generally produces no symptoms?

Prolapse

A client is admitted to the hospital with systolic left-sided heart failure. The nurse knows to look for which of the following assessment findings for this client?

Pulmonary congestion

After reviewing a patient's history, vital signs, physical assessment, and laboratory data, which information shown in the accompanying figure is most important for the nurse to communicate to the health care provider? d. Tachypnea and crackles in lungs

Pulmonary congestion and tachypnea suggest that the patient may be developing heart failure, a complication of myocardial infarction (MI).

A physician has scheduled a client with mitral stenosis for mitral valve replacement. Which condition may arise as a complication of mitral stenosis?

Pulmonary hypertension

Which valve lies between the right ventricle and the pulmonary artery?

Pulmonic

The nurse suspects cardiac tamponade in a patient who has acute pericarditis. To assess for the presence of pulsus paradoxus, the nurse should a. note when Korotkoff sounds are auscultated during both inspiration and expiration.

Pulsus paradoxus exists when there is a gap of greater than 10 mm Hg between when Korotkoff sounds can be heard during only expiration and when they can be heard throughout the respiratory cycle.

Which of the following would be inconsistent as a lifestyle change directive for the patient diagnosed with heart failure?

Push fluids

The client diagnosed with essential hypertension is taking a loop diuretic daily. Which assessment data would require immediate intervention by the nurse? 2. The client has a weight gain of 2 kg within 1-2 days.

Rapid weight gain—for example, 2 kg in 1-2 days—indicates that the loop diuretic is not working effectively; 2 kg equals 4.4 lbs; 1 L of fluid weighs l kg.

Part of the continued management of a patient with infective endocarditis is assessment for the presence of Janeway lesions. On inspection, the nurse recognizes these lesions by identifying which characteristic sign?

Red or purple macules found on the palms of the hands

The nurse is discussing discharge teaching with the client who is three (3) days postoperative abdominal aortic aneurysm repair. Which discharge instructions should the nurse include when teaching the client? 1. Notify HCP of any redness or irritation of incision.

Redness or irritation of the incision indicates infection and should be reported immediately to the HCP.

The unlicensed nursing assistant is caring for the client diagnosed with chronic venous insufficiency. Which action would warrant immediate intervention from the nurse? 1. Applying compression stockings before going to bed.

Research shows that removing the compression stockings while the client is in bed promotes perfusion of the subcutaneous tissue. The foot of the bed should be elevated.

The wife of a client with arterial occlusive disease tells the nurse, "My husband says he is having rest pain. What does that mean?" Which statement by the nurse would be most appropriate? 2. "His legs are deprived of oxygen during periods of inactivity."

Rest pain indicates a worsening of the arterial occlusive disease; the muscles of the legs are not getting enough oxygen when the client is resting to prevent muscle ischemia.

Which of the following types of cardiomyopathy are characterized by diastolic dysfunction caused by rigid ventricular walls that impair diastolic filling and ventricular stretch.

Restrictive cardiomyopathy (RCM)

A patient comes to the clinic with complaints of fever, chills, and sore throat and is diagnosed with streptococcal pharyngitis. A nurse knows that early diagnosis and effective treatment is essential to avoid which of the following preventable diseases?

Rheumatic fever

A 25-year-old patient with a group A streptococcal pharyngitis does not want to take the antibiotics prescribed. What should the nurse tell the patient to encourage the patient to take the medications and avoid complications of the infection? C. "Without treatment, you could get rheumatic fever, which can lead to rheumatic heart disease."

Rheumatic fever (RF) is not common because of effective use of antibiotics to treat streptococcal infections.

The nurse is admitting a patient with possible rheumatic fever. Which question on the admission health history will be most pertinent to ask? b. "Have you had a recent sore throat?"

Rheumatic fever occurs as a result of an abnormal immune response to a streptococcal infection.

The client is admitted to the medical unit to rule out carditis. Which question should the nurse ask the client during the admission interview to support this diagnosis? 2. "Did you have rheumatic fever as a child?"

Rheumatic fever, a systemic inflammatory disease caused by an abnormal immune response to pharyngeal infection by group A beta-hemolytic streptococci, causes carditis in about 50% of the people.

The client is being evaluated for valvular heart disease. Which information would be most significant? 4. The client has a history of rheumatic heart disease.

Rheumatic heart disease is the most common cause of valvular heart disease.

Which of the following is the most effective intervention for preventing progression of vascular disease?

Risk factor modification

The nurse is providing discharge teaching for a client with rheumatic endocarditis but no valvular dysfunction. On which nursing diagnosis should the nurse focus her teaching?

Risk for infection

Which of the following symptoms occurs in the patient diagnosed with mitral regurgitation when pulmonary congestion occurs?

SOB

A nurse has identified a group of people who are at risk for developing coronary artery disease. To prevent atherosclerosis, the nurse advises a reduction in salt consumption. How does salt consumption increase the risk of developing atherosclerosis?

Salt contains sodium, which causes water retention in the body and thus hypertension. The shearing stress due to elevated blood pressure causes endothelial injury. This makes the blood vessels more susceptible to develop atherosclerosis. Hormonal changes, hyperlipidemia, and high homocysteine levels also contribute to atherosclerosis but are not caused by increased salt intake. Changes in the hormonal levels like a decrease in estrogen levels can increase the risk of atherosclerosis. Hyperlipidemia is a major predisposing factor for the development of atherosclerosis. Homocysteine contributes to atherosclerosis by damaging the inner lining of blood vessels, promoting plaque buildup, and altering the clotting mechanism to make clots more likely to occur.

The nurse has written an outcome goal "demonstrates tolerance for increased activity" for a client diagnosed with congestive heart failure. Which intervention should the nurse implement to assist the client to achieve this outcome? 4. Plan for frequent rest periods.

Scheduling activities and rest periods allows the client to participate in his or her own care and addresses the desired outcome.

The nurse is teaching a class on arterial essential hypertension. Which modifiable risk factors would the nurse include when preparing this presentation? 2. Discuss sedentary lifestyle and smoking cessation.

Sedentary lifestyle is discouraged in clients with hypertension, and daily isotonic exercises are recommended. Smoking increases the atherosclerotic process in vessels; causes vasoconstriction of vessels; and adheres to hemoglobin, decreasing oxygen levels.

A nurse is caring for a client with acute mitral regurgitation related to an acute myocardial infarction. The nurse knows to monitor the client carefully for symptoms of which initial complication or result?

Severe heart failure

A patient has been diagnosed with systolic heart failure. The nurse would expect the patient's ejection fraction to be at which level?

Severely reduced

The elderly client has coronary artery disease. Which question should the nurse ask the client during the client teaching? 3. "Are you sexually active?"

Sexual activity is a risk factor for angina resulting from coronary artery disease. The client's being elderly should not affect the nurse's assessment of the client's concerns about sexual activity.

A patient who is recovering from an acute myocardial infarction (AMI) asks the nurse about when sexual intercourse can be resumed. Which response by the nurse is best? b. "Sexual activity uses about as much energy as climbing two flights of stairs."

Sexual activity places about as much physical stress on the cardiovascular system as most moderate-energy activities such as climbing two flights of stairs.

Which intervention should the nurse implement when defibrillating a client who is in ventricular fibrillation?

Shout "all clear" prior to defibrillating the client. - if anybody is touching the client or bed , that person could possibly be shocked - adults defibrillated at 360 joules - oxygen should be removed to prevent spark - use defib pads pr gels not petroleum gel

The nurse assesses a patient with diabetes who reports shortness of breath, neck pain, and hypoglycemic symptoms. The patient's blood pressure is 130/86 mm Hg, heart rate is 102 beats/min, respiratory rate is 24 breaths/min, and the fingerstick blood glucose is 136 mg/dL. The nurse recognizes that the patient may be experiencing:

Signs and symptoms of a myocardial infarction (MI) include shortness of breath, neck pain, and cool, clammy skin. Although cool, clammy skin may resemble a hypoglycemic reaction, when found along with shortness of breath and neck pain it is very specific for an MI. The patient is not experiencing a complication of diabetes (ketoacidosis or hyperosmolar hyperglycemic nonketotic syndrome). The blood glucose is close to normal, and further diagnostics would be required to determine a diabetic complication. Only cool, clammy skin is indicative of hypoglycemia

The client that is one (1)-day postoperative coronary artery bypass surgery is exhibiting sinus tachycardia. Which intervention should the nurse implement? 4. Determine if the client is having pain

Sinus tachycardia means the sino-atrial node is the pacemaker, but the rate is greater than 100 because of pain, anxiety, or fever. The nurse must determine the cause and treat appropriately. There is no specific medication for sinus tachycardia

The nurse has identified a nursing diagnosis of acute pain related to inflammatory process for a patient with acute pericarditis. The priority intervention by the nurse for this problem is to d. place the patient in Fowler's position, leaning forward on the overbed table.

Sitting upright and leaning forward frequently will decrease the pain associated with pericarditis.

Which is a potassium-sparing diuretic used in the treatment of heart failure?

Spironolactone (Aldactone)

The HCP prescribes an HMG-COA reductase inhibitor (statin) medication to a client with CAD. Which should the nurse teach the client about this medication? 2. This medication should be taken in the evening.

Statin medications should be taken in the evening for best results because the enzyme that destroys cholesterol works best in the evening and the medication enhances this process.

A nurse is providing education about the prevention of arterial constriction to a client with peripheral arterial disease. Which of the following includes priority information the nurse would give to the client?

Stop smoking.

The nurse is discussing angina with a client who is diagnosed with coronary artery disease. Which action should the client take first when experiencing angina?

Stop the activity immediately and rest. - that is first action , then nitroglycerine

The client diagnosed with rule-out myocardial infarction is experiencing chest pain while walking to the bathroom. Which action should the nurse implement first? 3. Have the client sit down immediately.

Stopping all activity will decrease the need of the myocardium for oxygen and may help decrease the chest pain.

The nurse is discussing angina with a client who is diagnosed with coronary artery disease. Which action should the client take first when experiencing angina? 2. Stop the activity immediately and rest.

Stopping the activity decreases the heart's need for oxygen and may help decrease the angina, chest pain.

Which assessment finding in a patient who is admitted with infective endocarditis (IE) is most important to communicate to the health care provider? b. Sudden onset right flank pain

Sudden onset of flank pain indicates possible embolization to the kidney and may require diagnostic testing such as a renal arteriogram and interventions to improve renal perfusion.

Along with persistent, crushing chest pain, which signs/symptoms would make the nurse suspect that the client is experiencing a myocardial infarction? 2. Diaphoresis and cool clammy skin.

Sweating is a systemic reaction to the MI. The body vasoconstricts to shunt blood from the periphery to the trunk of the body; this, in turn, leads to cold, clammy skin.

Abnormally fast heart rate

Tachydysrhythmias

The health-care provider has ordered an angiotensin-converting enzyme (ACE) inhibitor for the client diagnosed with congestive heart failure. Which discharge instructions should the nurse include?

Teach the client how to prevent orthostatic hypotension. - if cough stop the medication - may cause hyperkalemia - take 1 hour before meals or 2 hours after meals

What should the nurse do to manage the persistent swelling in a patient with severe lymphangitis and lymphadenitis?

Teach the patient how to apply a graduated compression stocking.

Which individuals would the nurse identify as having the highest risk for CAD? a) A 45-year-old depressed male with a high-stress job

The 45-year-old depressed male with a high-stress job is at the highest risk for CAD.

The charge nurse is making assignments for clients on a cardiac unit. Which client should the charge nurse assign to a new graduate nurse?

The 75-year-old client scheduled for a cardiac catheterization. - not a client diagnosed with MI - not a client with unstable angina - not a client who complaining of chest pain

Which statement by the client diagnosed with coronary artery disease indicates that the client understands the discharge teaching concerning diet? 2. "I should bake or grill any meats I eat."

The American Heart Association recommendS a low-fat, low-cholesterol diet for a client with coronary artery disease. The client should avoid any fried foods, especially meats, and bake, boil, or grill any meat.

When planning emergent care for a patient with a suspected MI, what should the nurse anticipate administrating? a) Oxygen, nitroglycerin, aspirin, and morphine

The American Heart Association's guidelines for emergency care of the patient with chest pain include the administration of oxygen, nitroglycerin, aspirin, and morphine.

The nurse is teaching the Dietary Approaches to Stop Hypertension (DASH) diet to a client diagnosed with essential hypertension. Which statement indicates that the client understands the client teaching concerning the DASH diet? 1. "I should eat at least four (4) to five (5) servings of vegetables a day."

The DASH diet has proved beneficial in lowering blood pressure. It recommends eating a diet high in vegetables and fruits.

The client with coronary artery disease is prescribed a Holter monitor. Which intervention should the nurse implement? 1. Instruct client to keep a diary of activity, especially when having chest pain.

The Holter monitor is a 24-hour electrocardiogram, and the client must keep an accurate record of activity so that the health-care provider can compare the ECG recordings with different levels of activity.

Which assessment finding obtained by the nurse when assessing a patient with acute pericarditis should be reported immediately to the health care provider? c. Jugular venous distention (JVD) to jaw level

The JVD indicates that the patient may have developed cardiac tamponade and may need rapid intervention to maintain adequate cardiac output.

The nurse and an unlicensed nursing assistant are caring for four clients on a telemetry unit. Which nursing task would be best for the nurse delegate to the unlicensed nursing assistant? 4. Help position the client who is having a portable x-ray done

The UAP can assist the x-ray technician in positioning the client for the portable xray. This does not require judgment.

The nurse and an unlicensed nursing assistant are bathing a bedfast client. Which action by the assistant warrants immediate intervention? 2. The assistant begins to massage and rub lotion into the client's calf.

The assistant could dislodge a blood clot in the leg when massaging the calf. The assistant can apply lotion gently, being sure not to massage the leg.

The nurse assesses the right femoral artery puncture site as soon as the patient arrives after having a stent inserted into a coronary artery. The insertion site is not bleeding or discolored. What should the nurse do next to ensure the femoral artery is intact? c) Inspect the patient's right side and back.

The best method to determine that the right femoral artery is intact after inspection of the insertion site is to logroll the patient to inspect the right side and back for retroperitoneal bleeding.

. A patient with hyperlipidemia has a new order for colesevelam (Welchol). Which nursing action is most appropriate when giving the medication? d. Give the patient's other medications 2 hours after the colesevelam.

The bile acid sequestrants interfere with the absorption of many other drugs, and giving other medications at the same time should be avoided.

During the administration of the thrombolytic agent to a patient with an acute myocardial infarction (AMI), the nurse should stop the drug infusion if the patient experiences c. a decrease in level of consciousness.

The change in level of consciousness indicates that the patient may be experiencing intracranial bleeding, a possible complication of thrombolytic therapy.

Which assessment finding by the nurse caring for a patient who has had coronary artery bypass grafting using a right radial artery graft is most important to communicate to the health care provider? b. Pallor and weakness of the right hand

The changes in the right hand indicate compromised blood flow, which requires immediate evaluation and actions such as prescribed calcium channel blockers or surgery.

The nurse just received the A.M. shift report. Which client should the nurse assess first? The client diagnosed with deep vein thrombosis who is complaining of chest pain.

The chest pain could be a pulmonary embolus secondary to deep vein thrombosis and requires immediate intervention by the nurse.

The nurse on the telemetry unit has just received the a.m. shift report. Which client should the nurse assess first?

The client diagnosed with myocardial infarction who has an audible S3 heart sounds. - S3 is ventricular failure it is an emergency

Which client problem would be priority in a client diagnosed with arterial occlusive disease who is admitted to the hospital with a foot ulcer? 1. Impaired skin integrity.

The client has a foot ulcer, therefore the protective lining of the body—the skin— has been impaired.

The nurse is preparing to administer a beta blocker to the client diagnosed with coronary artery disease. Which assessment data would cause the nurse to question administering the medication?

The client has an apical pulse of 56.

The client with a mechanical valve replacement asks the nurse, "Why do I have to take antibiotics before getting my teeth cleaned?" Which response by the nurse is most appropriate? 1. "You are at risk of developing an infection in your heart."

The client is at risk for developing endocarditis and should take prophylactic antibiotics before any invasive procedure.

A pregnant client who developed deep vein thrombosis (DVT) in her right leg is receiving heparin I.V. on the medical floor. Physical therapy is ordered to maintain her mobility and prevent additional DVT. A nursing assistant working on the medical unit helps the client with bathing, range-of-motion exercises, and personal care. Which collaborative multidisciplinary considerations should the care plan address?

The client is at risk for heparin-induced thrombocytopenia; therefore, the care plan should include reporting evidence of bleeding or easy bruising.

The client is exhibiting sinus bradycardia, is complaining of syncope and weakness, and has a BP of 98/60. Which collaborative treatment should the nurse anticipate being implemented? 3. Prepare for an insertion of a pacemaker.

The client is symptomatic and will require a pacemaker.

The client is one (1) day post-operative abdominal aortic aneurysm repair. Which information from the unlicensed nursing assistant would require immediate intervention from the nurse? 2. The client's urinary output is 90 mL in six (6) hours.

The client must have 30 mL urinary output every hour. Clients who are post-operative AAA are at high risk for renal failure because of the anatomical location of the AAA near the renal arteries.

The client tells the nurse that his cholesterol level is 240 mg/dL. Which action should the nurse implement? 2. Explain that the client needs to lower the cholesterol level.

The client needs to be taught ways to lower the cholesterol level.

The client has just returned from a cardiac catheterization. Which assessment data would warrant immediate intervention from the nurse?

The client refuses to keep the leg straight.

A client with right-sided heart failure is admitted to the medical-surgical unit. What information obtained from the client may indicate the presence of edema?

The client says his rings have become tight and are difficult to remove.

The nurse is developing a discharge-teaching plan for the client diagnosed with congestive heart failure. Which intervention should be included in the plan? Select all that apply. 2. Teach client how to count the radial pulse when taking digoxin, a cardiac glycoside. 3. Instruct client to remove the saltshaker from the dinner table.

The client should not take digoxin if radial pulse is less than 60. The client should be on a low-sodium diet to prevent water retention.

The male client is diagnosed with coronary artery disease (CAD) and is prescribed sublingual nitroglycerin. Which statement indicates the client needs more teaching? 4. "If my chest pain is not gone with one tablet, I will go to the ER."

The client should take one tablet every five (5) minutes and, if no relief occurs after the third tablet, have someone drive him to the emergency department or call 911.

The client is admitted to the telemetry unit diagnosed with acute exacerbation of congestive heart failure (CHF). Which signs/symptoms would the nurse expect to find when assessing this client? 1. Apical pulse rate of 110 and 4 pitting edema of feet.

The client with CHF would exhibit tachycardia (apical pulse rate of 110), dependent edema, fatigue, third heart sounds, lung congestion, and change in mental status.

The nurse is administering a calcium channel blocker to the client diagnosed with a myocardial infarction. Which assessment data would cause the nurse to question administering this medication?

The client's blood pressure is 90/62. - apical pulse between 60-100 can give - serum calcium is not monitored for Ca channel blockers - if BP lower than 90 do not give

The nurse is administering a calcium channel blocker to the client diagnosed with a myocardial infarction. Which assessment data would cause the nurse to question administering this medication? 4. The client's blood pressure is 90/62.

The client's blood pressure is low, and a calcium channel blocker would cause the blood pressure to bottom out.

The nurse obtains the following data when assessing a patient who experienced an ST-segment-elevation myocardial infarction (STEMI) 2 days previously. Which information is most important to report to the health care provider? c. Bilateral crackles are auscultated in the mid-lower lobes.

The crackles indicate that the patient may be developing heart failure, a possible complication of myocardial infarction (MI).

The nurse is assessing the client diagnosed with long-term arterial occlusive disease. Which assessment data support the diagnosis? 1. Hairless skin on the legs.

The decreased oxygen over time causes the loss of hair on top of feet and ascends both legs.

1. The 66-year-old male client has his blood pressure (BP) checked at a health fair. The B/P is 168/98. Which action should the nurse implement first? Instruct the client to see his health-care provider as soon as possible.

The diastolic blood pressure should be less than 85 according to the American Heart Association; therefore, this client should see the health-care provider.

The 66-year-old male client has his blood pressure (BP) checked at a health fair. The B/P is 168/98. Which action should the nurse implement first? Instruct the client to see his health-care provider as soon as possible.

The diastolic blood pressure should be less than 85 according to the American Heart Association; therefore, this client should see the health-care provider.

D. Increased pulmonary vascular congestion. The increased pulmonary vascular congestion is the primary complication. The shunt of blood is left to right. The increased pulmonary vascular congestion is the primary complication. A PDA allows blood to flow from the aorta (high pressure) to the pulmonary artery (low pressure). If the PDA stays open, increased pulmonary congestion can occur.

The doctor suggests that surgery be performed for patent ductus arteriosus (PDA) to prevent: A. Pulmonary infection. B. Right-to-left shunt of blood. C. Decreased workload on left side of heart. D. Increased pulmonary vascular congestion.

After the nurse has finished teaching a patient about the use of sublingual nitroglycerin (Nitrostat), which patient statement indicates that the teaching has been effective? c. "I will call an ambulance if I still have pain after taking 3 nitroglycerin 5 minutes apart."

The emergency medical services (EMS) system should be activated when chest pain or other symptoms are not completely relieved after 3 sublingual nitroglycerin tablets taken 5 minutes apart. Nitroglycerin can be taken to prevent chest pain or other symptoms from developing (e.g., before intercourse).

When titrating IV nitroglycerin (Tridil) for a patient with a myocardial infarction (MI), which action will the nurse take to evaluate the effectiveness of the medication? b. Ask about chest pain.

The goal of IV nitroglycerin administration in MI is relief of chest pain by improving the balance between myocardial oxygen supply and demand.

The client diagnosed with a myocardial infarction asks the nurse, "Why do I have to rest and take it easy? My chest doesn't hurt anymore." Which statement would be the nurse's best response? 1. "Your heart is damaged and needs about four (4) to six (6) weeks to heal."

The heart tissue is dead, stress or activity may cause heart failure, and it does take about six (6) weeks for scar tissue to form.

A nurse is caring for a 30-year-old client diagnosed with atrial fibrillation who has just had a mitral valve replacement. The client is being discharged with prescribed warfarin (Comaudin). She mentions to you that she relies on the rhythm method for birth control. What education would be a priority for the nurse to provide to this client?

The high risk for complications if she becomes pregnant while taking warfarin

Which patient at the cardiovascular clinic requires the most immediate action by the nurse? b. Patient with stable angina whose chest pain has recently increased in frequency

The history of more frequent chest pain suggests that the patient may have unstable angina, which is part of the acute coronary syndrome spectrum.

The nurse has just received the A.M. shift report. Which client would the nurse assess first? 3. The client with arterial occlusive disease who cannot move the foot.

The inability to move the foot means that a severe neurovascular compromise has occurred, and the nurse should assess this client first.

The community health nurse is planning health promotion teaching targeted at preventing coronary artery disease (CAD). Which ethnic group would the nurse select as the highest priority for this intervention? a) White male

The incidence of CAD and myocardial infarction (MI) is highest among white, middle-aged men.

When developing a community health program to decrease the incidence of rheumatic fever, which action would be most important for the community health nurse to include? b. Teach community members to seek treatment for streptococcal pharyngitis.

The incidence of rheumatic fever is decreased by treatment of streptococcal infections with antibiotics.

A nurse caring for a patient with mitral stenosis understands that the initial cause of disruption to the normal flow of blood through the heart is due to:

The increased resistance of a narrowed orifice between the left atrium and the left ventricle

The client diagnosed with arterial occlusive disease is one (1) day post-operative right femoral popliteal bypass. Which intervention should the nurse implement? 4. Assess the client's leg dressing every four (4) hours.

The leg dressing needs to be assessed for hemorrhaging or signs of infection.

The patient recently diagnosed with coronary artery disease (CAD) asks the nurse: "What caused my problem?" Which response by the nurse is the most appropriate?

The major cause of CAD is an atherosclerosis that is manifested by fatty deposits on the walls of coronary arteries. Decrease in pumping action of the heart will result in congestive heart failure (CHF). Low oxygen saturation of the blood is a result of respiratory problems. Hypertension, not orthostatic hypotension, will predispose a patient to development of CAD.

Which assessment data would support that the client has a venous stasis ulcer? 1. Superficial pink open area on the medial part of the ankle.

The medial part of the ankle usually ulcerates because of edema that leads to stasis, which, in turn, causes the skin to break down.

Which client would be most likely to develop an abdominal aortic aneurysm? 3. A 69-year-old male with peripheral vascular disease.

The most common cause of AAA is atherosclerosis (which is the cause of peripheral vascular disease); it occurs in men four (4) times more often than women and primarily in Caucasians.

The client is diagnosed with a small abdominal aortic aneurysm. Which interventions should be included in the discharge teaching? Select all that apply. 1. Tell the client to exercise three (3) times a week for 30 minutes. 2. Encourage the client to eat a low-fat, low-cholesterol diet. 4. Discuss with the client the importance of losing weight.

The most common cause of AAA is atherosclerosis, so teaching should address this area. 2. A low-fat, low-cholesterol diet will help decrease development of atherosclerosis. 4. Losing weight will help decrease the pressure on the AAA and will help address decreasing the cholesterol level.

A patient was admitted to the emergency department (ED) 24 hours earlier with complaints of chest pain that were subsequently attributed to ST-segment-elevation myocardial infarction (STEMI). What complication of MI should the nurse anticipate? d) Cardiac dysrhythmias

The most common complication after MI is dysrhythmias, which are present in 80% of patients

The client diagnosed with congestive heart failure is complaining of leg cramps at night. Which nursing interventions should be implemented? 2. Monitor the client's potassium level and assess the client's intake of bananas and orange juice.

The most probable cause of the leg cramping is potassium excretion as a result of diuretic medication. Bananas and orange juice are foods that are high in potassium.

Which assessment data would the nurse expect to auscultate in the client diagnosed with mitral valve insufficiency? 2. A holosystolic murmur heard best at cardiac apex.

The murmur associated with mitral valve insufficiency is loud, high-pitched, rumbling, and holosystolic (occurring throughout systole) and is heard best at the cardiac apex.

A. Not give the dose; suspect dosage error. Digoxin is often prescribed in micrograms. Rarely is more than 1 ml administered to an infant. Because it is a potentially dangerous drug, administration guidelines are very precise. Some institutions require that digoxin dosages be checked with another professional before administration. The nurse has drawn up too much medication. These are correct procedures, but too much medication has been prepared. These are correct procedures, but too much medication has been prepared.

The nurse is preparing to give digoxin to a 9-month-old infant. He or she checks the dose and draws up 4 ml of the drug. The MOST appropriate nursing action is to: A. Not give the dose; suspect dosage error. B. Mix the dose with juice to disguise its taste. C. Check heart rate; administer the dose by placing it to the back and side of the mouth. D. Check heart rate; administer the dose by letting the infant suck it through a nipple.

The client is one (1) day postoperative coronary artery bypass surgery. The client complains of chest pain. Which intervention should the nurse implement first? 2. Assess the client's chest dressing and vital signs.

The nurse must always assess the client to determine if the chest pain that is occurring is expected postoperatively or if it is a complication of the surgery.

The client is exhibiting ventricular tachycardia. Which intervention should the nurse implement first? 3. Assess the client's apical pulse and blood pressure

The nurse must assess the apical pulse and blood pressure to determine if the client is in cardiac arrest and then treat as ventricular defibrillation. If the client's heart is beating, the nurse would then administer lidocaine.

The client diagnosed with pericarditis is complaining of increased pain. Which intervention should the nurse implement first? 3. Assess the client for cardiac complications.

The nurse must assess the client to determine if the pain is expected pain secondary to pericarditis or if the pain is indicative of a complication that requires intervention from the health-care provider.

During the 48 hours after a myocardial infarction, a nurse should assign the highest priority to monitoring the patient for:

The nurse must be most alert for dysrhythmias, which may signal another MI or impending complications. The nurse should also be alert for increased anxiety, which may cause pain and lead to a secondary infarction. Anxiety and fear are highly likely but secondary in importance to monitoring the patient for dysrhythmias. Metabolic acidosis and reactions to new medications are not likely but should still be included as part of overall assessment of the patient.

The client shows ventricular fibrillation on the telemetry at the nurse's station. Which action should the telemetry nurse implement first? 3. Call a STAT code.

The nurse must call a code that activates the crash cart being brought to the room and a team of health-care providers that will care for the client according to an established protocol.

The nurse enters the room of the client diagnosed with congestive heart failure. The client is lying in bed gasping for breath, is cool and clammy, and has buccal cyanosis. Which intervention would the nurse implement first? 4. Assist the client to a sitting position.

The nurse must first put the client in a sitting position to decrease the workload of the heart by decreasing venous return and maximizing lung expansion. Then, the nurse could take vital signs and check the pulse oximeter and then sponge the client's forehead.

The client is scheduled for a right femoral cardiac catheterization. Which nursing intervention should the nurse implement after the procedure? 2. Assess the client's neurovascular status.

The nurse must make sure that blood is circulating to the right leg so the client should be assessed for pulses, paresthesia, paralysis, coldness, and pallor.

The client diagnosed with a DVT is on a heparin (an anticoagulant) drip at 1400 units per hour, and Coumadin (warfarin sodium; also an anticoagulant) 5 mg twice a day. Which intervention should the nurse implement first? 1. Check the PTT and PT/INR.

The nurse should check the laboratory values pertaining to the medications before administering the medications.

The nurse is teaching a community group about preventing rheumatic fever. What information should the nurse include? A. Prompt recognition and treatment of streptococcal pharyngitis

The nurse should emphasize the need for prompt and adequate treatment of streptococcal pharyngitis infection, which can lead to the complication of rheumatic fever

A. Diuretic. Furosemide (Lasix) is a diuretic used to eliminate excess water and salt to prevent reaccumulation of the fluid. Lasix is a diuretic. Lasix is a diuretic. Lasix is a diuretic.

The nurse should explain to the parents that their child is receiving Lasix for severe congestive heart failure because it is a/an: A. Diuretic. B. â-blocker. C. Form of digitalis. D. ACE inhibitor.

A. Echocardiography Echocardiography uses high-frequency sound waves. The child must lie completely still. With the improvements in technology, diagnosis can sometimes be made without cardiac catheterization. Electrocardiography is a tracing of the electrical path of the depolarization action of myocardial cells. Cardiac catheterization is an invasive procedure in which a catheter is threaded into the heart. Electrophysiology is an invasive procedure in which catheters with electrodes are used to record the impulses of the heart directly from the conduction system.

The nurse should instruct a child to remain completely still during which procedure in which high frequency sound waves are translated into images by a transducer? A. Echocardiography B. Electrocardiography C. Cardiac catheterization D. Electrophysiology

The client diagnosed with a myocardial infarction is on bed rest. The unlicensed nursing assistant is encouraging the client to move the legs. Which action should the nurse implement? 3. Praise the nursing assistant for encouraging the client to move legs

The nurse should praise and encourage assistants to participate in the client's care. Clients on bed rest are at risk for deep vein thrombosis, and moving the legs will help prevent that.

The nurse is administering a beta blocker to the client diagnosed with essential hypertension. Which intervention should the nurse implement? 2. Question administering the medication if the blood pressure is 90/60 mmHg.

The nurse should question administering the beta blocker if the B/P is low because this medication will cause the blood pressure to drop even lower, leading to hypotension.

A patient who is being admitted to the emergency department with intermittent chest pain gives the following list of medications to the nurse. Which medication has the most immediate implications for the patient's care? a. Sildenafil (Viagra)

The nurse will need to avoid giving nitrates to the patient because nitrate administration is contraindicated in patients who are using sildenafil because of the risk of severe hypotension caused by vasodilation.

A patient recovering from heart surgery develops pericarditis and complains of level 6 (0 to 10 scale) chest pain with deep breathing. Which ordered PRN medication will be the most appropriate for the nurse to give? c. Oral ibuprofen (Motrin) 600 mg

The pain associated with pericarditis is caused by inflammation, so nonsteroidal antiinflammatory drugs (NSAIDs) (e.g., ibuprofen) are most effective.

The patient comes to the ED with severe, prolonged angina that is not immediately reversible. The nurse knows that if the patient once had angina related to a stable atherosclerotic plaque and the plaque ruptures, there may be occlusion of a coronary vessel and this type of pain. How will the nurse document this situation related to pathophysiology, presentation, diagnosis, prognosis B) Acute coronary syndrome (ACS)

The pain with ACS is severe, prolonged, and not easy to relieve.

After reviewing information shown in the accompanying figure from the medical records of a 43-year-old, which risk factor modification for coronary artery disease should the nurse include in patient teaching? c. Dietary changes to improve lipid levels

The patient has an elevated low-density lipoprotein (LDL) cholesterol and low high-density lipoprotein (HDL) cholesterol, which will increase the risk of coronary artery disease.

What collaborative and nursing care of this patient should be done to improve cardiac output and the quality of life? (Select all that apply.) A. Decrease preload and afterload. C. Control heart failure by enhancing myocardial contractility.

The patient is experiencing dilated cardiomyopathy. To improve cardiac output and quality of life, drug, nutrition, and cardiac rehabilitation will be focused on controlling heart failure by decreasing preload and afterload and improving cardiac output, which will improve the quality of life.

A patient is being seen in a clinic to rule out mitral valve stenosis. Which assessment data would be most significant?

The patient reports shortness of breath when walking.

A patient admitted with acute dyspnea is newly diagnosed with dilated cardiomyopathy. Which information will the nurse plan to teach the patient about managing this disorder? d. Notify the doctor about any symptoms of heart failure such as shortness of breath.

The patient should be instructed to notify the health care provider about any worsening of heart failure symptoms.

In preparation for discharge, the nurse teaches a patient with chronic stable angina how to use the prescribed short-acting and long-acting nitrates. Which patient statement indicates that the teaching has been effective? c. "I will stop what I am doing and sit down before I put the nitroglycerin under my tongue."

The patient should sit down before taking the nitroglycerin to decrease cardiac workload and prevent orthostatic hypotension.

After receiving the following information about four patients during change-of-shift report, which patient should the nurse assess first? b. Patient who has just returned to the unit after balloon valvuloplasty

The patient who has just arrived after balloon valvuloplasty will need assessment for complications such as bleeding and hypotension.

The patient had a history of rheumatic fever and has been diagnosed with mitral valve stenosis. The patient is planning to have a biologic valve replacement. What protective mechanisms should the nurse teach the patient about using after the valve replacement? B. Antibiotic prophylaxis for dental care

The patient will need to use antibiotic prophylaxis for dental care to prevent endocarditis.

When caring for a patient with acute coronary syndrome who has returned to the coronary care unit after having angioplasty with stent placement, the nurse obtains the following assessment data. Which data indicate the need for immediate action by the nurse? d. Chest pain level 7 on a 0 to 10 point scale

The patient's chest pain indicates that restenosis of the coronary artery may be occurring and requires immediate actions, such as administration of oxygen and nitroglycerin, by the nurse.

Two days after an acute myocardial infarction (MI), a patient complains of stabbing chest pain that increases with a deep breath. Which action will the nurse take first? a. Auscultate the heart sounds.

The patient's clinical manifestations and history are consistent with pericarditis, and the first action by the nurse should be to listen for a pericardial friction rub.

A patient with rheumatic fever has subcutaneous nodules, erythema marginatum, and polyarthritis. Based on these findings, which nursing diagnosis would be most appropriate? b. Activity intolerance related to arthralgia

The patient's joint pain will lead to difficulty with activity

A patient is recovering from a myocardial infarction (MI) and develops chest pain on day 3 that increases when taking a deep breath and is relieved by leaning forward. Which action should the nurse take next? c. Auscultate for a pericardial friction rub.

The patient's symptoms are consistent with the development of pericarditis, a possible complication of MI.

The nurse is examining the ECG of a patient who has just been admitted with a suspected myocardial infarction (MI). Which ECG change is most indicative of prolonged or complete coronary occlusion?

The presence of a pathologic Q wave, as often accompanies ST segment elevation myocardial infarction (STEMI), is indicative of complete coronary occlusion. Sinus tachycardia, fibrillatory P waves (e.g., atrial fibrillation), or a prolonged PR interval (first-degree heart block) are not direct indicators of extensive occlusion.

The nurse is examining the ECG of a patient who has just been admitted with a suspected MI. Which ECG change is most indicative of prolonged or complete coronary occlusion? b) Pathologic Q wave

The presence of a pathologic Q wave, as often accompanies STEMI, is indicative of complete coronary occlusion.

When caring for a patient with mitral valve stenosis, it is most important that the nurse asses for c. shortness of breath on exertion.

The pressure gradient changes in mitral stenosis lead to fluid backup into the lungs, resulting in hypoxemia and dyspnea.

D. Treatment of underlying cause. These therapies are usually effective for essential hypertension. These therapies are usually effective for essential hypertension. These therapies are usually effective for essential hypertension. Secondary hypertension is a result of an underlying disease process or structural abnormality. It is usually necessary to treat the problem before the hypertension will be resolved.

The primary therapy for secondary hypertension in children is: A. Weight reduction. B. Low-salt diet. C. Increased exercise and fitness. D. Treatment of underlying cause.

A patient who has chest pain is admitted to the emergency department (ED) and all of the following are ordered. Which one should the nurse arrange to be completed first? c. Electrocardiogram (ECG)

The priority for the patient is to determine whether an acute myocardial infarction (AMI) is occurring so that reperfusion therapy can begin as quickly as possible.

A patient with diabetes mellitus and chronic stable angina has a new order for captopril (Capoten). The nurse should teach the patient that the primary purpose of captopril is to c. prevent changes in heart muscle.

The purpose for angiotensin-converting enzyme (ACE) inhibitors in patients with chronic stable angina who are at high risk for a cardiac event is to decrease ventricular remodeling.

The nurse is performing medication teaching to a patient who has been prescribed sublingual (SL) nitroglycerin (NTG) tablets. Which statement by the patient indicates a need for further education?

The recommended dose for the patient for whom NTG has been prescribed is one tablet taken SL or one metered spray for symptoms of angina. If symptoms are unchanged or worse after five minutes, the patient should contact the emergency medical services (EMS) system before taking additional NTG. Tell the patient to place an NTG tablet under the tongue and allow it to dissolve. NTG should cause a tingling sensation when administered; otherwise it may be outdated. Warn the patient that a headache, dizziness, or flushing may occur. Caution the patient to change positions slowly after NTG use because orthostatic hypotension may occur.

After teaching a patient with chronic stable angina about nitroglycerin, the nurse recognizes the need for further teaching when the patient makes which statement? b) "I can take up to five tablets every 3 minutes for relief of my chest pain."

The recommended dose of nitroglycerin is one tablet taken sublingually (SL) or one metered spray for symptoms of angina.

D. Have someone call for an ambulance/paramedic rescue squad. Because severe respiratory distress is occurring, treatment of the response is indicated first. The cause of the response can be determined later. Diphenhydramine will not be effective for this type of allergic reaction. The child should not be moved unless the child is in a place that puts the child at greater hazard. Because the child is in severe respiratory distress, the nurse should remain with the child while someone else calls for the rescue squad.

The school nurse is called to the cafeteria because a child "has eaten something he is allergic to." The child is in severe respiratory distress. FIRST the nurse should: A. Determine what the child has eaten. B. Administer diphenhydramine (Benadryl). C. Move the child to the nurse's office or hallway. D. Have someone call for an ambulance/paramedic rescue squad.

The 80-year-old client is being discharged home after having surgery to debride a chronic venous ulcer on the right ankle. Which referral would be most appropriate for the client? Social worker.

The social worker would assess the client to determine if home health care services or financial interventions were appropriate for the client. The client is elderly, immobility is a concern, and wound care must be a concern when the client is discharged home.

The nurse is preparing to administer warfarin (Coumadin), an oral anticoagulant, to a client with a mechanical valve replacement. The client's International Normalized Ratio (INR) is 2.7. Which action should the nurse implement? 1. Administer the medication as ordered.

The therapeutic range for most clients' INR is 2-3, but for a client with a mechanical valve replacement it is 2-3.5. The medication should be given as ordered and not withheld.

A nurse is preparing a teaching plan regarding biological tissue valve replacement. Which of the following identifies a disadvantage of this type of valve replacement?

The valve has to be replaced frequently

A nurse is teaching a patient about valve replacement surgery. Which statement by the patient indicates an understanding of the benefit of an autograft replacement valve?

The valve is made from my own heart valve, and I will not need to take any blood thinning drugs when I am discharged."

The nurse documents pitting edema in the bilateral lower extremities of the client. What does this documentation mean?

There is excess fluid volume in the interstitial space in areas affected by gravity.

The client diagnosed with essential hypertension asks the nurse, "Why do I have high blood pressure?" Which response by the nurse would be most appropriate? "There is no specific cause for hypertension, but there are many known risk factors."

There is no known cause for essential hypertension, but many factors, both modi- fiable (obesity, smoking, diet) and nonmodifiable (family history, age, gender) are risk factors for essential hypertension.

The charge nurse is making shift assignments for the medical floor. Which client should be assigned to the most experienced registered nurse? 3. The client with an apical pulse rate of 116, a respiratory rate of 26, and a blood pressure of 94/62.

This client is exhibiting signs/symptoms of shock, which makes this client the most unstable. An experienced nurse should care for this client.

The client with coronary artery disease asks the nurse, "Why do I get chest pain?" Which statement would be the most appropriate response by the nurse? 1. "Chest pain is caused by decreased oxygen to the heart muscle."

This is a correct statement presented in layman's terms. When the coronary arteries cannot supply adequate oxygen to the heart muscle, there is chest pain.

The client receiving low molecular weight heparin (LMWH) subcutaneously to prevent DVT following hip replacement surgery complains to the nurse that there are small purple hemorrhaged areas on the upper abdomen. Which action should the nurse implement? 3. Explain that this results from the medication.

This is not hemorrhaging, and the client should be reassured that this is a side effect of the medication.

The client comes to the clinic complaining of muscle cramping and pain in both legs when walking for short periods of time. Which medical term would the nurse document in the client's record? 2. Intermittent claudication.

This is the classic symptom of arterial occlusive disease.

The nurse is teaching a class on atherosclerosis. Which statement describes the scientific rationale as to why diabetes is a risk factor for developing atherosclerosis? 3. Diabetes speeds the atherosclerotic process by thickening the basement membrane of both large and small vessels.

This is the scientific rationale why diabetes mellitus is a modifiable risk factor for atherosclerosis.

After receiving change-of-shift report about the following four patients, which patient should the nurse assess first? d. 59-year-old with unstable angina who has just returned to the unit after having a percutaneous coronary intervention (PCI)

This patient is at risk for bleeding from the arterial access site for the PCI, so the nurse should assess the patient's blood pressure, pulse, and the access site immediately. \

The client asks the nurse, "My doctor just told me that atherosclerosis is why my legs hurt when I walk. What does that mean?" Which response by the nurse would be the best response? 4. "You have a hardening of your arteries that decreases the oxygen to your legs."

This response explains in plain terms why the client's legs hurt from atherosclerosis.

A patient admitted to the hospital is suspected to have rheumatic endocarditis. What diagnostic test does the nurse anticipate will be ordered?

Throat culture

The school nurse is providing care to a child with a sore throat. With any sign of throat infection, the nurse stresses which of the following?

Throat culture

A patient with ST-segment elevation in three contiguous electrocardiographic (ECG) leads is admitted to the emergency department (ED) and diagnosed as having an ST-segment-elevation myocardial infarction (STEMI). Which question should the nurse ask to determine whether the patient is a candidate for thrombolytic therapy? c. "What time did your chest pain begin?"

Thrombolytic therapy should be started within 6 hours of the onset of the myocardial infarction (MI), so the time at which the chest pain started is a major determinant of the appropriateness of this treatment.

The chart of a 75-year-old female client indicates Virchow's triad. The nurse recognizes that this client is at risk for which of the following complications?

Thrombophlebitis Virchow's triad refers to venous stasis (slowed circulation), altered blood coagulation, and trauma to the vein, and predisposes clients to thrombosis and thrombophlebitis. The diagnosis of Virchow's triad indicates the client already presents with venous stasis. The diagnosis of Virchow's triad indicates the client already presents with altered blood coagulation. Postphlebitic syndrome is a vascular complication that may occur for up to 5 years after the initial episode with thrombophlebitis. This client does not yet have thrombophlebitis, so she is not at risk for postphlebitic syndrome.

The client diagnosed with peripheral vascular disease is overweight, has smoked two (2) packs of cigarettes a day for 20 years, and sits behind a desk all day. What is the strongest factor in the development of atherosclerotic lesions? 4. Smoking cigarettes.

Tobacco use is the strongest factor in the development of atherosclerotic lesions. Nicotine decreases blood flow to the extremities and increases heart rate and blood pressure. It also increases the risk of clot formation by increasing the aggregation of platelets.

When planning care for a patient hospitalized with a streptococcal infective endocarditis (IE), which intervention is a priority for the nurse to include? b. Arrange for placement of a long-term IV catheter.

Treatment for IE involves 4 to 6 weeks of IV antibiotic therapy in order to eradicate the bacteria, which will require a long-term IV catheter such as a peripherally inserted central catheter (PICC) line.

Which admission order written by the health care provider for a patient admitted with infective endocarditis (IE) and a fever would be a priority for the nurse to implement? b. Order blood cultures drawn from two sites.

Treatment of the IE with antibiotics should be started as quickly as possible, but it is essential to obtain blood cultures before initiating antibiotic therapy to obtain accurate sensitivity results.

A nurse is caring for a patient with chest pain that began 10 days ago. Which serum cardiac marker should the nurse review to determine if a myocardial infarction occurred 10 days ago?

Troponin is a serum cardiac marker that is detectable in the blood up to two weeks after myocardial injury and is used to diagnose a myocardial infarction. Troponin has two subtypes: cardiac-specific troponin T (cTnT) and cardiac-specific troponin I (cTnI). Serum levels of cTnT and cTnI begin to increase 4 to 6 hours after the onset of myocardial injury, peak at 10 to 24 hours, and return to normal over 10 to 14 days. Myoglobin is a protein found in skeletal and cardiac muscle. It is a sensitive indicator of early myocardial injury but is not specific for cardiac muscle; therefore, it is not used to diagnose a myocardial infarction. Myoglobin peaks and returns to normal in 3 to 15 hours. Homocysteine is a protein amino acid. High levels of homocysteine may indicate an increased risk for coronary artery disease. It is not used to diagnose myocardial infarction. Creatine kinase is a serum cardiac marker for myocardial injury but lacks specificity for myocardial damage. Serum levels peak at about 18 hours and return to normal within 24 to 36 hours.

Which cardiac enzyme would the nurse expect to elevate first in a client diagnosed with a myocardial infarction? 3. Troponin.

Troponin is the enzyme that elevates within 1 to 2 hours.

A patient who has had chest pain for several hours is admitted with a diagnosis of rule out acute myocardial infarction (AMI). Which laboratory test should the nurse monitor to help determine whether the patient has had an AMI? d. Cardiac-specific troponin

Troponin levels increase about 4 to 6 hours after the onset of myocardial infarction (MI) and are highly specific indicators for MI

Which cardiac enzyme would the nurse expect to elevate first in a client diagnosed with a myocardial infarction?

Troponin. - elvated within 1-2 hours - CK-MB within 12-24 - LDH within 24-36

When providing nutritional counseling for patients at risk for CAD, which foods would the nurse encourage patients to include in their diet (select all that apply)? a) Tofu b) Walnuts c) Tuna fish

Tuna fish, tofu, and walnuts are all rich in omega-3 fatty acids, which have been shown to reduce the risks associated with CAD when consumed regularly.

The nurse is caring for a patient with venous insufficiency. What should the nurse assess the patient's lower extremities for?

Ulceration

What are the symptoms a nurse should assess for in a patient with lymphedema as a result of impaired nutrition to the tissue?

Ulcers and infection in the edematous area

Cardiovascular efficiency diminished; microcirculatory perfusion marginal despite compensatory adjustments; tissue hypoxia, metabolic acidosis, and impairment of organ systems function.

Uncompensated shock

The client diagnosed with pericarditis is being discharged home. Which intervention should the nurse include in the discharge teaching? 1. Be sure to allow for uninterrupted rest and sleep.

Uninterrupted rest and sleep help decrease the workload of the heart and help ensure the restoration of physical and emotional health.

While caring for a 23-year-old patient with mitral valve prolapse (MVP) without valvular regurgitation, the nurse determines that discharge teaching has been effective when the patient states that it will be necessary to d. avoid use of over-the-counter (OTC) medications that contain stimulant drugs.

Use of stimulant medications should be avoided by patients with MVP because these may exacerbate symptoms.

In caring for the patient with angina, the patient states, "I walked to the bathroom. While I was having a bowel movement, I started having the worst chest pain ever, like before I was admitted. I called for a nurse, but the pain is gone now." What further assessment data should the nurse obtain from the patient?

Using PQRST, the assessment data not volunteered by the patient is the radiation (R) of pain, the area in which the patient felt the pain and if it radiated. The precipitating event (P) was going to the bathroom and having a bowel movement. The quality of the pain (Q) was "like before I was admitted," although a more specific description may be helpful. Severity of the pain (S) was the "worst chest pain ever," although an actual number may be needed. Timing (T) is supplied by the patient describing when the pain occurred and that the patient had had this pain previously.

In caring for the patient with angina, the patient said, "I walked to the bathroom. While I was having a bowel movement, I started having the worst chest pain ever, like before I was admitted. I called for a nurse, but the pain is gone now." What further assessment data should the nurse obtain from the patient? c) "In what areas did you feel this pain?"

Using PQRST, the assessment data not volunteered by the patient is the radiation of pain, the area the patient felt the pain, and if it radiated.

The first treatment strategy to try for supraventricular tachycardia (SVT); performed by applying ice to the face, massaging one carotid artery, or having the child exhale against a closed glottis.

Vagal Maneuvers

The client with varicose veins asks the nurse, "What caused me to have these?" Which statement by the nurse would be most appropriate? 1. "You have incompetent valves in your legs."

Varicose veins are irregular, tortuous veins with incompetent valves that do not allow the venous blood to ascend the saphenous vein.

Which client would be most at risk for developing varicose veins? 1. A Caucasian female who is a nurse.

Varicose veins are more common in white females in occupations that involve prolonged standing.

A pharmacologic intervention for slowing the disease progress of aortic regurgitation by reducing afterload is the purpose of:

Vasodilators

The nurse teaches the patient with peripheral vascular disease (PVD) to refrain from smoking because nicotine causes which of the following?

Vasospasm

The nurse is teaching a class on venous insufficiency. The nurse would identify which condition as the most serious complication of chronic venous insufficiency? Venous ulcerations.

Venous ulcerations are the most serious complication of chronic venous insufficiency. It is very difficult for these ulcerations to heal, and often clients must be seen in wound care clinics for treatment.

The client is in ventricular fibrillation. Which interventions should the nurse implement? Select all that apply. 1. Start cardiopulmonary resuscitation. 3. Prepare to defibrillate the client. 4. Bring the crash cart to the bedside. 5. Prepare to administer the antidysrhythmic amiodarone IVP.

Ventricular fibrillation indicates the client does not have a heartbeat. Therefore CPR should be instituted. Defibrillation is the treatment of choice for ventricular fibrillation. The crash cart has the defibrillator and is used when performing advanced cardiopulmonary resuscitation. Amiodarone is an antidysrhythmic that is used in ventricular dysrhythmias.

Which of the following clinical manifestations would the nurse expect to find in the patient diagnosed with aortic regurgitation?

Visible neck vein pulsations

The home health nurse is admitting a client diagnosed with a DVT. Which action by the client warrants immediate intervention by the nurse? 3. The client takes vitamin E over-the-counter medications.

Vitamin E can affect the action of warfarin. The nurse should explain to the client that these and other medications could potentiate the action of warfarin.

2. The nurse is teaching the client recently diagnosed with essential hypertension. Which instruction should the nurse provide when discussing exercise? Walk at least 30 minutes a day on flat surfaces.

Walking 30 to 45 minutes a day will help in reducing blood pressure, weight, and stress and will increase a feeling of overall wellbeing.

The nurse is teaching the client recently diagnosed with essential hypertension. Which instruction should the nurse provide when discussing exercise? Walk at least 30 minutes a day on flat surfaces.

Walking 30 to 45 minutes a day will help in reducing blood pressure, weight, and stress and will increase a feeling of overall wellbeing.

Which instruction should be included when a client diagnosed with peripheral arterial disease is being discharged? 3. Instruct the client to walk daily for at least 30 minutes.

Walking promotes the development of collateral circulation to ischemic tissue and slows the process of atherosclerosis.

The nurse understands that which of the following medications will be administered for 6 to 12 weeks following prosthetic porcine valve surgery?

Warfarin

D. Apprehension Confusion indicates uncompensated shock. Sleepiness is not an indication of shock. Hypotension is a symptom of irreversible shock. Apprehension indicates compensated shock.

What should the nurse recognize as an early clinical sign of compensated shock in a child? A. Confusion B. Sleepiness C. Hypotension D. Apprehension

C. Irritability D. Head rubbing E. Waking up screaming in the night Clinical manifestations of hypertension are: For adolescents and older children: · Frequent headaches · Dizziness · Changes in vision For infants or young children: · Irritability · Head banging or head rubbing · Waking up screaming in the night

When assessing for hypertension in an infant, the nurse will expect the infant to exhibit which signs? (Select all that apply.) A. Dizziness B. Changes in vision C. Irritability D. Head rubbing E. Waking up screaming in the night

The nurse is discussing the importance of exercise with the client diagnosed with coronary artery disease. Which intervention should the nurse implement? 3. Do not walk if it is less than 40F.

When it is cold outside, vasoconstriction occurs, and this will decrease oxygen to the heart muscle. Therefore, the client should not exercise when it is cold outside.

The nurse caring for a client with cardiomyopathy plans to have an education session with the client and her spouse about ways to improve cardiac output and reduce the workload of the heart. Which of the following instructions would help reduce preload, and therefore, reduce the workload of the heart?

When resting, sit up with the legs down to pool blood in the legs.

Which medical treatment would be prescribed for the client with an AAA less than 3 cm? 1. Ultrasound every six (6) months.

When the aneurysm is small (5-6 cm) an abdominal sonogram will be done every six (6) months until the aneurysm reaches a size at which surgery to prevent rupture is of more benefit than possible complications of an abdominal aortic aneurysm repair.

When evaluating the effectiveness of preoperative teaching with a patient scheduled for coronary artery bypass graft (CABG) surgery using the internal mammary artery, the nurse determines that additional teaching is needed when the patient says which of the following? b. "I will have small incisions in my leg where they will remove the vein."

When the internal mammary artery is used there is no need to have a saphenous vein removed from the leg.

A few days after experiencing a myocardial infarction (MI) and successful percutaneous coronary intervention, the patient states, "I just had a little chest pain. As soon as I get out of here, I'm going for my vacation as planned." Which reply would be most appropriate for the nurse to make? a. "What do you think caused your chest pain?"

When the patient is experiencing denial, the nurse should assist the patient in testing reality until the patient has progressed beyond this step of the emotional adjustment to MI.

D. Transposition of the great arteries Pulmonic stenosis is classified as an obstructive defect. Atrial septal defect is classified as a defect with increased pulmonary blood flow. Patent ductus arteriosus is classified as a defect with increased pulmonary blood flow. Transposition of the great arteries allows the mixing of blood in the heart.

Which is considered a mixed cardiac defect? A. Pulmonic stenosis B. Atrial septal defect C. Patent ductus arteriosus D. Transposition of the great arteries

Which type of graft is utilized when a heart valve replacement is made of tissue from an animal heart valve?

Xenograft

A client with a forceful, pounding heartbeat is diagnosed with mitral valve prolapse. Which client statement indicates to the nurse a need for additional teaching? a) "I can still drink coffee and tea." b) "I should increase my fluid intake." c) "I should eat foods rich in protein." d) "I'll enroll in an aerobic exercise program."

a) "I can still drink coffee and tea." Explanation: The client requires more teaching if he states that he may drink coffee and tea. Caffeine is a stimulant, which can exacerbate palpitations, and should be avoided by a client with symptomatic mitral valve prolapse. High fluid intake helps maintain adequate preload and cardiac output. Aerobic exercise helps increase cardiac output and decrease heart rate. Protein-rich foods aren't restricted but high-calorie foods are. pg.699

The nurse is caring for a client who is being discharged after insertion of a permanent pacemaker. Which question by the client indicates a need for clarification? a) "I should ask for a handheld device search when I go through airport security." b) "I should avoid contact sports." c) "I'll watch the incision for swelling or redness and will report if either occurs." d) "I should avoid large magnetic fields, such as an MRI machine or large motors."

a) "I should ask for a handheld device search when I go through airport security." Explanation: At security gates at airports, government buildings, or other secured areas, the client with a permanent pacemaker should show a pacemaker ID card and request a hand (not handheld device) search. The client should obtain and carry a physician's letter about this requirement. pg.720

A client is unconscious on arrival to the emergency department. The nurse in the emergency department identifies that the client has a permanent pacemaker due to which characteristic? a) "Spike" on the rhythm strip b) Quality of the pulse c) Scar on the chest d) Vibration under the skin

a) "Spike" on the rhythm strip Explanation: Confirmation that the client has a permanent pacemaker is the characteristic "spike" identified by a thin, straight stroke on the rhythm strip. The scar on the chest is suggestive of pacer implantation but not definitive. There should be no change in pulse quality, and no vibration under the skin. pg.718

A 26-year-old Air Force staff sergeant is returning for diagnostic follow-up to the cardiologist's office where you practice nursing. Her Holter monitor strip reveals a heart rate with normal conduction but with a rate consistently above 105 beats/minute. What other conditions can cause this response in a healthy heart? a) All options are correct b) Elevated temperature c) Shock d) Strenuous exercise

a) All options are correct Explanation: It occurs in clients with healthy hearts as a physiologic response to strenuous exercise, anxiety and fear, pain, fever, hyperthyroidism, hemorrhage, shock, or hypoxemia. There are a variety of causes that can create an elevated heart rate in an otherwise healthy heart. Fever is one cause. There are a variety of causes that can create an elevated heart rate in an otherwise healthy heart. Shock is one cause. There are a variety of causes that can create an elevated heart rate in an otherwise healthy heart. Strenuous exercise is one cause. pg.694

A patient's ECG tracing reveals a ventricular rate between 250 and 400, with saw-toothed P waves. The nurse correctly identifies this dysrhythmia as which of the following? a) Atrial flutter b) Atrial fibrillation c) Ventricular tachycardia d) Ventricular fibrillation

a) Atrial flutter Explanation: The nurse correctly identifies the ECG tracing as atrial flutter. Atrial flutter occurs in the atrium and creates impulses at a regular atrial rate between 250 and 400 times per minute. The P waves are saw-toothed in appearance. Atrial fibrillation causes a rapid, disorganized, and uncoordinated twitching of atrial musculature. The atrial rate is 300 to 600, and the ventricular rate is usually 120 to 200 in untreated atrial fibrillation. There are no discernible P waves. Ventricular fibrillation is a rapid, disorganized ventricular rhythm that causes ineffective quivering of the ventricles. The ventricular rate is greater than 300 per minute and extremely irregular, without a specific pattern. The QRS shape and duration is irregular, undulating waves without recognizable QRS complexes. Ventricular tachycardia is defined as three or more PVCs in a row, occurring at a rate exceeding 100 beats per minute. pg.703

Which of the following postimplantation instructions must a nurse provide a patient with a permanent pacemaker? a) Avoid sources of electrical interference b) Keep moving the arm on the side where the pacemaker is inserted c) Keep the arm on the side of the pacemaker higher than the head d) Delay for at least 3 weeks activities such as swimming and bowling

a) Avoid sources of electrical interference Explanation: The nurse must instruct the patient with a permanent pacemaker to avoid sources of electrical interference. The nurse should also instruct the patient to avoid strenuous movement (especially of the arm on the side where the pacemaker is inserted), to keep the arm on the side of the pacemaker lower than the head except for brief moments when dressing or performing hygiene, and to delay for at least 8 weeks activities such as swimming, bowling, tennis, vacuum cleaning, carrying heavy objects, chopping wood, mowing, raking, and shoveling snow. pg.717

The nurse is caring for a client who is displaying a third-degree AV block on the EKG monitor. The client is symptomatic due to the slow heart rate. The most appropriate nursing diagnosis for this client would be which of the following? a) Decreased cardiac output b) Ineffective health maintenance c) Ineffective breathing pattern d) Risk for vascular trauma

a) Decreased cardiac output Explanation: Based on assessment data for this client, the most logical nursing diagnosis will be decreased cardiac output. Third-degree AV block that is causing symptoms will be a slow rhythm that will produce a decreased cardiac output. pg.712

Your client has been diagnosed with an atrial dysrhythmia. The client has come to the clinic for a follow-up appointment and to talk with the physician about options to stop this dysrhythmia. What would be a procedure used to treat this client? a) Elective electrical cardioversion b) Elective electrical defibrillation c) Chemical cardioversion d) Mace procedure

a) Elective electrical cardioversion Explanation: Elective electrical cardioversion is a nonemergency procedure done by a physician to stop rapid, but not necessarily life-threatening, atrial dysrhythmias. Chemical cardioversion is not a procedure; it is drug therapy. A Mace procedure is a distractor for this question. Defibrillation is not an elective procedure. pg.715

You enter your client's room and find him pulseless and unresponsive. What would be the treatment of choice for this client? a) Immediate defibrillation b) Electric cardioversion c) Chemical cardioversion d) IV lidocaine

a) Immediate defibrillation Explanation: Defibrillation is used during pulseless ventricular tachycardia, ventricular fibrillation, and asystole (cardiac arrest) when no identifiable R wave is present. pg.716

A patient with dilated cardiomyopathy is having frequent episodes of ventricular fibrillation. What choice would be best to sense and terminate these episodes? a) Implantable cardioverter defibrillator b) Epinephrine c) Pacemaker d) Atropine

a) Implantable cardioverter defibrillator Explanation: The implantable cardioverter defibrillator (ICD) is an electronic device that detects and terminates life-threatening episodes of tachycardia or fibrillation, especially those that are ventricular in origin. Patients at high risk of ventricular tachycardia (VT) or ventricular fibrillation and who would benefit from an ICD are those who have survived sudden cardiac death syndrome, which usually is caused by ventricular fibrillation, or have experienced spontaneous, symptomatic VT (syncope secondary to VT) not due to a reversible cause (called a secondary prevention intervention). pg.721

The nurse is caring for a client who has just been diagnosed with sinus bradycardia. The client asks the nurse to explain what sinus bradycardia is. What would be the nurse's best explanation? a) In many clients a heart rate slower than 60 beats per minute is considered to slow to maintain an adequate cardiac output. b) Sinus bradycardia means your heart is not beating fast enough to keep you alive. c) Sinus bradycardia is nothing to worry about. d) In many clients a heart rate slower than 70 beats per minute is considered to slow to maintain an adequate cardiac output.

a) In many clients a heart rate slower than 60 beats per minute is considered to slow to maintain an adequate cardiac output. Explanation: A heart rate slower than 60 beats per minute is pathologic in clients with heart disorders, increased intracranial pressure, hypothyroidism, or digitalis toxicity. The danger in sinus bradycardia is that the slow rate may be insufficient to maintain cardiac output. Option B is incorrect as it is an incomplete answer to the client's question. Option C minimizes the client's concern so it is incorrect. Option D is incorrect as it gives the client incorrect information. pg.698

A patient is 2 days postoperative after having a permanent pacemaker inserted. The nurse observes that the patient is having continuous hiccups as the patient states, "I thought this was normal." What does the nurse understand is occurring with this patient? a) Lead wire dislodgement b) Faulty generator c) Fracture of the lead wire d) Sensitivity is too low

a) Lead wire dislodgement Explanation: Phrenic nerve, diaphragmatic (hiccuping may be a sign), or skeletal muscle stimulation may occur if the lead is dislocated or if the delivered energy (mA) is set high. The occurrence of this complication is avoided by testing during device implantation. pg.720

Which of the following nursing interventions must a nurse perform when administering prescribed vasopressors to a patient with a cardiac dysrhythmia? a) Monitor vital signs and cardiac rhythm b) Keep the patient flat for one hour after administration c) Document heart rate before and after administration d) Administer every five minutes during cardiac resuscitation

a) Monitor vital signs and cardiac rhythm Explanation: The nurse should monitor the patient's vital signs and cardiac rhythm for effectiveness of the medication and for side effects and should always have emergency life support equipment available when caring for an acutely ill patient. The side effects of vasopressor drugs are hypertension, dysrhythmias, pallor, and oliguria. It is not necessary to place a patient flat during or after vasopressor administration. When administering cholinergic antagonists, documentation of the heart rate is necessary.

Which of the following tends to be prolonged on the electrocardiogram (ECG) during a first-degree atrioventricular (AV) block? a) PR interval b) T wave c) P wave d) QRS

a) PR interval Explanation: First-degree AV block occurs when atrial conduction is delayed through AV node resulting in a prolonged PR interval. The QRS complex, T wave, and P wave are not prolonged in first-degree AV block. pg.711

A patient admitted to the telemetry unit has a serum potassium level of 6.6 mEq/L. Which of the following electrocardiographic (ECG) characteristics is commonly associated with this laboratory finding? a) Peaked T waves b) Flattened P waves c) Prolonged QT interval d) Occasional U waves

a) Peaked T waves Explanation: The patient's serum potassium level is high. The T wave is an ECG characteristic reflecting repolarization of the ventricles. It may become tall or "peaked" if a patient's serum potassium level is high. The U wave is an ECG waveform characteristic that may reflect Purkinje fiber repolarization. It is usually seen when a patient's serum potassium level is low. The P wave is an ECG characteristic reflecting conduction of an electrical impulse through the atria and is not affected by a patient's serum potassium level. The QT interval is an ECG characteristic reflecting the time from ventricular depolarization to repolarization, and is not affected by a patient's serum potassium level. pg.696

When the nurse observes an electrocardiogram (ECG) tracing on a cardiac monitor with a pattern in lead II and observes a bizarre, abnormal shape to the QRS complex, the nurse has likely observed which of the following ventricular dysrhythmias? a) Premature ventricular contraction (PVC) b) Ventricular fibrillation c) Ventricular bigeminy d) Ventricular tachycardia

a) Premature ventricular contraction (PVC) Explanation: A PVC is an impulse that starts in a ventricle before the next normal sinus impulse. Ventricular bigeminy is a rhythm in which every other complex is a PVC. Ventricular tachycardia is defined as three or more PVCs in a row, occurring at a rate exceeding 100 beats per minute. Ventricular fibrillation is a rapid but disorganized ventricular rhythm that causes ineffective quivering of the ventricles. pg.707

You are an operating room nurse caring for a client who is having a pacemaker implanted. The physician has requested a demand mode pacemaker for this client. What is this type of pacemaker? a) Self-activated b) A fixed-rate pacemaker c) A temporary pacemaker d) Asynchronous

a) Self-activated Explanation: Demand (synchronous) mode pacemakers self-activate when the client's pulse falls below a certain level. A fixed-rate pacemaker is asynchronous and permanent. Temporary pacemakers are used until a permanent pacemaker can be implanted. pg.718

The nurse is preparing to defibrillate a client with no breathing or pulse. Which nursing action precedes the nurse pressing the discharge button? a) Shouts, "All clear" b) States, "Charging" c) Placing gel on the chest d) Checking the ECG rhythm

a) Shouts, "All clear" Explanation: Preceding pressing the discharge button, the nurse shouts "All clear" to ensure that no one is in contact with the client. The other options are correct but not the nursing action immediately preceding. pg.716

A patient comes to the emergency department with complaints of chest pain after using cocaine. The nurse assesses the patient and obtains vital signs with results as follows: blood pressure 140/92, heart rate 128, respiratory rate 26, and an oxygen saturation of 98%. What rhythm on the monitor does the nurse anticipate viewing? a) Sinus tachycardia b) Ventricular tachycardia c) Normal sinus rhythm d) Sinus bradycardia

a) Sinus tachycardia Explanation: Sinus tachycardia occurs when the sinus node creates an impulse at a faster-than-normal rate. Causes include medications that stimulate the sympathetic response (e.g., catecholamines, aminophylline, atropine), stimulants (e.g., caffeine, nicotine), and illicit drugs (e.g., amphetamines, cocaine, Ecstasy). pg.699

Your patient is experiencing asymptomatic sinus tachycardia with a rate of 118. The nurse understands that the treatment of this condition includes: a) Treating the underlying cause b) Immediate defibrillation c) Administration of amiodarone d) Electrical cardioversion

a) Treating the underlying cause Explanation: Sinus tachycardia occurs in response to an underlying condition and will usually resolve once that condition is corrected. pg.699

The nurse is participating in the care of a client requiring emergent defibrillation. The nurse will complete the following steps in which order? a) Turn on the defibrillator and place it in "not sync" mode. b) Call "clear" three times ensuring patient and environmental safety. c) Deliver the prescribed electrical charge. d) Charge the defibrillator to the prescribed voltage. e) Apply the multifunction conductor pads to the patient's chest.

a) Turn on the defibrillator and place it in "not sync" mode. d) Charge the defibrillator to the prescribed voltage. e) Apply the multifunction conductor pads to the patient's chest. b) Call "clear" three times ensuring patient and environmental safety. c) Deliver the prescribed electrical charge. Explanation: This is the sequence of events the nurse should implement when delivering emergent defibrillation. If not followed correctly, the patient and health care team may be placed in danger. pg.716

The nurse is proving discharge instruction for a patient with a new arrhythmia. Which of the following should the nurse include? a) Your family and friends may want to take a CPR class. b) Do not be concerned if you experience symptoms of lightheadedness and dizziness. c) If you miss a dose of your antiarrhythmia medication, double up on the next dose. d) It is not necessary to learn how to take your own pulse.

a) Your family and friends may want to take a CPR class. Explanation: Having friends and family learn to take a pulse and perform CPR will help patients to manage their condition. Antiarrhythmic medication should be taken on time. Lightheadedness and dizziness are symptoms which should be reported to the provider. pg.714

A physician orders esmolol (Brevibloc) for a client with supraventricular tachycardia. During esmolol therapy, the nurse should monitor the client's: a) heart rate and blood pressure. b) ocular pressure. c) cerebral perfusion pressure. d) body temperature.

a) heart rate and blood pressure. Explanation: Because class II antiarrhythmics such as esmolol inhibit sinus node stimulation, they may produce bradycardia. Hypotension with peripheral vascular insufficiency also may occur, especially with esmolol. Class II antiarrhythmics don't alter body temperature, ocular pressure, or cerebral perfusion pressure. pg.701

An infant is receiving Lanoxin elixir 0.028mg once daily. Lanoxin is available in an elixir concentration of 50mcg/mL. The correct dose to draw up and administer is: a. 0.56 mL b. 0.28 mL c. 0.84 mL d. 1.12 mL

a. 0.56 mL

Prostaglandin is administered to the newborn with a congenital heart defect to: a. keep the ductus arteriosus open b. close the ductus arteriosus c. keep the foramen ovale open d. close the foramen ovale

a. keep the ductus arteriosus open

A nurse is caring for a client receiving warfarin (Coumadin) therapy following a mechanical valve replacement. The nurse completed the client's prothrombin time and International Normalized Ratio (INR) at 7 a.m., before the morning meal. The client had an INR reading of 4. The nurse's first priority should be to:

assess the client for bleeding around the gums or in the stool and notify the physician of the laboratory results and most recent administration of warfarin.

The nursing instructor is discussing pacemakers with her clinical group. One of the students is caring for a client with a transvenous pacemaker. One of the students asks why this client has a transvenous pacemaker. What would be the instructor's best response? a) "A transvenous pacemaker is used in place of a transarterial pacemaker." b) "A transvenous pacemaker is used to manage transient bradydysrhythmias like those that occur during acute MIs." c) "A transvenous pacemaker is used for a ventricular tachyarrhythmia." d) "A transvenous pacemaker is a permanent pacemaker that is asynchronous."

b) "A transvenous pacemaker is used to manage transient bradydysrhythmias like those that occur during acute MIs." Explanation: A transvenous pacemaker is a temporary pulse-generating device that sometimes is necessary to manage transient bradydysrhythmias such as those that occur during acute MIs or after coronary artery bypass graft surgery, or to override tachydysrhythmias. pg.717

Electrocardiogram (ECG) waveforms are printed on graph paper that is divided by light and dark vertical and horizontal lines at standard intervals. When the nurse is interpreting the heart rhythm, he or she understands that each large block equals how many seconds? a) 0.3 b) 0.2 c) 0.1 d) 0.4

b) 0.2 Explanation: Each small block on the graph paper equals 0.04 second, and five small blocks form a large block, which equals 0.2 second. pg.696

A patient tells the nurse "my heart is skipping beats again; I'm having palpitations." After completing a physical assessment, the nurse concludes the patient is experiencing occasional premature atrial complexes (PACs). The nurse should instruct the patient to complete which of the following? a) Apply supplemental oxygen. b) Avoid caffeinated beverages. c) Lie down and elevate the feet. d) Request sublingual nitroglycerin.

b) Avoid caffeinated beverages. Explanation: If PACs are infrequent, no medical interventions are necessary. Causes of PACs include caffeine, alcohol, nicotine, stretched atrial myocardium (e.g., as in hypervolemia), anxiety, hypokalemia (low potassium level), hypermetabolic states (e.g., with pregnancy), or atrial ischemia, injury, or infarction. The nurse should instruct the patient to avoid caffeinated beverages. pg.700

A nurse is providing morning care for a patient in the ICU. Suddenly, the bedside monitor shows ventricular fibrillation and the patient becomes unresponsive. After calling for assistance, what action should the nurse take next? a) Prepare for endotracheal intubation. b) Begin cardiopulmonary resuscitation. c) Provide electrical cardioversion. d) Administer intravenous epinephrine.

b) Begin cardiopulmonary resuscitation. Explanation: In the acute care setting, when ventricular fibrillation is noted, the nurse should call for assistance and defibrillate the patient as soon as possible. If defibrillation is not readily available, CPR is begun until the patient can be defibrillated, followed by advanced cardiovascular life support (ACLS) intervention, which includes endotracheal intubation and administration of epinephrine. Electrical cardioversion is not indicated for a patient in ventricular fibrillation. pg.709

A 78-year-old client was just admitted to the cardiac step-down unit where you practice nursing. Upon stabilizing his condition, you begin a conversation about his symptoms and you answer his questions to the best of your ability. In your discussion of cardiac dysrhythmias, which of the following would you rule out as a likely origination point for cardiac dysrhythmias? a) Ventricles b) Bundle of His c) Atria d) Atrioventricular node

b) Bundle of His Explanation: Cardiac dysrhythmias may originate in the atria, atrioventricular node, or ventricles. They do not originate in the Bundle of His. Cardiac dysrhythmias do not originate in the Bundle of His. pg.693

Two clients in cardiac rehabilitation are discussing the differences between scheduled cardioversion and unexpected defibrillation. Which difference will the nurse confirm? a) Both procedures sedate the clients. b) Cardioversion uses less electrical energy. c) Both used to eliminate ventricular dysrhythmias. d) Machine determines when electrical energy is delivered.

b) Cardioversion uses less electrical energy. Explanation: Cardioversion uses less electrical energy (50 to 100 joules) than defibrillation (200 to 360 joules). All of the other statements are correct. pg.716

A patient has had several episodes of recurrent tachydysrhythmias over the last 5 months and medication therapy has not been effective. What procedure should the nurse prepare the patient for? a) Insertion of a permanent pacemaker b) Catheter ablation therapy c) Insertion of an ICD d) Maze procedure

b) Catheter ablation therapy Explanation: Catheter ablation destroys specific cells that are the cause or central conduction route of a tachydysrhythmia. It is performed with or after an electrophysiology study. Usual indications for ablation are atrioventricular nodal reentry tachycardia, a recurrent atrial dysrhythmia (especially atrial fibrillation), or ventricular tachycardia unresponsive to previous therapy (or for which the therapy produced significant side effects). pg.726

Elective cardioversion is similar to defibrillation except that the electrical stimulation waits to discharge until an R wave appears. What does this prevent? a) Disrupting the heart during the critical period of atrial repolarization. b) Disrupting the heart during the critical period of ventricular repolarization. c) Disrupting the heart during the critical period of ventricular depolarization. d) Disrupting the heart during the critical period of atrial depolarization.

b) Disrupting the heart during the critical period of ventricular repolarization. Explanation: It is similar to defibrillation. One difference is that the machine that delivers the electrical stimulation waits to discharge until it senses the appearance of an R wave. By doing so, the machine prevents disrupting the heart during the critical period of ventricular repolarization. Therefore, options A, C, and D are incorrect. pg.716

The nurse is analyzing the electrocardiogram (ECG) strip of a stable patient admitted to the telemetry unit. The patient's ECG strip demonstrates PR intervals that measure 0.24 seconds. Which of the following is the nurse's most appropriate action? a) Apply oxygen via nasal cannula and obtain a 12-lead ECG. b) Document the findings and continue to monitor the patient. c) Instruct the patient to bear down as if having a bowel movement. d) Notify the patient's primary care provider of the findings.

b) Document the findings and continue to monitor the patient. Explanation: The patient's ECG tracing indicates a first-degree atrioventricular (AV) block. First-degree AV block rarely causes any hemodynamic effect; the other blocks may result in decreased heart rate, causing a decrease in perfusion to vital organs, such as the brain, heart, kidneys, lungs, and skin. The most appropriate action by the nurse is to document the findings and continue to monitor the patient. pg.711

Jack Johnson is a 58-year-old who's been living with an internal, fixed-rate pacemaker. You're checking his readings on a cardiac monitor and notice an absence of spikes. What should you do? a) Take Jack's blood pressure. b) Double-check the monitoring equipment. c) Nothing, there's no cause for alarm. d) Suggest the need for a new beta-blocker to the doctor.

b) Double-check the monitoring equipment. Explanation: One of the reasons for lack of pacemaker spikes is faulty monitoring equipment. It's important to be careful. One of the reasons for lack of pacemaker spikes is faulty monitoring equipment. Focus on the monitor. One of the reasons for lack of pacemaker spikes is faulty monitoring equipment. Check the monitor. One of the reasons for lack of pacemaker spikes is faulty monitoring equipment. pg.721

You are caring for a client with atrial fibrillation. What procedure would be recommended if drug therapies did not control the dysrhythmia? a) Pacemaker implantation b) Elective cardioversion c) Mace procedure d) Defibrillation

b) Elective cardioversion Explanation: Atrial fibrillation also is treated with elective cardioversion or digitalis if the ventricular rate is not too slow. Defibrillation is used for a ventricular problem. A Mace procedure is only a distractor for this question. Pacemakers are implanted for bradycardia. pg.704

A 65-year-old client has come to the emergency department reporting light-headedness, chest pain, and shortness of breath. As you finish your assessment, the physician enters and orders tests to ascertain what is causing the client's problems. In your client education, you explain the tests. Which test is used to identify cardiac rhythms? a) Electroencephalogram b) Electrocardiogram c) Electrocautery d) Echocardiogram

b) Electrocardiogram Explanation: An electrocardiogram is used to identify normal and abnormal cardiac rhythms. An electrocardiogram is the device used to identify normal and abnormal cardiac rhythms. pg.694

A patient has a persistent third-degree heart block and has had several periods of syncope. What priority treatment should the nurse anticipate for this patient? a) Administration of epinephrine b) Insertion of a pacemaker c) Insertion of an implantable cardioverter defibrillator (ICD) d) Administration of atropine

b) Insertion of a pacemaker Explanation: Third-degree AV block, also known as a complete block, occurs when no atrial impulse is conducted through the AV node into the ventricles. A permanent pacemaker may be necessary if the block persists. pg.713

The staff educator is teaching a class in dysrhythmias. What statement is correct for defibrillation? a) It uses less electrical energy than cardioversion. b) It is used to eliminate ventricular dysrhythmias. c) The client is sedated before the procedure. d) It is a scheduled procedure 1 to 10 days in advance.

b) It is used to eliminate ventricular dysrhythmias. Explanation: The only treatment for a life-threatening ventricular dysrhythmia is immediate defibrillation, which has the exact same effect as cardioversion, except that defibrillation is used when there is no functional ventricular contraction. It is an emergency procedure performed during resuscitation. The client is not sedated but is unresponsive. Defibrillation uses more electrical energy (200 to 360 joules) than cardioversion. pg.715

The nurse is analyzing a 6-second electrocardiogram (ECG) tracing. The P waves and QRS complexes are regular. The PR interval is 0.18 seconds long, and the QRS complexes are 0.08 seconds long. The heart rate is calculated at 70 bpm. The nurse correctly identifies this rhythm as which of the following? a) Junctional tachycardia b) Normal sinus rhythm c) Sinus tachycardia d) First-degree atrioventricular (AV) block

b) Normal sinus rhythm Explanation: The ECG tracing shows normal sinus rhythm (NSR). NSR has the following characteristics: ventricular and atrial rate: 60 to 100 beats per minute (bpm) in the adult; ventricular and atrial rhythm: regular; and QRS shape and duration: usually normal, but may be regularly abnormal; P wave: normal and consistent shape, always in front of the QRS; PR interval: consistent interval between 0.12 and 0.20 seconds and P:QRS ratio: 1:1. pg.698

The nurse is assessing vital signs in a patient with a permanent pacemaker. What should the nurse document about the pacemaker? a) Date and time of insertion b) Pacer rate c) Location of the generator d) Model number

b) Pacer rate Explanation: After a permanent pacemaker is inserted, the patient's heart rate and rhythm are monitored by ECG. pg.723

The nurse is teaching a beginning EKG class to staff nurses. As the nurse begins to discuss the the parts of the EKG complex, one of the students asks what the normal order of conduction through the heart is. The correct response would be which of the following? a) SA node, AV node, right and left bundle branches, bundle of His, and the Purkinje fibers b) Sinoatrial (SA) node, atrioventricular (AV) node, bundle of His, right and left bundle branches, and the Purkinje fibers c) SA node, AV node, bundle of His, the Purkinje fibers, and the right and left bundle branches d) AV node, SA node, bundle of His, right and left bundle branches, and the Purkinje fibers

b) Sinoatrial (SA) node, atrioventricular (AV) node, bundle of His, right and left bundle branches, and the Purkinje fibers Explanation: The correct sequence of conduction through the normal heart is the SA node, AV node, bundle of His, right and left bundle branches, and Purkinje fibers. pg.693

A 26-year-old client is returning for diagnostic follow-up. Her Holter monitor strip reveals a heart rate with normal conduction but with a rate consistently above 105 beats/minutes. What type of dysrhythmia would you expect the cardiologist to diagnose? a) Supraventricular bradycardia b) Sinus tachycardia c) Supraventricular tachycardia d) Sinus bradycardia

b) Sinus tachycardia Explanation: Sinus tachycardia is a dysrhythmia that proceeds normally through the conduction pathway but at a faster than usual rate (100 to 150 beats/minute). Sinus tachycardia is the dysrhythmia with a faster than usual heart rate (100 to 150 beats/minute). pg.699

A patient who had a myocardial infarction is experiencing severe chest pain and alerts the nurse. The nurse begins the assessment but suddenly the patient becomes unresponsive, no pulse, with the monitor showing a rapid, disorganized ventricular rhythm. What does the nurse interpret this rhythm to be? a) Ventricular tachycardia b) Ventricular fibrillation c) Atrial fibrillation d) Third-degree heart block

b) Ventricular fibrillation Explanation: The most common dysrhythmia in patients with cardiac arrest is ventricular fibrillation, which is a rapid, disorganized ventricular rhythm that causes ineffective quivering of the ventricles. No atrial activity is seen on the ECG. The most common cause of ventricular fibrillation is coronary artery disease and resulting acute myocardial infarction. Ventricular fibrillation is always characterized by the absence of an audible heartbeat, a palpable pulse, and respirations. pg.709

Premature ventricular contractions (PVCs) are considered precursors of ventricular tachycardia (VT) when they: a) have the same shape b) occur at a rate of more than six per minute c) are paired with a normal beat d) occur during the QRS complex

b) occur at a rate of more than six per minute Explanation: When PVCs occur at a rate of more than six per minute, they indicate increasing ventricular irritability and are considered forerunners of VT. PVCs are dangerous when they occur on the T wave. PVCs are dangerous when they are multifocal (have different shapes). A PVC that is paired with a normal beat is termed bigeminy. pg.708

The nurse is caring for a patient who is 2 days post-MI. The patient reports that she is experiencing chest pain. She states, "It hurts when I take a deep breath." Which action would be a priority?

b. Obtain vital signs and auscultate for a pericardial friction rub.

The two main angiotensin-converting enzyme (ACE) inhibitors most commonly used for children with congestive heart failure are: a. digoxin and captopril b. enalapril and captopril c. enalapril and furosemide d. spironolactone and captopril

b. enalapril and captopril

A 12-month-old would be classified as significantly hypertensive with a blood pressure that: a. falls above the 99th percentile one time. b. persistently falls between the 95th and 99th percentiles c. falls between the 95th and 99th percentiles one time d. falls below the 99th percentile one time.

b. persistently falls between the 95th and 99th percentiles

A 12-month-old infant in heart failure is taking enalapril (ACE inhibitor) and spironolactone. The nurse should be especially alert for: a. sodium 142 mEq/L b. potassium 5.0 mEq/L c. potassium 3.1 mEq/L d. sodium 132 mEq/L

b. potassium 5.0 mEq/L

The peak age for the incidence of Kawasaki disease is in the: a. infant age group b. toddle age group c. school-age group d. adolescent age group

b. toddle age group

A hospitalized patient with a history of chronic stable angina tells the nurse that she is having chest pain. The nurse bases his actions on the knowledge that ischemia: b. will be relieved by rest, nitroglycerin, or both

b. will be relieved by rest, nitroglycerin, or both

Slower than normal heart rate

bradycardia

A nurse is performing discharge teaching with a client who has an implantable cardioverter defibrillator (ICD) placed. Which client statement indicates effective teaching? a) "I need to stay at least 10? away from the microwave." b) "I have an appointment for magnetic resonance imaging of my knee scheduled for next week." c) "I'll keep a log of each time my ICD discharges." d) "I can't wait to get back to my football league."

c) "I'll keep a log of each time my ICD discharges." Explanation: The client stating that he should keep a log of all ICD discharges indicates effective teaching. This log helps the client and physician identify activities that may cause the arrhythmias that make the ICD discharge. He should also record the events right before the discharge. Clients with ICDs should avoid contact sports such as football. They must also avoid magnetic fields, which could permanently damage the ICD. Household appliances don't interfere with the ICD. pg.725

A nursing student is caring for one of the nurse's assigned cardiac clients. The student asks, "How can I tell the difference between sinus rhythm and sinus bradycardia when I look at the EKG strip" The best reply by the nurse is which of the following? a) "The QRS complex will be smaller in sinus bradycardia." b) "The P waves will be shaped differently." c) "The only difference is the rate, which will be below 60 bpm in sinus bradycardia." d) "The P-R interval will be prolonged in sinus bradycardia, and you will have to measure carefully to note the width."

c) "The only difference is the rate, which will be below 60 bpm in sinus bradycardia." Explanation: All characterestics of sinus bradycardia are the same as those of normal sinus rhythm, except for the rate, which will be below 60 in sinus bradycardia. pg.726

A patient is admitted to the emergency department (ED) with complaints of chest pain and shortness of breath. The nurse notes an irregular rhythm on the bedside electrocardiograph (ECG) monitor. The nurse counts 9 RR intervals on the patient's 6-second rhythm tracing. The nurse correctly identifies the patient's heart rate as which of the following? a) 70 bpm b) 100 bpm c) 90 bpm d) 80 bpm

c) 90 bpm Explanation: An alternative but less accurate method for estimating heart rate, which is usually used when the rhythm is irregular, is to count the number of RR intervals in 6 seconds and multiply that number by 10. The RR intervals are counted, rather than QRS complexes, because a computed heart rate based on the latter might be inaccurately high. The same methods may be used for determining atrial rate, using the PP interval instead of the RR interval. In this instance, 9 × 10 = 90. pg.697

The nurse is caring for clients on a telemetry unit. Which nursing consideration best represents concerns of altered rhythmic patterns of the heart? a) Altered patterns frequently cause a variety of home safety issues. b) Altered patterns frequently turn into life-threatening arrhythmias. c) Altered patterns frequently affect the heart's ability to pump blood effectively. d) Altered patterns frequently produce neurological deficits.

c) Altered patterns frequently affect the heart's ability to pump blood effectively. Explanation: The best representation of a nursing concern related to a cardiac arrhythmia is the inability of the heart to fill the chambers and eject blow flow efficiently. Lack of an efficient method to circulate blood and bodily fluids produces a variety of complications such as tissue ischemia, pulmonary edema, hypotension, decreased urine output, and impaired level of consciousness. The other options can occur with dysrhythmias, but the cause stemming from the altered pattern is the best answer. pg.693

The nurse is in the mall and observes a client slump to the floor. The nurse assesses the client and notes no pulse. The nurse calls for assistance to others in the mall and requests which piece of equipment? a) A cell phone to call 911 b) A stethoscope c) An automatic external defibrillator d) A blood pressure cuff

c) An automatic external defibrillator Explanation: Most malls in the United States now have automatic external defibrillators in common areas. These defibrillators can easily be applied and obtain electrical confirmation of no ventricular contraction or R wave. The machine allows an electrical stimulation when the discharge button is depressed. A blood pressure cuff and stethoscope will not provide the equipment needed to save the client's life. The 911 can be called by a bystander, but the priority is to obtain the life-saving equipment. If defibrillation is performed within the first 3 minutes of cardiac arrest, the potential for survival is 74%. pg.717

The nurse caring for a patient with a dysrhythmia understands that the P wave on an electrocardiogram (ECG) represents what phase of the cardiac cycle? a) Ventricular repolarization b) Ventricular depolarization c) Atrial depolarization d) Early ventricular repolarization

c) Atrial depolarization Explanation: The P wave represents atrial depolarization. The QRS complex represents ventricular depolarization. The T wave represents ventricular repolarization. The ST segment represents early ventricular repolarization, and lasts from the end of the QRS complex to the beginning of the T wave. pg.696

A client presents to the emergency department via ambulance with a heart rate of 210 beats/minute and a sawtooth waveform pattern per cardiac monitor. The nurse is most correct to alert the medical team of the presence of a client with which disorder? a) Asystole b) Ventricular fibrillation c) Atrial flutter d) Premature ventricular contraction

c) Atrial flutter Explanation: Atrial flutter is a disorder in which a single atrial impulse outside the SA node causes the atria to contract at an exceedingly rapid rate. The atrioventricular (AV) node conducts only some impulses to the ventricle, resulting in a ventricular rate slower than the atrial rate, thus forming a sawtooth pattern on the heart monitor. Asystole is the absence of cardiac function and can indicate death. Premature ventricular contraction indicates an early electric impulse and does not necessarily produce an exceedingly rapid heart rate. Ventricular fibrillation is the inefficient quivering of the ventricles and indicative of a dying heart. pg.702

Electrocardiogram (ECG) characteristics of atrial fibrillation include which of the following? a) P wave resent before each QRS b) Normal PR interval c) Atrial rate of 300 to 400 d) Regular rhythm

c) Atrial rate of 300 to 400 Explanation: ECG characteristics of atrial fibrillation include an atrial rate of 300 to 400, a nonmeasurable PR interval, irregular rhythm, and no discernible P waves. pg.703

The nurse is caring for a patient following the insertion of a permanent pacemaker. Which of the following discharge instructions are appropriate for the nurse to review with the patient? Select all that apply. a) Wear a medical alert noting the presence of a pacemaker. b) Avoid the usage of microwave ovens and electronic tools. c) Avoid handheld screening devices in airports. d) Refrain from walking through antitheft devices. e) Check pulse daily, reporting sudden slowing or increase.

c) Avoid handheld screening devices in airports. e) Check pulse daily, reporting sudden slowing or increase. a) Wear a medical alert noting the presence of a pacemaker. Explanation: Handheld screening devices used in airports may interfere with the pacemaker. Patients should be advised to ask security personnel to perform a hand search instead of using the handheld screening device. With a permanent pacemaker, the patient should be instructed initially to restrict activity on the side of implantation. Patients also should be educated to perform a pulse check daily and to wear or carry medical identification to alert personnel to the presence of the pacemaker. Patients should walk through antitheft devices quickly and avoid standing in or near these devices. Patients can safely use microwave ovens and electronic tools. pg.720

A 73-year-old client has returned to the postanesthesia care unit where you practice nursing. The client had a pacemaker implanted and it is your responsibility to begin client education upon his becoming alert. Which of the following postimplantation instructions must you provide to the client now that he has a permanent pacemaker? a) Keep moving the arm on the side where the pacemaker is inserted. b) Keep the arm on the side of the pacemaker higher than the head. c) Avoid sources of electrical interference. d) Delay for at least 3 weeks activities such as swimming and bowling.

c) Avoid sources of electrical interference. Explanation: The nurse must instruct the client with a permanent pacemaker to avoid sources of electrical interference, such as MRI devices, large industrial motors, peripheral nerve stimulators, etc. The main warning to a client with a pacemaker is to avoid sources of electrical interference. pg.720

The nurse in the intensive care unit (ICU) hears an alarm sound in the patient's room. Arriving in the room, the patient is unresponsive, without a pulse, and a flat line on the monitor. What is the first action by the nurse? a) Defibrillate with 360 joules (monophasic defibrillator) b) Administer atropine 0.5 mg c) Begin cardiopulmonary resuscitation (CPR) d) Administer epinephrine

c) Begin cardiopulmonary resuscitation (CPR) Explanation: Commonly called flatline, ventricular asystole (Fig. 26-19) is characterized by absent QRS complexes confirmed in two different leads, although P waves may be apparent for a short duration. There is no heartbeat, no palpable pulse, and no respiration. Without immediate treatment, ventricular asystole is fatal. Ventricular asystole is treated the same as PEA, focusing on high-quality CPR with minimal interruptions and identifying underlying and contributing factors. pg.710

A client has had a pacemaker inserted and is ready for discharge. The nurse is providing education about pacemaker safety. Which of the following are items that the nurse will be sure to address? Choose all that apply. a) Avoid large magnetic fields. b) Sit at least 12 feet from television sets. c) Carry a card identifying yourself as a pacemaker recipient. d) Monitor your pulse once a month. e) Do not spend time near a microwave oven.

c) Carry a card identifying yourself as a pacemaker recipient. a) Avoid large magnetic fields. Explanation: Recent pacemaker technology allows clients to safely use most household electronic appliances and devices, including microwave ovens, electric tools, and televisions. The client with a pacemaker should monitor his or her pulse daily. He or she should always carry medical identification of pacemaker use. The client should avoid large magnetic fields (eg, large motors, magnetic resonance imaging, arc welding, electrical substations). pg.720

After evaluating a client for hypertension, a physician orders atenolol (Tenormin), 50 mg P.O. daily. Which therapeutic effect should atenolol have? a) Decreased blood pressure with reflex tachycardia b) Decreased peripheral vascular resistance c) Decreased cardiac output and decreased systolic and diastolic blood pressure d) Increased cardiac output and increased systolic and diastolic blood pressure

c) Decreased cardiac output and decreased systolic and diastolic blood pressure Explanation: As a long-acting, selective beta1-adrenergic blocker, atenolol decreases cardiac output and systolic and diastolic blood pressure; however, like other beta-adrenergic blockers, it increases peripheral vascular resistance at rest and with exercise. Atenolol may cause bradycardia, not tachycardia. pg.701

The nurse is preparing a patient for upcoming electrophysiology (EP) studies and possible ablation for treatment of atrial tachycardia. Which of the following information should the nurse include? a) The procedure will occur in the operating room under general anesthesia. b) After the procedure, the arrhythmia will not recur. c) During the procedure, the arrhythmia will be reproduced under controlled conditions. d) The procedure takes less time than a cardiac catheterization.

c) During the procedure, the arrhythmia will be reproduced under controlled conditions. Explanation: During EP studies, the patient is awake and may experience symptoms related to the arrhythmia. EP studies do not always include ablation of the arrhythmia. pg.724

You are caring for a client who has premature ventricular contractions. What sign or symptom is observed in this client? a) Nausea b) Hypotension c) Fluttering d) Fever

c) Fluttering Explanation: Premature ventricular contractions usually cause a flip-flop sensation in the chest, sometimes described as "fluttering." Associated signs and symptoms include pallor, nervousness, sweating, and faintness. Symptoms of premature ventricular contractions are not nausea, hypotension, and fever. pg.702

Which of the following is a potential cause of premature ventricular complexes (PVCs)? a) Bradycardia b) Alkalosis c) Hypokalemia d) Hypovolemia

c) Hypokalemia Explanation: PVCs can be caused by cardiac ischemia or infarction, increased workload on the heart (eg, exercise, fever, hypervolemia, heart failure, tachycardia), digitalis toxicity, acidosis, or electrolyte imbalances, especially hypokalemia. pg.707

While assessing a client, the nurse finds a heart rate of 120 beats per minute. The nurse recalls that causes of sinus tachycardia include which of the following? a) Hypothyroidism and athletic training b) Vagal stimulation and sleep c) Hypovolemia and fever d) Digoxin and vagal stimulation

c) Hypovolemia and fever Explanation: Causes of sinus tachycardia include physiologic or psychological stress (acute blood loss, anemia, shock, hypovolemia, fever, and exercise). Vagal stimulation, sleep, hypothyroidism, athletic training, and Digoxin all will cause a slow heart rate. pg.698

A client has a medical diagnosis of an advanced AV block and is symptomatic due to a slow heart rate. With what initial treatment(s) should the nurse be prepared to assist? a) A maze procedure or IV bolus of furosemide b) Cardiac catheterization c) IV bolus of atropine or temporary pacing d) Cardioversion or IV bolus of dopamine

c) IV bolus of atropine or temporary pacing Explanation: The initial treatment of choice is an IV bolus of atropine. If the client does not respond to atropine, has advanced AV block, or has had an acute MI, temporary pacing may be started. A permanent pacemaker my be necessary if the block persists. pg.713

Which of the following nursing interventions is required to prepare a patient with cardiac dysrhythmia for an elective electrical cardioversion? a) Administer digitalis and diuretics 24 hours before cardioversion b) Facilitate CPR until the patient is prepared for cardioversion c) Instruct the patient to restrict food and oral intake d) Monitor blood pressure every 4 hours

c) Instruct the patient to restrict food and oral intake Explanation: The nurse should instruct the patient to restrict food and oral intake before the cardioversion procedure. Digitalis and diuretics are withheld for 24 to 72 hours before cardioversion. The presence of digitalis and diuretics in myocardial cells decreases the ability to restore normal conduction and increases the chances of a fatal dysrhythmia developing after cardioversion. When the patient is in cardiopulmonary arrest, the nurse should facilitate CPR until the patient is prepared for defibrillation and not for cardioversion. Monitoring blood pressure every 4 hours is not required to prepare a patient with cardiac dysrhythmia. pg.715

A 28-year-old female patient presents to the emergency department (ED) stating severe restlessness and anxiety. Upon assessment, the patient's heart rate is 118 bpm and regular, the patient's pupils are dilated, and the patient appears excitable. Which action should the nurse take next? a) Place the patient on supplemental oxygen. b) Prepare to administer a calcium channel blocker. c) Question the patient about alcohol and illicit drug use. d) Instruct the patient to hold her breath and bear down.

c) Question the patient about alcohol and illicit drug use. Explanation: The patient is experiencing sinus tachycardia. Since the patient's findings of tachycardia, dilated pupils, restlessness, anxiety, and excitability can indicate illicit drug use (cocaine), the nurse should question the patient about alcohol and illicit drug use. This information will direct the patient's plan of care. Causes of tachycardia include medications that stimulate the sympathetic response, stimulants, and illicit drugs. The treatment goals for sinus tachycardia is usually determined by the severity of symptoms and directed at identifying and abolishing its cause. The other interventions may be implemented, but determining the cause of the tachycardia is essential. pg.700

A patient has had an implantable cardioverter defibrillator inserted. What should the nurse be sure to include in the education of this patient prior to discharge? (Select all that apply.) a) The patient will have to schedule monthly chest x-rays to make sure the device is patent. b) The patient may have a throbbing pain that is normal c) Record events that trigger a shock sensation. d) Call for emergency assistance if feeling dizzy. e) Avoid magnetic fields such as metal detection booths.

c) Record events that trigger a shock sensation. d) Call for emergency assistance if feeling dizzy. e) Avoid magnetic fields such as metal detection booths. Correct Explanation: The nurse should instruct the patient to avoid large magnetic fields such as those created by magnetic resonance imaging, large motors, arc welding, electrical substations, and so forth. Magnetic fields may deactivate the device, negating its effect on a dysrhythmia. The patient should call 911 for emergency assistance if a feeling of dizziness occurs. The patient should maintain a log that records discharges of an implantable cardioverter defibrillator (ICD). Record events that precipitate the sensation of shock. This provides important data for the physician to use in readjusting the medical regimen. Throbbing pain is not normal and should be reported immediately. An initial x-ray is indicated prior to discharge, but monthly x-rays are unnecessary. pg.725

A 66-year-old female client is having cardiac diagnostic tests to determine the cause of her symptoms. In her follow-up visit to the cardiologist, she is told that she has a dysrhythmia at a rate slower than 60 beats/minute. What type of dysrhythmia did the tests reveal? a) Heart block b) Atrial bradycardia c) Sinus bradycardia d) None

c) Sinus bradycardia Explanation: Sinus bradycardia is a dysrhythmia that proceeds normally through the conduction pathway but at a slower than usual (≤60 beats/minute) rate. Sinus bradycardia is a slower than usual (≤60 beats/minute) heart rate. pg.698

The licensed practical nurse is co-assigned with a registered nurse in the care of a client admitted to the cardiac unit with chest pain. The licensed practical nurse is assessing the accuracy of the cardiac monitor, which notes a heart rate of 34 beats/minute. The client appears anxious and states not feeling well. The licensed practical nurse confirms the monitor reading. When consulting with the registered nurse, which of the following is anticipated? a) The registered nurse stating to hold all medication until the pulse rate returns to 60 beats/minute b) The registered nurse stating to administer Lanoxin (digoxin) c) The registered nurse administering atropine sulfate intravenously d) The registered nurse stating to administer all medications accept those which are cardiotonics

c) The registered nurse administering atropine sulfate intravenously Explanation: The licensed practical nurse and registered nurse both identify that client's bradycardia. Atropine sulfate, a cholinergic blocking agent, is given intravenously (IV) to increase a dangerously slow heart rate. Lanoxin is not administered when the pulse rate falls under 60 beats/minute. It is dangerous to wait until the pulse rate increases without nursing intervention or administering additional medications until the imminent concern is addressed. pg.699

Your client has just been diagnosed with a dysrhythmia. The client asks you to explain normal sinus rhythm. What would you explain are the characteristics of normal sinus rhythm? a) Impulse travels to the atrioventricular (AV) node in 0.15 to 0.5 seconds. b) Heart rate between 60 and 150 beats per minute. c) The sinoatrial (SA) node initiates the impulse. d) The ventricles depolarize in 0.5 seconds or less.

c) The sinoatrial (SA) node initiates the impulse. Explanation: The characteristics of normal sinus rhythm are heart rate between 60 and 100 beats per minute; the SA node initiates the impulse; the impulse travels to the AV node in 0.12 to 0.2 seconds; the ventricles depolarize in 0.12 seconds or less; and each impulse occurs regularly. pg.698

The nurse is working on a monitored unit assessing the cardiac monitor rhythms. Which waveform pattern needs attention first? a) Sustained asystole b) Supraventricular tachycardia c) Ventricular fibrillation d) Atrial fibrillation

c) Ventricular fibrillation Explanation: Ventricular fibrillation is called the rhythm of a dying heart. It is the rhythm that needs attention first because there is no cardiac output, and it is an indication for CPR and immediate defibrillation. Sustained asystole either is from death, or the client is off of the cardiac monitor. Supraventricular tachycardia and atrial fibrillation is monitored and reported to the physician but is not addressed first. pg.709

A patient with hypertension has a newly diagnosed atrial fibrillation. What medication does the nurse anticipate administering to prevent the complication of atrial thrombi? a) Adenosine (Adenocard) b) Atropine c) Warfarin (Coumadin) d) Amiodarone (Pacerone)

c) Warfarin (Coumadin) Explanation: Because atrial function may be impaired for several weeks after cardioversion, warfarin is indicated for at least 4 weeks after the procedure. Patients may be given amiodarone (Cordarone), flecainide (Tambocor), ibutilide (Corvert), propafenone (Rythmol), or sotalol (Betapace) prior to cardioversion to enhance the success of cardioversion and prevent relapse of the atrial fibrillation (Fuster, Rydén et al., 2011). pg.704

Which of the following congenital heart defects usually has the best prognosis? a. Tetralogy of Fallot b. Ventricular septal defect c. Atrial septal defect d. Hypoplastic left heart syndrome

c. Atrial septal defect

A patient is recovering from an uncomplicated MI. Which rehabilitation guideline is a priority to include in the teaching plan?

c. Begin an exercise program that aims for at least five 30-minute sessions per week

Fluid and nutritional guidelines for an infant with congestive heart failure rarely include: a. sodium restriction b. sodium supplements c. fluid restriction d. decreased caloric intake

c. fluid restriction

The calories are usually modified for an infant with congestive heart failure by: a. feeding every 2 hours b. increasing the volume of each feeding c. increasing the caloric density of the formula d. increasing the feeding duration to 1 hour

c. increasing the caloric density of the formula

The most common finding in individuals at risk for sudden cardiac death is:

c. left ventricular dysfunction

The electrolye most commonly depleted with diuretic therapy is: a. sodium b. chloride c. potassium d. magnesium

c. potassium

The nurse determines that a patient has a characteristic symptom of pericarditis. What symptom does the nurse recognize as significant for this diagnosis?

constant chest pain

A patient has had a successful heart transplant for end-stage heart disease. What immunosuppressant will be necessary for this patient to take to prevent rejection?

cyclosporine

Two nursing students are reading EKG strips. One of the students asks the instructor what the P-R interval represents. The correct response should be which of the following? a) "It shows the time it takes the AV node impulse to depolarize the atria and travel through the SA node." b) "It shows the time it takes the AV node impulse to depolarize the ventricles and travel through the SA node." c) "It shows the time it takes the AV node impulse to depolarize the septum and travel through the Purkinje fibers." d) "It shows the time needed for the SA node impulse to depolarize the atria and travel through the AV node."

d) "It shows the time needed for the SA node impulse to depolarize the atria and travel through the AV node." Explanation: The PR interval is measured from the beginning of the P wave to the beginning of the QRS complex and represents the time needed for sinus node stimulation, atrial depolarization, and conduction through the AV node before ventricular depolarization. In a normal heart the impulses do not travel backward. The PR interval does not include the time it take to travel through the Purkinje fibers. pg.696

The nurse receives a telephone call from a client with an implanted pacemaker who reports that his pulse is 68 beats per minute, but his pacemaker rate is set at 72 beats per minute. The best response by the nurse is which of the following? a) "This is okay as long as you are not having any symptoms." b) "Try walking briskly for about 5 minutes to see if that gets your heart rate to increase." c) "Don't worry. The pacemaker's rate is often higher than the client's actual heart rate." d) "Please come to the clinic right away so that we may interrogate the pacemaker to see if it is malfunctioning."

d) "Please come to the clinic right away so that we may interrogate the pacemaker to see if it is malfunctioning." Explanation: A client experiencing pacemaker malfunctioning may develop bradycardia as well as signs and symptoms of decreased cardiac output. The client should check the pulse daily and report immediately any sudden slowing or increasing of the pulse rate. This may indicate pacemaker malfunction. pg.720

The nurse is caring for a client who had a permanent pacemaker surgically placed yesterday and is now ready for discharge. Which statement made by the client indicates the need for more education. a) "I will call the doctor if my incision becomes swollen and red." b) "I will check my pulse every day and report to the doctor if the rate is below the pacemaker setting." c) "I will avoid any large magnets that may affect my pacemaker." d) "We will be getting rid of our microwave oven so it will not affect my pacemaker."

d) "We will be getting rid of our microwave oven so it will not affect my pacemaker." Explanation: Permanent pacemaker generators have filters that protect them from electrical interference from most household devices, motors, and appliances. pg.717

A home care nurse is visiting a left-handed client who has an implantable cardioverter-defibrillator (ICD) implanted in his left chest. The client tells the nurse how excited he is because he's planning to go rifle hunting with his grandson. How should the nurse respond? a) "You'll need to take an extra dose of your antiarrhythmic before you shoot." b) "Being that close to a rifle might make your ICD fire." c) "Enjoy your time with your grandson." d) "You can't shoot a rifle left-handed because the rifle's recoil will traumatize the ICD site."

d) "You can't shoot a rifle left-handed because the rifle's recoil will traumatize the ICD site." Explanation: The recoil from the rifle can damage the ICD, so the client should be warned against shooting a rifle with his left hand. Close proximity to a rifle won't cause the ICD to fire inadvertently. The client shouldn't take an extra dose of his antiarrhythmic. pg.724

The nurse is caring for a client who is being discharged after insertion of a permanent pacemaker. The client, an avid tennis player, is scheduled to play in a tournament in 1 week. What is the best advice the nurse can give related to this activity? a) "You should avoid tennis; basketball or football would be a good substitute." b) "You may resume all normal activity in 1 week; if you are used to playing tennis, you may proceed with this activity." c) "Cancel your tennis tournament and wait until fall, then try hockey; skating is much easier on pacemakers." d) "You will need to cancel this activity; you must restrict arm movement above your head for 2 weeks."

d) "You will need to cancel this activity; you must restrict arm movement above your head for 2 weeks." Explanation: It is important to restrict movement of the arm until the incision heals. The client should not raise the arm above the head for 2 weeks afterward to avoid dislodging the leads. The client must avoid contact sports (eg, basketball, football, hockey). pg.725

The nurse is caring for a client with atrial fibrillation. The client's symptoms started about 1 week ago, but he is just now seeking medical attention. The client asks the nurse why he has to wait several weeks before the cardioversion takes place. The best answer by the nurse is which of the following? a) "There is a long list of clients in line to be cardioverted." b) "We have to allow your heart to rest for a few weeks before it is stressed by the cardioversion." c) "The doctor wants to see if your heart will switch back to its normal rhythm by itself." d) "Your atrial chambers may contain blood clots now, so you must take an anticoagulant for a few weeks before the cardioversion."

d) "Your atrial chambers may contain blood clots now, so you must take an anticoagulant for a few weeks before the cardioversion." Explanation: Because of the high risk of embolization of atrial thrombi, cardioversion of atrial fibrillation that has lasted longer than 48 hours should be avoided unless the client has received warfarin for at least 3 to 4 weeks prior to cardioversion. pg.702

Which PR interval presents a first-degree heart block? a) 0.14 seconds b) 0.18 seconds c) 0.16 seconds d) 0.24 seconds

d) 0.24 seconds Explanation: In adults, the normal range for the PR is 0.12 to 0.20 seconds. A PR internal of 0.24 seconds would indicate a first-degree heart block. pg.711

The licensed practical nurse is setting up the room for a client arriving at the emergency department with ventricular arrhythmias. The nurse is most correct to place which of the following in the room for treatment? a) A suction machine b) Cardioversion equipment c) An ECG machine d) A defibrillator

d) A defibrillator Explanation: The nurse is most correct to place a defibrillator close to the client room if not in the room. The nurse realizes that clients with ventricular dysrhythmias are at a high risk for fatal heart dysrhythmia and death. A suction machine is used to remove respiratory secretions. Cardioversion is used in a planned setting for atrial dysrhythmias. An ECG machine records tracings of the heart for diagnostic purposes. Most clients with history of cardiac disorders have an ECG completed. pg.716

You are overseeing a 62-year-old who has started to exhibit dangerous PVCs in the cardiac postoperative unit. He's been given a bolus of lidocaine and is under continuous IV infusion, but serious side effects, including hypotension during administration, could occur. What should you be ready to do? a) Call for the doctor and just wait. b) Administer additional lidocaine. c) Prepare for defibrillation. d) Adjust the IV infusion.

d) Adjust the IV infusion. Explanation: Call for the physician while adjusting the IV infusion to the slowest possible rate until the physician can examine the patient. Call for the physician while adjusting the IV infusion to the slowest possible rate until the physician can examine the patient. Do not do anything else. Call for the physician while adjusting the IV infusion to the slowest possible rate until the physician can examine the patient. Stay focused on the IV. Call for the physician and while waiting, adjust the IV infusion to the slowest possible rate until the physician can examine the patient. pg.701

In looking at the ECG of a new patient, 43-year-old Mrs. Smith, you see what appears to be a P wave slightly different than normal. You're considering the possibility of premature atrial contractions (PAC). Which of the following questions will you ask her when taking her history? a) Caffeine b) Nicotine c) Hyperthyroidism or other metabolic disorders d) All options are correct

d) All options are correct Explanation: There are a number of causes of premature atrial contractions (PAC), which it is why it's so important to know and review a patient's complete history when examining for dysrhythmias. There are a number of causes of premature atrial contractions (PAC). pg.700

Which of the following medication classifications is more likely to be expected when the nurse is caring for a client with atrial fibrillation? a) Potassium supplement b) Diuretic c) Antihypertensive d) Anticoagulant

d) Anticoagulant Explanation: Clients with persistent atrial fibrillation are prescribed anticoagulation therapy to reduce the risk of emboli formation associated with ineffective circulation. The other options may be prescribed but not expected in most situations. pg.704

An 83-year-old resident in the long-term care facility where you practice nursing has an irregular heart rate of around 100 beats/minute. He also has a significant pulse deficit. What component of his history would produce such symptoms? a) Bundle branch block b) Heart block c) Atrial flutter d) Atrial fibrillation

d) Atrial fibrillation Explanation: In atrial fibrillation, several areas in the right atrium initiate impulses resulting in disorganized, rapid activity. The atria quiver rather than contract, producing a pulse deficit due to irregular impulse conduction to the AV node. The ventricles respond to the atrial stimulus randomly, causing an irregular ventricular heart rate, which may be too infrequent to maintain adequate cardiac output. Atrial fibrillation generally causes disorganized activity, irregular heart rates, and pulse deficits. It is not atrial flutter. Atrial fibrillation generally causes disorganized activity, irregular heart rates, and pulse deficits. It is not heart block. Atrial fibrillation generally causes disorganized activity, irregular heart rates, and pulse deficits. It is not bundle branch block. pg.704

The nurse is monitoring a patient in the postanesthesia care unit (PACU) following a coronary artery bypass graft, observing a regular ventricular rate of 82 beats/min and "sawtooth" P waves with an atrial rate of approximately 300 beat/min. How does the nurse interpret this rhythm? a) Ventricular fibrillation b) Atrial fibrillation c) Ventricular tachycardia d) Atrial flutter

d) Atrial flutter Explanation: Atrial flutter occurs because of a conduction defect in the atrium and causes a rapid, regular atrial rate, usually between 250 and 400 bpm and results in P waves that are saw-toothed. Because the atrial rate is faster than the AV node can conduct, not all atrial impulses are conducted into the ventricle, causing a therapeutic block at the AV node. This is an important feature of this dysrhythmia. If all atrial impulses were conducted to the ventricle, the ventricular rate would also be 250 to 400 bpm, which would result in ventricular fibrillation, a life-threatening dysrhythmia. Atrial flutter often occurs in patients with chronic obstructive pulmonary disease, pulmonary hypertension, valvular disease, and thyrotoxicosis, as well as following open heart surgery and repair of congenital cardiac defects (Fuster, Walsh et al., 2011). pg.702

Which medication is the drug of choice for sinus bradycardia? a) Pronestyl b) Cardizem c) Lidocaine d) Atropine

d) Atropine Explanation: Atropine is the medication of choice in treating symptomatic sinus bradycardia. Lidocaine treats ventricular dysrhythmias. Pronestyl treats and prevents atrial and ventricular dysrhythmias. Cardizem is a calcium channel blocker and treats atrial dysrhythmias. pg.699

Treatment of symptomatic bradycardia includes which of the following? a) Cardioversion b) Adenocard c) Lidocaine d) Atropine

d) Atropine Explanation: Treatment of symptomatic bradycardia includes transcutaneous pacing and atropine. Lidocaine may be used in the treatment of ventricular fibrillation. Cardioversion and Adenocard may be used in patients diagnosed with atrial flutter. pg.699

The treatment for symptomatic junctional rhythm is the same as for which of the following other heart rhythms? a) Atrial flutter b) Tachycardia c) Atrial fibrillation d) Bradycardia

d) Bradycardia Explanation: If symptomatic, the treatment is the same as for bradycardia: the patient may be treated with pacing (temporary or permanent), IV atropine, or epinephrine. pg.706

You are caring for a client who has been admitted to have a cardioverter defibrillator implanted. You would know that implanted cardioverter defibrillators are used in what clients? a) Clients with recurrent life-threatening bradycardias b) Clients with sinus tachycardia c) Clients with ventricular bradycardia d) Clients with recurrent life-threatening tachydysrhythmias

d) Clients with recurrent life-threatening tachydysrhythmias Explanation: The automatic implanted cardioverter defibrillator (AICD) is an internal electrical device used for selected clients with recurrent life-threatening tachydysrhythmias. Therefore, options A, B, and C are incorrect. pg.715

You are caring for a client who has been admitted to have a cardioverter defibrillator implanted. You would know that implanted cardioverter defibrillators are used in what clients? a) Clients with recurrent life-threatening bradycardias b) Clients with sinus tachycardia c) Clients with ventricular bradycardia d) Clients with recurrent life-threatening tachydysrhythmias

d) Clients with recurrent life-threatening tachydysrhythmias Explanation: The automatic implanted cardioverter defibrillator (AICD) is an internal electrical device used for selected clients with recurrent life-threatening tachydysrhythmias. Therefore, options A, B, and C are incorrect. pg.715

Which medication is indicated for the patient with atrial fibrillation who is at high risk for stroke? a) Plavix b) Lovenox c) Aspirin d) Coumadin

d) Coumadin Explanation: Warfarin (Coumadin) is indicated if the patient with atrial fibrillation is at high risk for stroke. Aspirin, Lovenox, and Plavix are not indicated. If immediate anticoagulation is necessary, the patient may be placed on heparin until the warfarin level is therapeutic. pg.705

After observing a code blue situation, a nursing student asks a member of the code team what the treatment of choice is for witnessed ventricular fibrillation. The best response by the nurse is which of the following? a) IV bolus of dobutamine b) Cardiac catheterization c) IV bolus of lidocaine d) Defibrillation

d) Defibrillation Explanation: Because there is no coordinated cardiac activity, cardiac arrest and death are imminent if the dysrhythmia is not corrected. Early defibrillation is critical to survival. pg.717

The nurse is observing the monitor of a patient with a first-degree atrioventricular (AV) block. What is the nurse aware characterizes this block? a) P waves hidden with the QRS complex b) An irregular rhythm c) A variable heart rate, usually fewer than 60 bpm d) Delayed conduction, producing a prolonged PR interval

d) Delayed conduction, producing a prolonged PR interval Explanation: First-degree AV block occurs when all the atrial impulses are conducted through the AV node into the ventricles at a rate slower than normal. Thus the PR interval is prolonged (>0.20 seconds). pg.711

A nurse is evaluating a client with a temporary pacemaker. The patient's ECG tracing shows each P wave followed by the pacing spike. The nurse's best response is which of the following? a) Reposition the extremity and turn the patient to left side. b) Obtain a 12-lead ECG and a portable chest x-ray. c) Check the security of all connections and increase the milliamperage. d) Document the findings and continue to monitor the patient.

d) Document the findings and continue to monitor the patient Explanation: Capture is a term used to denote that the appropriate complex is followed by the pacing spike. In this instance, the patient's temporary pacemaker is functioning appropriately; all Ps wave followed by an atrial pacing spike. The nurse should document the findings and continue to monitor the patient. Repositioning the patient, placing the patient on the left side, checking the security of all connections, and increasing the milliamperage are nursing interventions used when the pacemaker has a loss of capture. Obtaining a 12-lead ECG and chest x-ray are indicated when there is a loss of pacing-total absence of pacing spikes or when there is a change in pacing QRS shape. pg.718

The nurse is working on a telemetry unit, caring for a client who has been in a sinus rhythm for the past 2 days with a heart rate of 88 to 96 beats per minute. The client puts on the call light in the bathroom and reports severe dizziness. The telemetry shows a heart rate of 46 beats per minute. What should the nurse be prepared to do? a) Assist with a temporary pacemaker. b) Prepare the client for maze surgery. c) Send the client to the cardiac catheterization laboratory. d) Give an IV bolus of atropine.

d) Give an IV bolus of atropine. Explanation: Atropine 0.5 mg given rapidly as an intravenous bolus every 3 to 5 minutes to a maximum total dose of 3.0 mg is the medication of choice in treating symptomatic sinus bradycardia. pg.699

A 63-year-old client is in the cardiac step-down unit where you practice nursing. In your discussions about his condition, the client is puzzled as to what causes the heart to be an effective pump. Which of the following statements would you include in your response? a) Sufficient blood pressure b) Inherent rhythmicity of all muscle tissue c) Inherent electrons in muscle tissue d) Inherent rhythmicity of cardiac muscle tissue

d) Inherent rhythmicity of cardiac muscle tissue Explanation: Cardiac rhythm refers to the pattern (or pace) of the heartbeat. The conduction system of the heart and the inherent rhythmicity of cardiac muscle produce a rhythm pattern, which greatly influences the heart's ability to pump blood effectively. pg.693

What nursing interventions could you institute with a client who has a suspected dysrhythmia that would help detect life-threatening dysrhythmias and would manage and minimize any that occur? a) Palpate the client's pulse and observe the client's response. b) Provide supplemental oxygen. c) Monitor blood pressure continuously. d) Monitor cardiac rhythm continuously.

d) Monitor cardiac rhythm continuously. Explanation: The nurse should monitor cardiac rhythm continuously. Cardiac monitors display real-time heart rate and rhythm and alert the nurse to potentially life-threatening dysrhythmias. Monitoring blood pressure continuously and palpating the client's pulse do not help detect life-threatening dysrhythmias. Providing supplemental oxygen helps maintain adequate cardiac output and does not help detect life-threatening dysrhythmias. pg.692

A nursing instructor is reviewing the parts of an EKG strip with a group of students. One student asks about the names of all the EKG cardiac complex parts. Which of the following items are considered a part of the cardiac complex on an EKG strip? Choose all that apply. a) T wave b) P-R interval c) QRT wave d) P wave e) S-Q segment

d) P wave b) P-R interval a) T wave Explanation: The EKG cardiac complex waves include the P wave, the QRS complex, the T wave, and possibly the U wave. The intervals and segments include the PR interval, the ST segment, and the QT interval. pg.696

Two days after discharge following a permanent pacemaker insertion, the client returns to the clinic for a follow-up appointment. He reports tenderness and throbbing around the incision. The nurse observes mild swelling, erythema, and warmth at the site and suspects which of the following: a) Postoperative site hematoma b) Normal postoperative healing c) Internal bleeding at pacemaker site d) Pacemaker site infection

d) Pacemaker site infection Explanation: Postoperative care for a pacemaker insertion includes observing for symptoms of infection. These include swelling, unusual tenderness, drainage, and increased warmth. pg.723

The nurse is analyzing the electrocardiogram (ECG) tracing of a client newly admitted to the cardiac step-down unit with a diagnosis of chest pain. Which of the following findings indicate the need for follow-up? a) QRS complex that is 0.10 seconds long b) ST segment that is isoelectric in appearance c) PR interval that is 0.18 seconds long d) QT interval that is 0. 46 seconds long

d) QT interval that is 0. 46 seconds long Explanation: The QT interval that is 0.46 seconds long needs to be investigated. The QT interval is usually 0.32 to 0.40 seconds in duration if the heart rate is 65 to 95 bpm. If the QT interval becomes prolonged, the patient may be at risk for a lethal ventricular dysrhythmia called torsades de pointes. The other findings are normal. pg.696

John, an 82-year-old retired librarian is brought into the ED where you practice nursing. The client's heart rate is greater than 155 beats/minute. As you connect him to the ECG, you notice that his rhythm is regular, rate is 162 beats/minute, and diastole is shortened. He is intermittently alert and reports chest pain. P waves cannot be identified. What condition would you expect the physician to diagnose? a) Atrial flutter b) Sinus tachycardia c) Heart block d) Supraventricular tachycardia

d) Supraventricular tachycardia Explanation: Supraventricular tachycardia (SVT) is a dysrhythmia in which the heart rate has a consistent rhythm but beats at a dangerously high rate (≥150 beats/minute). P waves cannot be identified on the ECG. Diastole is shortened and the heart does not have sufficient time to fill. Supraventricular tachycardia (SVT) is a dysrhythmia in which the heart rate has a consistent rhythm but a dangerously high heartbeat (≥150 beats/minute). It is not sinus tachycardia. Supraventricular tachycardia (SVT) is a dysrhythmia in which the heart rate has a consistent rhythm but a dangerously high heartbeat (≥150 beats/minute). It is not heart block. Supraventricular tachycardia (SVT) is a dysrhythmia in which the heart rate has a consistent rhythm but a dangerously high heartbeat (≥150 beats/minute). It is not atrial flutter. pg. 707

When the nurse observes that the patient's heart rate increases during inspiration and decreases during expiration, the nurse reports that the patient is demonstrating a) sinus bradycardia. b) normal sinus rhythm. c) sinus tachycardia. d) sinus dysrhythmia.

d) sinus dysrhythmia. Explanation: Sinus dysrhythmia occurs when the sinus node creates an impulse at an irregular rhythm. Normal sinus rhythm occurs when the electrical impulse starts at a regular rate and rhythm in the SA node and travels through the normal conduction pathway. Sinus bradycardia occurs when the sinus node regularly creates an impulse at a slower-than-normal rate. Sinus tachycardia occurs when the sinus node regularly creates an impulse at a faster-than-normal rate. pg.700

The nurse is teaching a mother how to administer digoxin (Lanoxin) at home to her 3-year-old child. The nurse tells the mother that as a general rule, digoxin should be administered to the older child whose pulse is: a. 108 b. 98 c. 78 d. 68

d. 68

Discharge teaching for a child with Kawasaki disease who received IVIG should include: a. Peeling of the hands and feet should be reported immediately. b. Arthritis, especially in the weight-bearing joints, should be reported immediately. c. Defer measles, mumps, and rubella vaccine for 11 months. d. All of the above should be included in the instructions.

d. All of the above should be included in the instructions.

Which of the following clinical manifestations is a sign of chronic hypoxemia in a child? a. squatting b. polycythemia c. clubbing d. a, b,and c

d. a, b,and c

The test that provides the most reliable evidence of recent streptococcal infection is the: a. throat culture b. Mantoux test c. liver enzymes test d. antistreptolysin O test

d. antistreptolysin O test

The nutritional needs of the infant with congestive heart failure are usually: a. the same as an adult's b. less than a healthy infant's c. the same as a healthy infant's d. greater than a healthy infant's

d. greater than a healthy infant's

If bleeding occurs at the insertion site after a cardiac catheterization, the nurse should apply; a. warmth to the unaffected extremity b. pressure below the insertion site c. warmth to the affected extremity. d. pressure above the insertion site.

d. pressure above the insertion site.

Signs and symptoms of supraventricular tachycardia (SVT) in an infant or young child include: a. pallor b. irritability c. poor feeding d. diaphoresis e. a,b, and c.

e. a,b, and c.

The client had open-heart surgery to replace the mitral valve. Which intervention should the intensive care unit nurse implement? 1. Restrict the client's fluids as ordered.

fluid intake may be restricted to reduce the cardiac workload and pressures within the heart and pulmonary circuit.

The client is exhibiting ventricular tachycardia. Which intervention should the nurse implement first?

left off at 81 page at question 55

Incomplete closure of the mitral valve results in backflow of blood from the:

left ventricle to left atrium

A 72-year-old man with a history of aortic stenosis is admitted to the emergency department. He reports severe left-sided chest pressure radiating to the jaw. Which medication, if ordered by the health care provider, should the nurse question? C. Nitroglycerin

nitroglycerin should be used cautiously or avoided in patients with aortic stenosis as a significant reduction in blood pressure may occur. Chest pain can worsen because of a drop in blood pressure.

The nurse is caring for a client with aortic regurgitation. The nurse knows to expect which of the following symptoms on physical examination?

orthopnea and dyspnea

What is the most common cause of mitral stenosis?

rheumatic endocarditis

It is important for a nurse to be aware of the normal hemodynamics of blood flow to recognize and understand pathology when it occurs. The nurse should know that incomplete closure of the tricuspid valve results in a backward flow of blood from the:

right ventricle to right atrium

A patient with infective endocarditis of a prosthetic mitral valve returns to the emergency department with a second episode of left-sided weakness and visual changes. The nurse expects that collaborative management of the patient will include

surgical valve replacement

Faster than normal heart rate

tachycardia

The client teaching instructions for a 57-year-old male client with thrombophlebitis who is being discharged should include which of the following? Select all that apply.

• Ambulate as tolerated. • Avoid sitting for too long. • Perform leg exercises each hour.


Ensembles d'études connexes

Diabetes and Nutrition Nursing 1 Lone Star North Harris

View Set

Chapter 12: Hardware and Network Troubleshooting

View Set

Anatomy & Physiology Chapter 1 & 2 Review

View Set

ELA 7 - The Writing Process Quiz

View Set

БІБЛІОТЕКОЗНАВСТВО

View Set

Open economy: international trade and finance: Unit test

View Set

Chapter 6: Naturalistic Observation

View Set

Financial Systems FIN 3113 Test#2 Learn Smart/HW Mississippi State University Whitledge

View Set